You are on page 1of 140

1

केन्द्रीय विद्यालय संगठन


KENDRIYA VIDYALAYA SANGATHAN
जयपुर संभाग / JAIPUR REGION
STUDY MATERIAL SESSION (2022-23)
CLASS XI MATHEMATICS

CHIEF PATRON
Sh. B.L. MORODIA
DEPUTY COMMISSIONER

PATRON PATRON
Sh. D R MEENA Sh. M R RAWAL
ASSISTANT COMMISSIONER ASSISTANT COMMISSIONER

CO-ORDINATOR
Sh ATUL VYAS
PRINCIPAL, KV , BHARATPUR
2
CONTENT TEAM MEMBERS (PGTs)

RAJEEV Dr B S PREM SINGH HEM SINGH CHETAN JAIN


KUMAR GUPTA INDOLIA SHEKHAWAT KV KV
KV No-5, KV No-3, KV No-1, BHARATPUR DEOLI
JAIPUR JAIPUR ALWAR

INDEX
SN TOPICS/ CHAPTER PAGES
1 SYLLABUS 4-6
2 SETS 6-10
3 RELATIONS AND FUNCTIONS 11-15
4 TRIGONOMETRIC FUNCTIONS 16-23
5 COMPLEX NUMBERS 24-30
6 LINEAR INEQUALITIES 31-34
7 PERMUTATIONS AND COMBINATIONS 35-39
8 BINOMIAL THEOREM 40-43
9 SEQUENCES AND SERIES 44-50
10 STRAIGHT LINES 51-58
11 CONIC SECTIONS 59-65
12 INTRODUCTION TO THREE-DIMENSIONAL 66-69
GEOMETRY
13 LIMITS AND DERIVATIVES 70-78
14 STATISTICS 79-86
15 PROBABILITY 87-91
16 BLUE PRINT OF SQP 92
17 SAMPLE QUESTION PAPER-1 93-101
18 SAMPLE QUESTION PAPER-2 102-111
19 SAMPLE QUESTION PAPER-3 112-119
20 SAMPLE QUESTION PAPER-4 120-128
21 SAMPLE QUESTION PAPER-5 129-140

3
CLASS XI (2022-23)
One Paper Total Period–240 [35 Minutes each]
Three Hours Max Marks: 80
No. Units No. of Periods Marks
I Sets and Functions 60 23
II Algebra 50 25
III Coordinate Geometry 50 12
IV Calculus 40 8
V Statistics and Probability 40 12
TOTAL 240 80
Internal Assessment 20
*No chapter/unit-wise weightage. Care to be taken to cover all the chapters.
Unit-I: Sets and Functions
1.Sets: (20) Periods
Sets and their representations, Empty set, Finite and Infinite sets, Equal sets, Subsets,
Subsets of a set of real numbers especially intervals (with notations). Universal set. Venn
diagrams. Union and Intersection of sets. Difference of sets. Complement of a set.
Properties of Complement.
2. Relations & Functions: (20) Periods
Ordered pairs. Cartesian product of sets. Number of elements in the Cartesian product of
two finite sets. Cartesian product of the set of reals with itself (up to R x R x R). Definition
of relation, pictorial diagrams, domain, co-domain and range of a relation. Function as a
special type of relation. Pictorial representation of a function, domain, co-domain and
range of a function. Real valued functions, domain and range of these functions, constant,
identity, polynomial, rational, modulus, signum, exponential, logarithmic and greatest
integer functions, with their graphs. Sum, difference, product and quotients of functions.
3. Trigonometric Functions: (20) Periods
Positive and negative angles. Measuring angles in radians and in degrees and conversion
from one measure to another. Definition of trigonometric functions with the help of unit
circle. Truth of the identity sin2x + cos2x = 1, for all x. Signs of trigonometric functions.
Domain and range of trigonometric functions and their graphs. Expressing sin (x ± y) and
cos (x ± y) in terms of sin x, sin y, cos x & cos y and their simple applications. Deducing
identities like the following:
tan(x ± y) = (tan x ± tan y) /(1 ∓ tan x tan y) , cot(x ± y) = (cot x cot y ∓ 1)/( cot y ± cot x)
sinα ± sinβ = 2sin (α ± β)/2 cos (α ∓ β)/2
cosα + cosβ = 2cos (α + β)/2 cos (α − β)/2

4
𝑐𝑜𝑠𝛼 − 𝑐𝑜𝑠𝛽 = −2𝑠𝑖𝑛(𝛼 + 𝛽)/2 𝑠𝑖𝑛(𝛼 − 𝛽)/2
Identities related to sin2x, cos2x, tan2 x, sin3x, cos3x and tan3x.
Unit-II: Algebra
1. Complex Numbers and Quadratic Equations: (10) Periods
Need for complex numbers, especially√−1, to be motivated by inability to solve some of
the quadratic equations. Algebraic properties of complex numbers. Argand plane
2. Linear Inequalities : (10) Periods
Linear inequalities. Algebraic solutions of linear inequalities in one variable and their
representation on the number line.
3. Permutations and Combinations: (10) Periods
Fundamental principle of counting. Factorial n. (n!) Permutations and combinations,
derivation of Formulae for nPr and nCr and their connections, simple applications.
4. Binomial Theorem: (10) Periods
Historical perspective, statement and proof of the binomial theorem for positive integral
indices. Pascal’s triangle, simple applications.
5. Sequence and Series: (10) Periods
Sequence and Series. Arithmetic Mean (A.M.) Geometric Progression (G.P.), general term
of a G.P., sum of n terms of a G.P., infinite G.P. and its sum, geometric mean (G.M.), relation
between A.M. and G.M.
Unit-III: Coordinate Geometry
1. Straight Lines: (15) Periods
Brief recall of two-dimensional geometry from earlier classes. Slope of a line and angle
between two lines. Various forms of equations of a line: parallel to axis, point -slope form,
slope-intercept form, two-point form, intercept form, Distance of a point from a line.
2. Conic Sections: (25) Periods
Sections of a cone: circles, ellipse, parabola, hyperbola, a point, a straight line and a pair
of intersecting lines as a degenerated case of a conic section. Standard equations and
simple properties of parabola, ellipse and hyperbola. Standard equation of a circle.
3. Introduction to Three-dimensional Geometry: (10) Periods
Coordinate axes and coordinate planes in three dimensions. Coordinates of a point.
Distance between two points.
Unit-IV: Calculus
1. Limits and Derivatives: (40) Periods

5
Derivative introduced as rate of change both as that of distance function and
geometrically. Intuitive idea of limit. Limits of polynomials and rational functions
trigonometric, exponential and logarithmic functions. Definition of derivative relate it to
scope of tangent of the curve, derivative of sum, difference, product and quotient of
functions. Derivatives of polynomial and trigonometric functions.
Unit-V Statistics and Probability
1. Statistics: (20) Periods
Measures of Dispersion: Range, Mean Deviation, Variance and Standard Deviation of
ungrouped/grouped data.
2. Probability: (20) Periods
Events; occurrence of events, ‘not’, ‘and’ and ‘or’ events, exhaustive events, mutually
exclusive events, Axiomatic (set theoretic) probability, connections with other
theories of earlier classes. Probability of an event, probability of ‘not’, ‘and’ and ‘or’
events.

SET THEORY

A set is a well-defined collection of objects.


Some example
N : the set of all natural numbers
Z : the set of all integers
Q : the set of all rational numbers
R : the set of real numbers
Z + : the set of positive integers
Q+ : the set of positive rational numbers, and
R+ : the set of positive real numbers.
Note:-
(i) Objects, elements and members of a set are synonym terms.
(ii) Sets are usually denoted by capital letters A, B, C, X, Y, Z, etc.
(iii) The elements of a set are represented by small letters a, b, c, x, y, z, etc. If a
is an element of a set A, we say that “ a belongs to A” the Greek symbol ∈ (epsilon)
is used to denote the phrase ‘belongs to’. Thus, we write a ∈ A. If ‘b’ is not an
element of a set A, we write b ∉ A and read “b does not belong to A”. Thus, in the
set V of vowels in the English alphabet, a ∈ V but b ∉ V. In the set P of prime
factors of 30, 3 ∈ P but 15 ∉ P.
There are two methods of representing a set :
(i) In roster form, all the elements of a set are listed, the elements are
being separated by commas and are enclosed within braces { }. For
example, the set of all even positive integers less than 7 is described
in roster form as {2, 4, 6}.
(ii) In set-builder form, all the elements of a set possess a single common
property which is not possessed by any element outside the set. For
example, in the set {a, e, i, o, u}, all the elements possess a common
property, namely, each of them is a vowel in the English alphabet, and

6
no other letter possess this property. Denoting this set by V, we write
V = {x : x is a vowel in English alphabet}
➢ Note In roster form, the order in which the elements are listed is
immaterial. Thus, the above set can also be represented as {1, 3, 7, 21, 2,
6, 14, 42}.
➢ It may be noted that while writing the set in roster form an element is
not generally repeated, i.e., all the elements are taken as distinct. For
example, the set of letters forming the word ‘SCHOOL’ is { S, C, H, O, L} or
{H, O, L, C, S}. Here, the order of listing elements has no relevance
The Empty Set
A set which does not contain any element is called the empty set or the null
set or the void set. According to this definition, B is an empty set while A is
not an empty set. The empty set is denoted by the symbol φ or { }.
Finite and Infinite Sets
A set which is empty or consists of a definite number of elements is called
finite otherwise, the set is called infinite
Equal Sets
Two sets A and B are said to be equal if they have exactly the same elements
and we write A = B. Otherwise, the sets are said to be unequal and we write
A ≠ B.
Singleton set
If a set A has only one element, we call it a singleton set. Thus,{ a } is a
singleton set.
Subsets
A set A is said to be a subset of a set B if every element of A is also an element
of B. In other words, A ⊂ B if whenever a ∈ A, then a ∈ B. It is often convenient
to use the symbol “⇒” which means implies. Using this symbol, we can write
the definition of subset as follows: A ⊂ B if a ∈ A ⇒ a ∈ B We read the above
statement as “A is a subset of B if a is an element of A implies that a is also an
element of B”. If A is not a subset of B, we write A ⊄ B.
➢ every set A is a subset of itself, i.e., A ⊂ A. Since the empty set φ has
no elements, we agree to say that φ is a subset of every set.
➢ If a set has “n” elements, then the number of subsets of the given set
is 2n and the number of proper subsets of the given subset is given by
2n-1.
Intervals as subsets of R
the set of real numbers { y : a < y < b} is called an open interval and is denoted
by (a, b). All the points between a and b belong to the open interval (a, b) but a,
b themselves do not belong to this interval. The interval which contains the end
points also is called closed interval and is denoted by [ a, b ]. Thus [ a, b ] = {x : a
≤ x ≤ b} We can also have intervals closed at one end and open at the other, i.e.,
[ a, b ) = {x : a ≤ x < b} is an open interval from a to b, including a but excluding
b. ( a, b ] = { x : a < x ≤ b } is an open interval from a to b including b but excluding
a.
Power Set
The collection of all subsets of a set A is called the power set of A. It is denoted
by P(A). In P(A), every element is a set. Thus, as in above, if A = { 1, 2 }, then P( A
) = { φ,{ 1 }, { 2 }, { 1,2 }}
Universal Set

7
while studying the system of numbers, we are interested in the set of natural
numbers and its subsets such as the set of all prime numbers, the set of all even
numbers, and so forth. This basic set is called the “Universal Set”. The universal
set is usually denoted by U, and all its subsets by the letters A, B, C, etc.
Operations on Sets
➢ Union of sets Let A and B be any two sets. The union of A and B is the set
which consists of all the elements of A and all the elements of B, the
common elements being taken only once. The symbol ‘∪’ is used to
denote the union. Symbolically, we write A ∪ B and usually read as ‘A
union B’. The union of two sets A and B is the set C which consists of all
those elements which are either in A or in B (including those which are
in both). In symbols, we write. A ∪ B = { x : x ∈A or x ∈B }
➢ Intersection of sets The intersection of sets A and B is the set of all
elements which are common to both A and B. The symbol ‘∩’ is used to
denote the intersection. The intersection of two sets A and B is the set of
all those elements which belong to both A and B. Symbolically, we write
A ∩ B = {x : x ∈ A and x ∈ B}. The intersection of two sets A and B is the
set of all those elements which belong to both A and B. Symbolically, we
write A ∩ B = {x : x ∈ A and x ∈ B
➢ Difference of sets The difference of the sets A and B in this order is the
set of elements which belong to A but not to B. Symbolically, we write A
– B and read as “ A minus B”. Using the set builder notation, we can
rewrite the definition of difference as A – B = { x : x ∈ A and x ∉ B }
Complement of a Set
Let U be the universal set and A a subset of U. Then the complement of A is
the set of all elements of U which are not the elements of A. Symbolically, we
write A′ to denote the complement of A with respect to U. Thus, A′ = {x : x ∈
U and x ∉ A }. Obviously, A′ = U – A It is clear from the definition of the
complement that for any subset of the universal set U,
we have ( A′)′ = A
Some Properties of Complement Sets
1. Complement laws: (i) A ∪ A′ = U (ii) A ∩ A′ = φ
2. De Morgan’s law: (i) (A ∪ B)´ = A′ ∩ B′ (ii) (A ∩ B)′ = A′ ∪ B′
3. Law of double complementation: (A′)′ = A 4. Laws of empty set and
universal set φ′ = U and U′ = φ.
if A and B are finite sets, then (ii) n ( A ∪ B ) = n ( A ) + n ( B ) – n ( A ∩ B )
If A, B and C are finite sets, then n ( A ∪ B ∪ C ) = n ( A ) + n ( B ) + n ( C ) – n
( A ∩ B ) – n ( B ∩ C) – n ( A ∩ C ) + n ( A ∩ B ∩ C )

MCQ’s
Q1. The set A = {14,21,28,35,….98}in set-builder form is
(a) A= {x: x=7n, n∈N and 1 ≤ n ≤ 15}
(b) A= {x: x=7n, n∈N and 2 ≤ n ≤ 14}
(c) A= {x: x=7n, n∈N and 3 ≤ n ≤ 13}
(d) A= {x: x=7n, n∈N and 4 ≤ n ≤ 1}
Q2. The set of all natural numbers x such that 4x+9< 50 in roster form is
(a) {1,2,4,6,8,10}
(b) {1,3,5,7,9}
(c) {1,2,3,4,5,6,7,8,9,10}

8
(d) None of the above
Q3. If S={x: x is a positive multiple of 3 less than 100} and P={x: x is a prime
number less than 20}. Then , n(S) + n (P) is equal to
(a) 34
(b) 31
(c) 33
(d) 41
Q4. If X={x: x=8n-7n-1, n∈N} and Y={x: x=49n-49 , n∈N }, then
(a) X⊂ 𝑌
(b) Y⊂ 𝑋
(c) X=Y
(d) X∩ 𝑌 = ∅
Q5. Which of the following sets is empty?
(a) {𝑥: 𝑥 ∈ 𝑅, −3 < 𝑥 𝑎𝑛𝑑 𝑥 < 20}
(b) {𝑥: 𝑥 ∈ 𝑅, 𝑥 2 = −8}
(c) {𝑥: 𝑥 ∈ 𝑅, 𝑥 3 = −8}
(d) Set of even prime numbers
Answer
Q1 Q2 Q3 Q4 Q5
(b) (c) (d) (a) (b)

Very short answer type Questions


Q1. Describe set A= {x:x is a prime number which is divisor of 60} in Roster
form
Q2. Let A ={x: |x|< 2, x∈ 𝑍} and B = {-1, 1} is A = B? Justify your answer
Q3. Describe set A = {3, 5, 9, 17, 33} in set-builder form
Q4. Let A ={x: 𝑥 2 +1 = 0, x∈ 𝑅 } is A = ∅ ? Justify your answer
Q5. Describe set A= {𝑥: 4𝑥 + 9 < 50, 𝑥 ∈ 𝑁} in Roster form
Q6. Find the number of proper subset of set A = {a, b, c, d}
Q7. Write power set of set A= {p, q, r}
1 2 3 4 5 6
Q8. Describe set A = {2 , 3 , 4 , 5 , 6 , 7} in set-builder form
Q9. If A = {x: x is an even natural number} and B = {x: x is prime number}
find A∩ 𝐵
Q10.If A ={x: x ∈ 𝑁, 0 < 𝑥 < 5} and B ={x: 𝑥 ∈ 𝑁, 4 < 𝑥 < 6} find A∩ 𝐵
Q11. Describe set A= {𝑥: |𝑥 − 3| < 8, 𝑥 ∈ 𝑍} in Roster form
Q12. List all proper subsets of set A = {a, b}
Q13. Let set A has 4 elements and set B has 6 elements. What can be the
minimum number of elements in AUB
Q14. If set A = {1} .How many elements P[P{P(A)}] contains ?
Q15. Write {x: x∈ 𝑅, -4< 𝑥 ≤ 6} as interval
Q16. Let A = {x: 𝑥 2 − 5𝑥 + 7 = 0, 𝑥 ∈ 𝑅} and B = ∅ .Is A = B Justify your
answer
ANSWER

9
Q1 {2. 3, 5} Q9 A ∩ 𝐵 = {2}
Q2 . A≠B Q10 A∩𝐵=∅
Q3 A = {x: x =2𝑛 + 1, 𝑛 ∈ 𝑁, 𝑛 ≤ 5} Q11 .{-4, -3, -2, -1, 0, 1, 2, 3, 4, 5, 6,
7, 8, 9, 10}
Q4 𝐴=∅ Q12 ∅, {a}, {b}
Q5 {1, 2, 3, 4, 5, 6, 7, 8, 9, 10,} Q13 4
Q6 15 Q14 16
Q7 P (A) = { ∅, {p}, {q}, {r},{p, q}, Q15 (-4, 6]
{p, r},{q, r},{p, q, r} }
𝑛
Q8 A = {x: x =𝑛+1 , 𝑛 ∈ 𝑁, 𝑛 ≤ 6} Q16 A=B

Short answer type Questions


Q1. Let A = {𝑥: 𝑥 ∈ 𝑁, 𝑥 𝑖𝑠 𝑎 𝑓𝑎𝑐𝑡𝑜𝑟 𝑜𝑓 6} and B = { 𝑥: 𝑥 ∈ 𝑁, 𝑥 𝑖𝑠 𝑎 𝑓𝑎𝑐𝑡𝑜𝑟 𝑜𝑓 8}
Find (A∩ 𝐵) and AUB
Q2. Using properties of sets, show that AU (A∩ 𝐵) =A
Q3. Show that A∩B = A∩C need not imply B = C
Q4. In a group of 950 persons, 750 can speak Hindi and 460 can speak English.
Find how many can speak Hindi only?
Q5. Let A, B and C be three sets. If 𝐴 ∈ 𝐵 𝑎𝑛𝑑 B ⊆ 𝐶 , is it true that A ⊆ 𝐶 ? If
not, give an example.
Q6. Let A = {𝑥: 𝑥 3 − 1 = 0, 𝑥 ∈ 𝑁} and B = {𝑥: 𝑥 2 − 4𝑥 + 3 = 0, 𝑥 ∈ 𝑁} . Is A⊆ 𝐵
justify your answer
ANSWER
Q1 Q4 Q5 Q6
A ∩ 𝐵 = {1, 2} , AUB = 490 NO, Let A = {4} B = A⊆ 𝐵
{1, 2, 3, 4, 6, 8} {{4},5}, C = {{4},5,6}

LONG Answer type questions


Q1. If A = {3, 4, 8}, B = {3, 6, 9}, and C = {3, 12, 14}, then verify that: AU (B∩ 𝐶) =
(AUB) ∩ (AUC)
Q2. Assume that P (A) = P (B). Prove that A = B
Q3. Two finite sets have m and n elements. The number of subsets of first set is
112 more than that of the second set .Find the values of m and n
[Ans m=7, n=4]
Q4. Let A and B be sets. If (A∩ 𝑋) = (B ∩ 𝑋) = ∅ and AUX = BUX for some set
X, Show that A = B
Q5. Let A, B, and C be the sets such that AUB = AUC and (A ∩ 𝐵) = (A ∩ 𝐶)
show that B = C

10
RELATIONS AND FUNCTIONS
ORDERED PAIR
An ordered pair consists of two objects or elements in a given fixed order.
e.g. the position of a point in a two dimensional plane in Cartesian coordinates.
Cartesian Product of Sets
For given two nonempty sets A and B, the Cartesian product 𝐴 × 𝐵 is the set of
all ordered pair from A to B.
𝐴 × 𝐵 = {(𝑥, 𝑦): 𝑥 ∈ 𝐴 𝑎𝑛𝑑 𝑦 ∈ 𝐵}
Number of elements in the Cartesian product of two sets
𝑛(𝐴 × 𝐵) = 𝑛(𝐴). 𝑛(𝐵)
RELATION
Relation from set A to set B is a subset of 𝐴 × 𝐵 by describing a relation
between first element and second element of the ordered pair in 𝐴 × 𝐵.
Denoted by 𝑅: 𝐴 → 𝐵 ⇒ If (𝑥, 𝑦) ∈ 𝑅, 𝑡ℎ𝑒𝑛
Image of 𝑥 = 𝑦 ⇒ Pre-image of 𝑦 = 𝑥
Domain
The set of all first elements of the ordered pairs in a relation R from a set A to a
set B is called the domain of the relation R.
Range
The set of all second elements of the ordered pairs in a relation R from a set A
to a set B is called the Range of the relation R.
Codomain
If there is a relation 𝑅: 𝐴 → 𝐵, the B is known as codomain of the Relation R.
FUNCTION
Function is a special type of Relation.
A relation 𝑅: 𝐴 → 𝐵 is said to be a function if every element of A has unique
image in B.
Denoted by 𝑓: 𝐴 → 𝐵 ⇒ 𝑓(𝑥) = 𝑦
Image of 𝑥 = 𝑦 ⇒ Pre-image of 𝑦 = 𝑥
If 𝑓: 𝐴 → 𝐵 , is a function, such that 𝑦 = 𝑓(𝑥)
Domain = A, Co domain = B
Range: The set of all values of f (x) taken together is called the range of
function.
Range ={𝑦 ∈ 𝑌 : 𝑦 = 𝑓(𝑥), 𝑓𝑜𝑟 𝑠𝑜𝑚𝑒 𝑥 ∈ 𝑋
Number of Relations: If n(A) = p and n(B) = q then n(A × B) = p × q,
Number of subsets of A ×B = 2pq. Therefore, total number of relations from set
A to set B = 2pq (Since every subset of A X B is a relation from A to B)
Note: Number of non-empty relations are 2pq– 1 (Subtract one empty
relation)
Example: Let n(A)=2 & n(B)=3 then total number of relations from A to
B=22X3=26=64 & Number of non-empty relations=64 – 1=63.
Note: Total number of relations from set A to A (in set A) = 22X2 = 24= 16

11
Number of functions: For n(A) = p and n(B) = q,

Total number of functions from A to B = qp.

DIFFERENCE BETWEEN RELATION AND FUNCTION


RELATION FUNCTION
𝑅: 𝐴 → 𝐵 𝑓 :𝐴 → 𝐵
No restriction between elements of A Every Elements of A has unique
&B image in B
elements of A may have many elements of A have unique image in
images in B B
Some elements of A may have no Every element of A must has image in
image in B B
Example 1. :Let f = {(1,1), (2,3), (0, –1), (–1, –3)} be a linear function from Z into
Z. Find f(x).
Solution: Since f is a linear function, f (x) = mx + c. Also, since (1, 1), (0, – 1) ∈ R,
f (1) = m + c = 1 and f (0) = c = –1. This gives m = 2 and f(x) = 2x – 1.
Some functions and their graphs

(i) Identity function Let R be the set of real numbers.


Define the real valued function f : R → R by y = f(x) = x
for each x ∈ R. Such a function is called the identity
function.
(ii) Constant function Define the function f: R → R by y = f (x)
= c, x ∈ R where c is a constant and each x ∈ R.
(iii) Polynomial function A function f : R → R is
said to be polynomial function if for each x in R,
y = f (x) = a0 + a1 x + a2 x 2 + ...+ an x n , where n is
a non-negative integer and a0 , a1 , a2 ,...,an ∈ R.
(iv) Rational functions are functions of the type f (x)/ g( x) , where
f(x) and g(x) are polynomial functions of x defined in a domain,
where g(x) ≠ 0.
(v) The Modulus function The function f: R→R defined by f(x) = |x| for each x
∈R is called modulus function. For each non-
negative value of x, f(x) is equal to x. But for
negative values of x, the value of f(x) is the negative
of the value of x,
i.e.
x, x ≥ 0
f(x) = |x| = {
−x, x < 0
(vi) Signum function The function f:R→R defined by
1, if x > 0
f(x) = { 0, if x = 0
−1, if x < 0
(vii) Greatest integer function The
function f: R → R defined by f(x) = [x], x ∈R assumes the value of
the greatest integer, less than or equal to x.

MCQs
12
Q1 Let 𝐴 = {𝑎, 𝑏, 𝑐} 𝑎𝑛𝑑 𝐵 = {𝑝, 𝑞, 𝑟} then which of the following is a function
from A to B?

(a) {(a, p) , (b, q) , (b, r)}


(b) {(a, p) , (b, q) , (c, r)}
(c) {(a, q) , (b, q) , (b, r)}
(d) {(a, p) , (a, q) , (a, r)}

Q2 Which of the following is not a function?

(a) {(𝑥, 𝑦): 𝑥, 𝑦 ∈ 𝑹, 𝒚 = 𝑥 2 }


(b) {(𝑥, 𝑦): 𝑥, 𝑦 ∈ 𝑹, 𝒚 = 𝑥 3 }
(c) {(𝑥, 𝑦): 𝑥, 𝑦 ∈ 𝑹, 𝑦 2 = 𝑥}
(d) {(𝑥, 𝑦): 𝑥, 𝑦 ∈ 𝑹, 𝑦 3 = 𝑥}

Q3 Find the range of 𝑓(𝑥) = |𝑥 − 1| − 1 .

(a) [−1, ∞[
(b) [−1, 0]
(c) ]−∞, −1]
(d) None of these

Q4 𝑎𝑥 + 𝑎−𝑥
If 𝑓(𝑥) = , then 𝑓(𝑥 + 𝑦) 𝑓(𝑥 − 𝑦) = ⋯
2

1
(a) {𝑓(2𝑥) + 𝑓(2𝑦)}
2
1
(b) {𝑓(2𝑥) − 𝑓(2𝑦)}
2
1
(c) {𝑓(2𝑥) + 𝑓(2𝑦)}
4
1
(d) {𝑓(2𝑥) − 𝑓(2𝑦)}
4

Q5 If 𝑥 ≠ 1 and 𝑓(𝑥) = 𝑥
𝑥 + 1
is a real function, then find 𝑓{𝑓(2)} .
− 1

(a) 1
(b) 2
(c) 3
(d) 4

13
Q6 1
Let 𝑓(𝑥) = 𝑥 and 𝑔(𝑥) = 𝑥 and ℎ(𝑥) = 𝑓(𝑥) . 𝑔(𝑥) . Then domain of ℎ(𝑥)

is:

(a) 𝑥∈𝑹
(b) 𝑥, 𝑦 ∈ 𝑸
(c) 𝑥 ∈𝑹−𝑸
(d) 𝑥 ∈ 𝑹 − {𝟎}

Q7 Find the value of 𝑥 𝑎𝑛𝑑 𝑦 , if (2 𝑥 + 3 𝑦, −11) = (6, 𝑥 − 2 𝑦) .


(a) 𝑥 = 3 𝑎𝑛𝑑 𝑦 = −4
(b) 𝑥 = −3 𝑎𝑛𝑑 𝑦 = −4
(c) 𝑥 = −3 𝑎𝑛𝑑 𝑦 = 4
(d) 𝑥 = 3 𝑎𝑛𝑑 𝑦 = 4
Q8 Let 𝐴 = {1, 2, 3, … , 20} and 𝑅 be a relation on set A defined as 𝑅=
{(𝑎, 𝑏): 2 𝑎 + 𝑏 = 20 𝑎𝑛𝑑 𝑎 𝑖𝑠 𝑝𝑒𝑟𝑓𝑒𝑐𝑡 𝑠𝑞𝑢𝑎𝑟𝑒} . Then the range of 𝑅 is:
(a) {1, 4, 9}
(b) {2, 12, 18}
(c) {2, 14, 20}
(d) {10, 14, 16}
Q9 Let 𝑅 be a relation on set of integers Z defined as 𝑅 = {(𝑎, 𝑏): 𝑎2 + 𝑏 2 = 25,
𝑎, 𝑏 ∈ 𝑍} . Then the range of 𝑅 is:
(a) {±2, ±3, ±5}
(b) {±4, ±9, ±16}
(c) {0, 3, 4, 16}
(d) {0, ±3, ±4, ±5}
Q1 Q2 Q3 Q4 Q5
(b) (c) (a) (a) (b)
Q6 Q7 Q8 Q9
(d) (c) (b) (d)

VERY SHORT ANSWER TYPE QUESTIONS


Q1 If A = {x∈ 𝑁: 𝑥 ≤ 3 } B = {x∈ 𝑊: 𝑥 < 2} then find AXB.

Q2 Let A = {2, 3} B= {4,5} then find number of subsets of AXB .


Q3 Let A = {1,2,3,4,6} and Let R = {(a, b) : a, b ∈ 𝐴 𝑎𝑛𝑑 𝑎 𝑑𝑖𝑣𝑖𝑑𝑒𝑠 𝑏} then write
range of R .

14
Q4 Let R be the relation on a finite set having n elements, then find number of
relations on A .
Q5 |𝑥|
If f(x) = 𝑥 and then find the value of |𝑓(8) − 𝑓(−8)| .
Q6 1
f(x) = 𝑥 2 then find f(x) – f(x+1)
Q7 A function f defined by f(x) = 2x-3 then find f(-2)
Q8 𝑓
If f(x) =𝑥 2 g(x) = 3x+2 be two real functions then 𝑔 (𝑥) is equal to
Q9 Find the domain of the function f(x) = √𝑥 − 1 .
Q10 Find the domain of the function f(x) = 𝑥 2 + 2 .
Q11 If U is a universal set and A, B are its subsets. Where U= {1, 2, 3, 4, 5}.
A = {1,3,5}, B = {x : x is a prime number}, find A' × B'
Q12 Given A = {a, b, c}, B = {2, 3}. Find the number of relations from A to B.

Q13 If (x + 1, y – 2) = (3, 1), find the values of x and y.


Q14 1
If 𝑓(𝑥) = 𝑥 3 − 𝑥 3 then find the value of 𝑓(𝑥) + 𝑓 (𝑥)
1

Q15 Let A and B be two sets such that 𝑛(𝐵) = 𝑝, 𝑛(𝐴) = 𝑞 ,then how many number
of functions 𝑓: 𝐴 → 𝐵 are possible?
Answers
Q1 Q2 Q3 Q4 Q5
{(1,0), (1,1), (2,0),(2,1), (3,0), 16 2𝑛
2
2
{1,2,3,4,6}
(3,1)}
Q6 Q7 Q8 Q9 Q10
2𝑥 + 1 -7 𝑥 2
R
2 2
[1,∞)
𝑥 (𝑥 + 1) 3𝑥 + 2
Q11 Q12 Q13 Q14 Q15
{(2, 1) ,( 4, 1) , (2, 4) , (4, 4)} 6
2 = 64 𝑥 = 2, 𝑦 = 3 0 𝑝𝑞
Long answer type questions
Q1. Find the domain for which the functions f (x) = 2x2 – 1 and g (x) = 1 – 3x
are equal
Solution: For f (x) = g (x) ⇒ 2x 2 – 1 = 1 – 3x ⇒ 2x 2 + 3x – 2 = 0
⇒ 2x 2 + 4x – x – 2 = 0 ⇒ 2x (x + 2) – 1 (x + 2) = 0 ⇒ (2x – 1) (x + 2) = 0
Thus domain for which the function f (x) = g (x) is {1/2 ,– 2} .
𝑥
Q2. Find the domain of each of the function. 𝑓(𝑥) = 𝑥 2+5𝑥+2
Solution: f is a rational function of the form g (x)/ h( )x ,
where g (x) = x and h (x) = x 2 + 3x + 2. Now h (x) ≠ 0
⇒ x 2 + 3x + 2 ≠ 0 ⇒ (x + 1) (x + 2) ≠ 0 and
hence domain of the given function is R – {– 1, – 2}.

15
TRIGONOMETRIC FUNCTIONS

MEASUREMENT OF AN ANGLE:

DEGREE MEASURE: If the rotation of a ray from the initial side to terminal side is( )

th of a revolution, the angle is said to have a measure of one degree, written as .

= 60’ (sixty minutes) and 1’ = 60”(sixty seconds).

RADIAN MEASURE: Angle subtended at the centre by an arc of length 1 unit in a unit
𝑙
circle (circle of radius 1 unit) is said to have a measure of 1 radian. θ=𝑟 where l is arc and

r is radius of circle.

RELATION BETWEEN DEGREE AND RADIAN:

2π radian= or π radian=

1 radian = = 16’ approximately.

Also radian = 0.01746 radian approximately.

SIGN OF TRIGONOMETRIC FUNCTIONS

I QUADRANT II QUADRANT III QUADRANT IV QUADRANT


(3π/2<x<2π)
(0<x<π/2) (π/2<x<π) (π<x<3π/2)
sin x Positive Positive Negative Negative

cos x Positive Negative Negative Positive

tan x Positive Negative Positive Negative

cosec x Positive Positive Negative Negative

sec x Positive Negative Negative Positive

cot x Positive Negative Positive Negative

16
VALUES OF TRIGONOMETRIC RATIOS:
Rad.→ 0 π/6 π/4 π/3 π/2 Π 3π/2 2π
Sin 0 ½ 1/√2 √3/2 1 0 -1 0
Cos 1 √3/2 1/√2 1/2 0 -1 0 1
Tan 0 1/√3 1 √3 Not defined 0 Not defined 0
The values of cosec x, sec x and cot x are the reciprocal of the values sin x, cos x and tan
x respectively.
DOMAIN AND RANGE OF TRIGONOMETRIC FUNCTIONS
Functions Domain Range
sine R [ -1 , 1 ]
Cosine R [ -1 , 1 ]
Tangent R – {( 2n + 1 )π/ 2 : n ϵ Z} R
co tangent R – {nπ : n ϵ Z} R
Secant R – {( 2n + 1) π / 2 : n ϵ Z } R – ( -1,1)
co secant R – { nπ : n ϵ Z } R – ( -1,1)
ALLIED OR RELATED ANGLES The angles nπ / 2 ± θ are called allied or related angles
and θ ± n x 360o are called co-terminal angles. For general reduction, we have the
following rules. The value of any trigonometric function for ( nπ /2 ± θ ) is numerically
equal to
(a) the value of the same function if n is an even integer with algebraic sign of the
function as per the quadrant in which angle lies.
(b) corresponding co-function of θ if n is an odd integer with algebraic sign of the
function for the quadrant in which it lies. Here sine and cosine ; tan and cot ; sec
and cosec are co-functions of each other.
Examples: sin(π/2 –x) = cos x, cos(π/2 –x)= sin x , sin(π-x)= sin x ,sin(π+x)= -sin
x ,cos(π- x)= - cos x ,tan(π+x)= tan x, .cos(π/2 +x)= -sin x , cos(2π-x)= cos x etc.
FUNCTIONS OF NEGATIVE ANGLES Let θ be any angle. Then
sin (-θ ) = - sin θ, cos (-θ ) = cos θ
tan (-θ ) = - tan θ, cot (-θ ) = -cot θ
sec (-θ ) = sec θ, cosec (-θ ) = -cosec θ
SOME FORMULAE REGARDING COMPOUND ANGLES
(i) sin ( A + B ) = sin A cos B + cos A sinB
(ii) sin ( A - B ) = sin A cos B – cos A sin B
(iii) cos(A+B) = cos A cos B – sin A Sin B
(iv) cos(A - B) = cos A cos B +sin A Sin B

17
(v) tan(A+B)=

(vi) tan(A-B)=

(vii) cot(A+B)=

(viii) cot(A-B)=
(ix) sin2A= 2sinAcosA = 2tanA /(1+tan2A) ,A≠ nπ+π/2 where n is an integer.
(x) cos 2A= cos2A – sin2A = 2cos2A – 1= 1- 2sin2A = (1- tan2A)/(1 +tan2A)
(xi) tan2A= 2tanA /(1-tan2A)
(xii) sin3A= 3sinA – 4sin3A
(xiii) cos3A= 4cos3A – 3cosA
(xiv) tan 3A= (3tanA – tan3A) / (1- 3tan2A)
(xv) cos x + cos y = 2cos cos
(xvi) cos x – cos y = - 2sin sin
(xvii) sin x +sin y = 2sin cos
(xviii) sin x -sin y = 2sin cos
(xix) 2cos x cos y= cos(x +y) +cos(x-y)
(xx) -2sin x sin y= cos(x +y) -cos(x-y)
(xxi) 2sin x cos y = sin(x + y) +sin(x - y)
(xxii) 2cos x sin y = sin(x + y) -sin(x-y)
(xxiii) sin2x - sin2y = sin(x+y).sin(x-y)
Some solved questions:-
Q.1Convert 260 40’ into radian measure
800 𝜋 4𝜋
26040’ = = × 1800 = 27 rad
3

Q2.If tanα= ½ and tanβ= 1/3 , find the value of α+β


𝑡𝑎𝑛α+tanβ
Sol. tan(α + β) = 1−𝑡𝑎𝑛αtanβ put the values

1/2+1/3 𝜋
tan(α + β) = 1 1 =1 α+β= 4
1− ∗
2 3

Q3.If sinα + cosα = 1 , find the value of sin2α.


Sol. (sinα + cosα)2=1 ⇒ 1+2 sinα cosα=1 ⇒ 1+ sin2α =1 ⇒ sin2α =0 ⇒ α =0
Q4.Fine the radian measure corresponding to 5037’30’’.
𝜋 𝜋
Sol. 50+370/60+300/60.60 ⇒ 675/120*180=32

Q5. Find the value of tan150


𝑡𝑎𝑛α−tanβ 𝑡𝑎𝑛45−tan30
Sol. tan(α − β) = 1+𝑡𝑎𝑛αtanβ ⇒ tan(45 − 30) = 1+𝑡𝑎𝑛45tan30

1−1/√3 √3−1
tan(15) = 1+1.1/√3=1+√3 by rationalising ⇒ 2-√3

18
Q6. If cos x = - 5/13, x lies in second quadrant, find the values of other five trigonometric
functions.
Sol. Cos x = - 5/13=B/H (x lies in 2nd quadrant) ⇒ P=4
sin x=P/H=4/13 , tan x=P/B=-4/5, cot x=1/tan x=-5/4, sec x=1/cos x=-13/5
cosec x=1/sin x=13/4
sin 5x−2 sin 3x +sin x
Q7. Prove that = tan x .
cos 5x−cos x
(sin 5x +sin x)−2 sin 3x 2 sin 3x .cos 2x −2 sin3x
SoL =
cos 5x−cos x −2sin3x .sin2x

2 sin 3x .(cos 2x −1) −2sin2 x sin x


= = =cos x = tan x
−2sin3x .sin2x 2 sin x cos x

π π 3
Q8. Prove that cos2 x + cos 2 (x + 3 ) + cos 2 ( x − 3 ) = 2
2π 2π
1+cos2x 1+cos(2x+ ) 1+cos(2x− ) 1 2π
SOL.LHS= + 3
+ 3
= (3 + cos 2x + 2 cos2x. cos )
2 2 2 2 3
1
= 2 (3 + cos 2x − cos2x)=3/2
x 9x 5x
Q9.Prove that : 𝑐os2x cos − cos3x cos = sin5x sin
2 2 2

1 x 9x 1 5x 3x 15x 3x
LHS= 2 (2cos2x cos − 2cos3x cos ) = 2 (cos + cos − cos − cos )
2 2 2 2 2 2

1 5x 15x
=2 (cos − cos )
2 2
5x 15x 5x 15x
1 ( + ) ( − ) 5x
= 2 (− 2sin 2 2
. sin 2 2
) = sin5xsin
2 2 2

𝟓
Q10.If cot x = , x lies in third quadrant, find the values of other five trigonometric
𝟏𝟐

Functions
5
SOL. Given cot x= 12, 1800≤ x ≤ 2700

By Pythagorean theorem H2 = B2 + P2⇒ H = √25 + 144 = 13


𝑃 12 𝐵 5 𝑃 12 𝐻 13 𝐻 13
So sin x = 𝐻 = − , cos x =𝐻=− , tan x = 𝐵 = , sec x = 𝐵 = − , cosec x = 𝑃 = −
13 13 5 5 12

MCQs
−4
Q 1 If tan 𝑥 = , then sin 𝑥 is
3
−4 4 −4 4
(a) 𝑏𝑢𝑡 𝑛𝑜𝑡 (b) 5 𝑜𝑟
5 5 5
4 −4
(c) 𝑏𝑢𝑡 𝑛𝑜𝑡 (d) None of these
5 5

19
1 1
Q 2 If tan 𝑥 = 𝑎𝑛𝑑 tan 𝑦 = 3 ,then the value of x+y is
2
 
(a) 6 (b) (c) 0 (d)4

Q 3 Which of the following is not correct?


−1 1
(a) sin 𝑥 = (b)cos 𝑥 = 1 (c) sec 𝑥 = 2 (d)tan 𝑥 = 20
5

Q 4 The value of sin 50° − sin 70° + sin 10°is


(a) 1 (b) 0 (c) ½ (d) 2
Q 5 If sin x + cos x = 1, then the value of sin2x is equal to
(a) 1 (b) 1/2 (c) 0 (d) 2
1 − 𝑡𝑎𝑛2 15°
Q 6 The value of 1 + 𝑡𝑎𝑛2 15°is

√3
(a) 1 (b)√3 (c) (d) 2
2
11
Q 7 The degree measure of 16radian is

(a) 39° 22’ 30” (b)39° 22.3’ (c) 39° 30’ 22” (d)39°
19
Q 8 The value of tan is
3
1
(a) Not defined (b)√3 (c) 1 (d)√3

Q 9 The radian measure of 5° 37’ 30” is


   
(a) 16 (b)8 (c) 32 (d)24

Q 10 The value of sin (–1125°) is


1 −1
(a) ½ (b) -1/2 (c) √2 (d) √2

Q 11 The value of 2 sin75° sin15° is


(a) ½ (b) -1/2 (c) 1 (d) -1
Q 12 If sin y + cosec y = 2, then sin2y + cosec2y is equal to -
(a) 1 (b) 4 (c) 2 (d) None of these
Q 13The value of tan1° × tan2° × tan3° ……..… tan89° is
(a) 0 (b) 1 (c) ½ (d) Not defined
Q 14The value of cos1° × cos2° × cos3° ……..… cos179° is
1
(a) √2 (b) 0 (c) 1 (d) -1
5
Q 15The point on the Unit Circle associated with the angle is
3

20
1 1 −√3 1 1 −√3 −√3 −1
(a) (2 , 2) (b)( , 2) (c) (2 , ) (d)( , )
2 2 2 2

Q 16Which of the following is not the same as tan t?


(a) - tan(-t) (b)tan(t + 2) (c) tan(t + ) (d)tan(t + / 2)
Q 17 If tan x = 5, then tan (2 x) =
(a) 10 (b) -5/12 (c) 1/10 (d) 5/12
Q 18cos x + cos( - x) =?
(a) 2 cos x (b)cos x – sin x (c) cos x + sin x (d) 0
Q 19 What is the domain of the function cot x?

(a) R (b) R – (-1, 1) (c) R – {(𝑛 ∶ 𝑛 𝜖 𝑁} (d)R – {(2𝑛 + 1) 2 : 𝑛 𝜖 𝑁}

Q 20Which of the following is not correct?


(a) sin x = - sin (-x) (b) sec (-x) = sec x
(c) sin (+x) = sin x (d) cos ( - x) = - cos x
Q 21 If tan x = 13, then cot(- x) =
(a) 13 (b) 1/13 (c) -1/13 (d) -13
Q 22 1 radian is _______________
(a) 54°48’ (b)57°16’ (c) 180° (d)17°46’
Q 23 The second hand of the watch is 2 cm long. How far the tip will move in 40 seconds?
(a) 6.28 cm (b) 12.56 cm (c) 3.14 cm (d) 1.57 cm
Q 24 If in two circles, arcs of the same length subtend angles 45° and 60° at Centre, find the
ratio of their radii.
(a) 2:3 (b) 2:5 (c) 3:4 (d) 4:3
Q 25 cos 15° =_____________
1+√3 1−√3 −1+√3 −1−√3
(a) (b) 2√2 (c) (d)
2√2 2√2 2√2
1
Q 26 If sin x = 2, then cos 2x =

1 1 √3
(a) 2 (b)√2 (c) (d) 1
2
1
Q 27 If tan x = √3, then sin 2x =

1 1 √3
(a) 2 (b)√2 (c) (d) 1
2

Q 28sin 75° – sin 15° =_________________

21
1 1 √3
(a) 2 (b)√2 (c) (d) 1
2

Q 29tan 1560°=
1 −1
(a) −√3 (b)√3 (c) √3 (d) √3

Q 30 Which of the following is not correct?


(a) cos 2𝑥 = 𝑐𝑜𝑠 2 𝑥 − 𝑠𝑖𝑛2 𝑥 (b)cos 2𝑥 = 2𝑐𝑜𝑠 2 𝑥 − 1
1+𝑡𝑎𝑛2 𝑥
(c) cos 2𝑥 = 1−𝑡𝑎𝑛2 𝑥 (d)cos 2𝑥 = 1 − 2𝑠𝑖𝑛2 𝑥

ANSWERS
Q1 b Q7 a Q13 b Q19 c Q25 a
Q2 d Q8 d Q14 b Q20 c Q26 a
Q3 c Q9 c Q15 c Q21 c Q27 c
Q4 b Q10 d Q16 d Q22 b Q28 b
Q5 c Q11 a Q17 b Q23 b Q29 a
Q6 c Q12 c Q18 d Q24 d Q30 c

SHORT ANSWER TYPE QUESTIONS (2 MARKS)


1.Find the value of (cot215o -1) /(cot215o +1).
2.Find the value of tan75o – tan30o –tan75otan30o.
3.Find the value of cos10o+cos110o+cos130o.
4..Find the value of (cot47ocot43o -1)/ (cot47o +cot43o).
5..If A+B+C= π,then what is cos(A+B) +cos C?
6..A wheel makes 36 revolutions in 6 minutes. How many radians does it turn in
1minute?
7..Convert 4 radians into degree measure?
8.If in two circles, arcs of same length subtend angles 45o and 60o at the centre, find the
ratio of their radii.
9.Prove that cos29x - cos211x = sin20x.sin2x.

10.Prove that = tan x

11.Prove that cos4x= 1-8sin2xcos2x.

12.Prove that =

13.Evaluate : sin272o - sin260o [given cos36o= (√5 +1)/4 ]


14.Find the value of tan2 (π/3) +2cos2(π/4) +5cos2(π/2)+ 3sec2(π/6) .
15.Prove that sin2x+2sin4x+sin6x = 4cos2xsin4x.

22
PART C (3 marks Questions)
1.In a circle of diameter 40 cm, the length of a chord is 20 cm. Find the length of minor arc
of the chord.
2. Prove that cot x cot2x –cot2x cot3x- cot3xcotx= 1

3..Prove that (cos x-cosy)2+ (sin x- sin y)2 = 4 sin2

4..Prove that ( + )tan2θ

5..Prove that tan4x = 4tanx(1- tan2x) / (1-6tan2x+tan4x)

6..Prove that = cot3x

PART D (5 marks Questions)

1.Find sin , cos and tan if cot x= 4/3, x lies in quadrant III.

2. Prove that sin3x+sin2x – sin x= 4sinx cos cos

3.Prove that cos6x = 32 cos6x – 48cos4x +18 cos2x -1


4.Prove that sin10osin30osin50osin70o =1/16
5. Find the value of the expression cos4(π/8) + cos4(3π/8)+ cos4(5π/8) +cos4(7π/8).
ANSWERS
(2 marks Questions)
1.√3/2 2.1 3..0 4..0 5..0
6..12π 7.229o5’29’’
8.4:3 13.(√5 -1)/8 14.8
PART C (3 marks Questions)
1.20π/3 cm.
PART D (5 marks Questions)
1. Sin(x/2)= 3/√10, cos(x/2)= -1/√10, tan(x/2)= -3
5. 3/2

23
Complex Numbers and Quadratic Equations

Imaginary Numbers
The square root of a negative real number is called an imaginary number, e.g. √-2, √-5
etc.
The quantity √-1 is an imaginary unit and it is denoted by ‘i’ called Iota.

Integral Power of IOTA (i)


i = √-1, i2 = -1, i3 = -i, i4 = 1
So, i4n+1 = i, i4n+2 = -1, i4n+3 = -i, i4n = 1
Note:
For any two real numbers a and b, the result √a × √b = √ab is true only, when atleast
one of the given numbers i.e. either zero or positive.
√-a × √-b ≠ √ab
So, i2 = √-1 × √-1 ≠ 1

• ‘i’ is neither positive, zero nor negative.


• in + in+1 + in+2 + in+3 = 0

Complex Number
A number of the form x + iy, where x and y are real numbers, is called a complex
number, x is called real part and y is called imaginary part of the complex number i.e.
Re(Z) = x and Im(Z) = y.

Purely Real and Purely Imaginary Complex Number


A complex number Z = x + iy is a purely real if its imaginary part is 0, i.e. Im(z) = 0 and
purely imaginary if its real part is 0 i.e. Re (z) = 0.

Equality of Complex Number


Two complex numbers z1 = x1 + iy1 and z2 = x2 + iy2 are equal, iff x1 = x2 and y1 = y2 i.e.
Re(z1) = Re(z2) and Im(z1) = Im(z2)
Note: Order relation “greater than’’ and “less than” are not defined for complex number.

Algebra of Complex Numbers


Addition of complex numbers
Let z1 = x1 + iy1 and z2 = x2 + iy2 be any two complex numbers, then their sum defined as
z1 + z2 = (x1 + iy1) + (x2 + iy2) = (x1 + x2) + i (y1 + y2)

Subtraction of complex numbers


Let z1 = (x1 + iy1) and z2 = (x2 + iy2) be any two complex numbers, then their difference
is defined as
z1 – z2 = (x1 + iy1) – (x2 + iy2) = (x1 – x2) + i(y1 – y2)

24
Multiplication of complex numbers
Let z1 = (x1 + iy1) and z2 = (x2 + iy2) be any two complex numbers, then their
multiplication is defined as
z1z2 = (x1 + iy1) (x2 + iy2) = (x1x2 – y1y2) + i (x1y2 + x2y1)

Division of Complex Numbers


Let z1 = x1 + iy1 and z2 = x2 + iy2 be any two complex numbers, then their division is
defined as

Conjugate of Complex Number


Let z = x + iy, if ‘i’ is replaced by (-i), then said to be conjugate of the complex number z
and it is denoted by 𝑧̅, 𝑖. 𝑒. 𝑧̅ = 𝑥 − 𝑖𝑦
Properties of Conjugate

Modulus of a Complex Number


Let z = x + iy be a complex number. Then, the positive square root of the sum of square
of real part and square of imaginary part is called modulus (absolute values) of z and it
is denoted by |z| i.e. |𝑧| = √𝑥 2 + 𝑦 2
It represents a distance of z from origin in the set of complex number c, the order
relation is not defined
i.e. z1 > z2 or z1 < z2 has no meaning but |z1| > |z2| or |z1|<|z2| has got its meaning, since
|z1| and |z2| are real numbers.

25
Properties of modulus of a Complex numbers

Example:-

Q1.Find the argument of −1 − √3 i


√3 2𝜋
Sol. – 𝜋 + tan−1 =−
1 3

1 + 3i
Q2.Find the modulus and argument:
1 − 2i
1+3i 1+2i −5+5i
Sol. × 1+2i = = -1+i (it lies in second quadrant)
1−2i 5

=𝑚𝑜𝑑𝑢𝑙𝑢𝑠 = √2 , Argument = 4
3+2isinθ
Q3.Find the real ‘θ’ such that 1−2isinθ is purely imaginary.

3+2isinθ 1+2isinθ 3+6isinθ+2isinθ−4sin2θ


Sol. 1−2isinθ x 1+2isinθ = 1+4sin2 θ

( 3−4sin2θ) 8isinθ
= + 1+4sin2θ If it is purely imaginary number than real part must be zero
1+4sin2 θ

3−4sin2 θ
∴ 1+4sin2θ = 0 ⇒ 3-4sin2 θ = 0 ⇒

√3 π
4sin2 θ = 3 ⇒ Sinθ = 2 ⇒ θ = nπ + ( −1)n 3 , n ∈ I

Q.4 Express the complex number i9 + i19 In the form a + ib.


Sol. i9 + i19 = (i2 )4i + (i2 )9I i – i = 0 =0 + 0i
Q.5 Find the multiplicative inverse of -2-3i.
𝑍 − 2+ 3𝑖
Sol. Multiplicative inverse of Z = Z-1 = ⃓𝑧⃓⃓ 2 = 13

26
𝐮 𝐯
Q6. If (x + iy)3 = u + iv, then show that + 𝐲 = 4(x2 – y2)
𝐱

Sol. (x + iy)3 = u + iv ⇒ x3 – y3i – 3xy2 + 3ix2y = u + iv

⇒x3 – 3xy2 = u, 3x2y – y3 = v

u v x3 – 3xy2 3x2 y−y3


Now x + y = + ⇒ 4x2 – 4y2 = 4(x2 – y2)
x y

MCQ,s
Q1 𝐼𝑓 ( 7 + 3𝑖)(1 + 5𝑖) = ( 𝑥 + 𝑖𝑦)𝑡ℎ𝑒𝑛 ( 𝑥, 𝑦) =.
(a) (-8,-38) (b) (-8, 38) (c) (7, -15) (d) (7, 8)
Q2 𝑧 − 𝑧̅ = 0 𝑖𝑓 𝑎𝑛𝑑 𝑜𝑛𝑙𝑦 𝑖𝑓 ∶.
(a) Re (z) = 0 (b) Im(z) =0 (c) z=0 (d) None of the above
Q3 𝐼𝑓 𝑎 > 0 , 𝑏 > 0 , 𝑡ℎ𝑒𝑛 √−𝑎 √𝑏 =.

(a) √𝑎𝑏 (b)−√𝑎𝑏 (c) √𝑎𝑏 𝑖 (d) None of the above


Q4 𝑧 = −1 + 𝑖√3 𝑎𝑛𝑑 𝑛 𝑏𝑒 𝑎𝑛 𝑖𝑛𝑡𝑒𝑔𝑟𝑎𝑙 𝑚𝑢𝑙𝑡𝑖𝑝𝑙𝑒 𝑜𝑓 3 , 𝑡ℎ𝑒𝑛 𝑧 𝑛 𝑖𝑠 𝑒𝑞𝑢𝑎𝑙 𝑡𝑜 ∶.
(a) -1 (b) 2n (c) 0 (d) 3n
Q5 10 𝑖 2 − 6𝑖 3 + 13𝑖 4 − 6𝑖 5 =.
(a) -11 (b) 0 (c) 3 (d) 23
Q6 𝐼𝑓 𝑧 =
(2+𝑖)2
, 𝑡ℎ𝑒𝑛 𝑅𝑒(𝑧) =.
2−𝑖
2 −1 −2 11
(a) (b) (c) (d)
5 3 5 5

Q7 The value of (𝑧 + 3)(𝑧̅ + 3) is equivalent to.


(a) |𝑧 + 3|2 (b) 𝑧 2 + 3 (c) |𝑧 − 3| (d) 𝑁𝑜𝑛𝑒 𝑜𝑓 𝑡ℎ𝑒 𝑎𝑏𝑜𝑣𝑒
Q8 1+𝑖
If (1−𝑖)𝑥 = 1 𝑡ℎ𝑒𝑛 .

(a) 𝑥 = 2𝑛 + 1 (b) 𝑥 = 4𝑛 (c) 𝑥 = 4𝑛 + 1 (d) 𝑥 = 2𝑛


Q9 Who denoted the square root of -1 as the imaginary part 𝑖 ?
(a) Bhaskara (b) Euler (c) Leibnitz (d) Newton
Q10 If imaginary part of the complex number is zero , what can you say about the
complex number ?
(a) It is purely real (b) It is purely imaginary
(c) Either (a) or (b) (d) None of the above
Q11 What is the modulus and argument of −1 + √3𝑖.

27
−2𝜋 2𝜋 𝜋 𝜋
(a) ( 2 , ) (b) ( 2 , ) (c) ( 4 , ) (d) ( −4 , )
3 3 6 3

Q12 𝑧1 = 1 + 2𝑖 𝑎𝑛𝑑 𝑧2 = 2 + 3𝑖 . Then what is 𝑧1 𝑧2 ?


(a) 2 + 6𝑖 (b) −4 + 0𝑖 (c) −4 + 7𝑖 (d) 8 + 7𝑖
Q13 𝑖 241 = … … ..
(a) 1 (b) -1 (c) 𝑖 (d) −𝑖
Q14 The modulus of 5 + 4𝑖 is …….
(a) 41 (b) -41 (c) √41 (d) √-41
Q15 1 + 0𝑖 is ………………………….for complex number z.
(a) Additive Inverse (b) Additive Identity Element
(c) Multiplicative Identity Element (d) Multiplicative Inverse
CASE STUDY : 1
Four friends Amar(A), Bhanu(B), Chetan(C) and
Dhanesh(D) are standing at vertices of a square if
joined. Origin is the point of intersection of
diagonals so formed. The location of Dhanesh is
(𝑧) 3 − 3𝑖 in Argand plane.
Q1. What is the location of Amar ?
Q2. The location of Bhanu(B) can be expressed as-
Q3. Conjugate of location of Chetan(C) is represented by
Q4. If Chetan changes his position by rotating anti-clockwise with 900, then what will
be his location ?
Q5. If Bhanu and Dhanesh move towards each other covering same distance, where
will they meet ?
CASE STUDY : 3
Indian mathematician Mahavira and Bhaskara stated that ‘A negative is not a square
of any number so it has no square root’ but Euler named the useless number √−1 as
𝑖. Given that 𝑥 2 + 9 = 0.
Q1 How many roots does the given equation have in the domain as Real Numbers ?
Q2 What can you say about the roots of the given equation-
Q3 -9 is the square of ………………..
Q4 Which of the following is false ?

Solutions

28
Question No. Answer ( Option)
1 (b) (-8, 38)
2 (b) Im(z) =0
3 (c) √𝑎𝑏 𝑖
4 (b) 2n
5 (c) 3
6 (a)
2
5

7 (a) |𝑧 + 3|2
8 (b) 𝑥 = 4𝑛
9 (b) Euler
10 (a) It is purely real
11 (b) ( 2 ,
2𝜋
)
3

12 (c) −4 + 7𝑖
13 (c) 𝑖
14 (c) √41
15 (c) Multiplicative Identity Element
CBQ-I 1 𝑧̅=3+3i
2 −3 + 3𝑖
3 Bhanu
4 𝑧
5 0+0𝑖
CBQ-III 1 0
2 It has two roots but both are not real
3 3i

4 Real Numbers include Complex Numbers.

Very Short & short Answer Questions


1. (1 + i)4 + (1 – i)4 = ...........
2. Find the least positive integer n such that ( )n is a positive integer .

29
3.. If a= 1+i , then find the value of a2

4. If a= cos α +i sin α, then find the value of

5..Express i-39 in the form of a + ib

6. The complex number z which satisfies the condition | | =1 lies on ………….

7.Find the real values of x and y for which the complex number -3+ix2 and x2 +y +4i are
conjugate of each other.
8. If z1 =2-i, z2 =-2+ i, find the value of Re(z1 z2)
9.For complex number Z1=-1+I , Z2=3-2i Show that

Im(Z1.Z2)=Re(Z1)Im(Z2)+Im(Z1)Re(Z2)

10.Find the non –zero integral solutions of |1-i|x =2x

11.

12.

SOLUTIONS
1. -8
2. 8
3. 2i
4. i cot α/2
5. i
6. The x axis
7. and y=±4

LINEAR INEQUATIONS

30
Inequation
A statement involving variables and the sign of inequality viz. >, <, ≥ or ≤ is
called an inequation or an inequality.

Numerical Inequalities
Inequalities which do not contain any variable is called numerical inequalities,
e.g. 3 < 7, 2 ≥ -1, etc. Literal Inequalities which contains variables are called
literal inequalities e.g. x – y > 0, x > 5, etc.

Linear Inequation of One Variable


Let a be non-zero real number and x be a variable. Then, inequalities of the
form ax + b > 0, ax + b < 0, ax + b ≥ 0 and ax + b ≤ 0 are known as linear
inequalities in one variable.

Linear Inequation of Two Variables


Let a, b be non-zero real numbers and x, y be variables. Then, inequation of the
form ax + by < c, ax + by > c, ax + by ≤ c and ax + by ≥ c are known as linear
inequalities in two variables x and y.

Solution of an Inequality
The value(s) of the variable(s) which makes the inequality a true statement is
called its solutions. The set of all solutions of an inequality is called the solution
set of the inequality.

Solving Linear Inequations in One Variable


Same number may be added (or subtracted) to both sides of an inequation
without changing the sign of inequality.

MCQs
Q1. Solve 5x < 24 when x  N
(a) {1,2,3,4}
(b) {0,2,3,4}
(c) {1,2,3,5}
(d) {0,2,3,4}
Q2. Solve 3 – 2x < 9 when x  R. Express the solution in the form of interval.
(a) [-3, )
(b) (-3, )
(c) [-3, ]
(d) [-2, )
Q3. If (x-3)/(x-2)> 0 then x belongs to
(a) (–, 2)  (3, )

31
(b) (–, –3)  (–5, )
(c) (–, 2]  [5, )
(d) (2, 3)
Q4. Solution set for inequality |x – 1|  5 is
(a) [–6, 4]
(b) [–4, 0]
(c) [–4, 6]
(d) [0, 6].
Q5. If x > 0 and y < 0 then (x, y) lies in
(a) I quadrant
(b) II quadrant
(c) III quadrant
(d) IV quadrant.
Q6. Solution set for inequality –8x-1  5x – 3 < 7 is
(a) (–1, 2)
(b) (2, 3)
(c) [–1, 2)
(d) [2, 3]
Q7. The solution of |2/(x – 4)| > 1 where x ≠ 4 is
(a) (2, 6)
(b) (2, 4) ∪ (4, 6)
(c) (2, 4) ∪ (4, ∞)
(d) (-∞, 4) ∪ (4, 6)
Q8. If x² = -4 then the value of x is
(a) (-2, 2)
(b) (-2, ∞)
(c) (2, ∞)
(d) No solution
Q9. If (x + 3)/(x – 2) > 1/2 then x lies in the interval
(a) (-8, ∞)
(b) (8, ∞)
(c) (∞, -8)
(d) (∞, 8)
Q10. Find the solution for the pair of solution x > 1 and x > -1

(a) No solution

(b) -1 < x < 1

32
(c) x < -1

(d) x > 1

ANSWERS
Q1 Q2 Q3 Q4 Q5 Q6
(a) (b) (a) (c) (d) (a)
Q7. (b) (2, 4) ∪ (4, 6)
|2/(x – 4)| > 1 ⇒ 2/|x – 4| > 1 ⇒ 2 > |x – 4|
⇒ |x – 4| < 2 ⇒ -2 < x – 4 < 2
⇒ -2 + 4 < x < 2 + 4 ⇒ 2 < x < 6
⇒ x ∈ (2, 6) , where x ≠ 4 ⇒ x ∈ (2, 4) ∪ (4, 6)
Q8. (d) No solution
Given, x² = -4 Since LHS ≥ 0 and RHS < 0
So, No solution is possible.
Q9. (a) (-8, ∞)
Given,
(x + 3)/(x – 2) > ½ ⇒ 2(x + 3) > x – 2
⇒ 2x + 6 > x – 2 ⇒ 2x – x > -2 – 6
⇒ x > -8 ⇒ x ∈ (-8, ∞)
Q10 (d) x > 1

Long answer type questions


Q 1 How many litres of water will have to be added to 1125 litres of the 45%
solution of acid so that the resulting mixture will contain more than 25% but
less than 30% acid content?

Q 2 A company manufactures cassettes and its cost and revenue functions are
C(x) = 26000 + 30x and R(x) = 43x respectively, where x is number of cassettes
produced and sold in a week. How many cassettes must be sold per week to
realise some profit.

Q 3 While drilling a hole in the earth, it was found that the temperature (T°C) at
x km below the surface of the earth was given by T = 30 + 25(x – 3), when 3  x
 15. Between which depths will the temperature be between 200°C and 300°C?

Q 4 To receive Grade ‘A’ in a course, one must obtain an average of 90 marks or


more in five examinations (each of 100 marks). If Sunita’s marks in first four
examinations are 87, 92, 94 and 95, find minimum marks that Sunita must
obtain in fifth examination to get grade ‘A’ in the course

Answers

33
Q 1 562.5< x < 900

Q 2 Cost function: C(x) = 26000 + 3Ox


Revenue function: R(x) = 43x
for profit, R(x) > C(x) ⟹ 26000 + 30x < 43x ⟹ 43x – 30x > 26000
⟹ 13x > 26000 ⟹ x > 2000
Q 3 Given that, T=30+25(x−3),
3≤x≤15T=30+25(x-3), 3≤x≤15
According to the question,
155<T<205 155<T<205
⇒155<30+25(x−3)<205⇒155<30+25(x-3)<205
⇒155−30<25(x−3)<205−30
⇒155-30<25(x-3)<205-30
[ subtracting 30 in whole]
⇒125<25(x−3)<175
⇒125<25(x-3)<175
⇒12525<x−3<17525
⇒12525<x-3<17525 [ dividing by 25 in whole]
⇒5<x−3<7
⇒5<x-3<7
⇒5+3<x<7+3
⇒5+3<x<7+3 [adding 3 in whole]

⇒8<x<10⇒8<x<10

Hence, at the depth 8 to 10 km temperature lies between 155∘→205∘C .

Q 4 x≥82

PERMUTATION AND COMBINATION

34
Fundamental principle of counting:
1.Multiplication principle
If an operation can be performed in ‘m’ different ways, following which a second
operation can be performed in ‘n’ different ways, then the two operations in
succession can be performed in m x n ways”. This can be extended to any finite
number of operations. Here, the word ‘and’ to complete the all stages of operation
and the meaning of ‘and’ is multiplication.
2.Addition principle
If there are two jobs such that they can be performed independently in m and n
ways respectively, then either of the two jobs can be performed in (m +n ) ways.

FACTORIAL NOTATION: The continued product of first n natural number is called


factorial n .Denoted by n!
n! = 1.2.3.4…….n = n (n-1)!
0! = 1
Permutations: Each of the arrangement which can be made by taking some or all
of a number of things is called permutation.
1. nPr = number of permutation of n distinct things taken r things at a time
= n(n-1)(n-2)(n-3)(n-4)………..(n-r+1)
𝑛(𝑛−1)(𝑛−2)(𝑛−3)……….(𝑛−𝑟+1)(𝑛−𝑟)……..3.2.1
= 1.2.3………(𝑛−𝑟)

𝑛!
=(𝑛−𝑟)! , Where 0 ≤r ≤n

2. Number of permutation of n distinct things taken r things at a time where


repetition is allowed = nr
3. Number of permutations of n distinct things taken all at a time = n!
4. Number of permutation of n things taken all at a time, of which p are alike of
𝑛!
one kind , q are alike of second kind = 𝑝!𝑞! where p+q =n
Combinations: Each of the different selection made by taking some or all of a
number of things is called combination.
1. . nCr = number of combination of n distinct things taken r things at a time
𝒏!
= 𝒓!(𝒏−𝒓)! , Where 0 ≤r ≤n

2. nPr = r! nCr
3 nC0 = nCn =1
4 nCr = nCn-r
5. nCx = nCy then either x=y or x+y=n
6. nCr + nCr-1 = n+1Cr
7.Number of selections of r things out of n things in which p things always
included are n-pCr-p
8. Number of selections of r things out of n things in which p things never
included are n-pCr

35
9.Number of diagonals of a polygon having n side = nC2 -n
10.Number of triangles formed by joining n non collinear points = nC3
11. Number of line segments joining n non – colinear points is 𝑛𝐶2
12. Number of line segments joining n points in which p points are collinear are
𝑛𝐶2 − 𝑝𝐶2 + 1.

Example:
1. Is 3!+4!=7! ?
Sol. 3!+4!=1.2.3+1.2.3.4=6+24=30 , 7!=1.2.3.4.5.6.7=5040
Hence 3!+4!≠7!
1 1 𝑥
2. 𝐼𝑓 + = Find x
6! 7! 8!
1 1 𝑥 1 1 𝑥
Sol. + 7(6)! = 8.7(6)! ⟹ + 7 = 8.7 ⟹ 𝑥 = 64
6! 1

3.Find r if 5Pr=2 6Pr-1

5! 6!
Sol. 5Pr=2 6Pr-1 ⟹ (5−𝑟)! = 2 (6−𝑟+1)!

5! 6(5)!
⟹ (5−𝑟)! = 2 (7−𝑟)(6−𝑟)(5−𝑟)!

⟹ (7 − 𝑟)(6 − 𝑟) = 12 ⟹ 𝑟 = 3
4.Determine n if 2nC2 : nC2=12:1
2𝑛! 𝑛!
Sol. 2nC3 : nC3=11:1 ⟹ (2𝑛−3)!3! : (𝑛−3)!3! = 11: 1

2𝑛(2𝑛−1)(2𝑛−2)(2𝑛−3)! 𝑛(𝑛−1)(𝑛−2)(𝑛−3)!
⟹ (2𝑛−3)!3!
: (𝑛−3)!3!
= 11: 1

2𝑛(2𝑛−1)(2𝑛−2)
⟹ = 11 ⟹ 𝑛 = 6
𝑛(𝑛−1)(𝑛−2)

5.How many 3-digit even number can be formed from the digit 1,2,3,4,5,6 if the
digit can be repeated.
Sol. Given digits 1,2,3,4,5,6
An even number has even digit at unit place
3-digit number has 3 place, these places can be filling as ( digits are repeated)
6 6 3
Total no of 3-digit even number =6.6.3=108
6.Find the number of permutation of the letters of the word ALLAHABAD. In how
many of these permutation
(i) All the vowels always occur together
(ii) The vowels never occur together
Sol. Here , 9 letters of which there are 4A’s,2L’s and rest are different

36
9! 9.8.7.6.5.4!
Required number of permutations =4!2! = = 7560
4!2!

(i)There are 4 vowels which are 4A’s . Consider 4A’s as one letter. Now total
letters in account are 9-4+1=6
6! 6.5.4.3.2
Total number of permutations in which all vowels occur together =2! = =
2
360
(ii) Total number of permutations in which all vowels never occur together
= Total number of permutation - The number of permutations in which all
vowels are together
=7560-360 =7200
7.A group consists of 4 girls and 7 boys. In how many ways can a team of 5
members be selected if the team has (i) no girl ? (ii) at least one boy and one girl
? (ii) at least 3 girls ?
Sol.(i) Since team has no girl. Therefore only 5 boys selected out of 7.
Required number of ways = 7C5 = 21
(ii) Since at least one boy and one girl are in every team. Therefore the team can
consist of (a)1 boy 4 girl (b) 2boys 3 girls (c) 3 boys 2 girls (d) 4 boys 1 girl
Required number of ways=7 C 1 × 4C4 +7 C 2 × 4C3+7 C 3 × 4C2+7 C 4 × 4C1
=7+84+210+140=441
(iii) Since at least 3 girls are in every team. Therefore the team can consist of
(a) 2boys 3 girls (b) 1 boys 3 girls
Required number of ways=7 C 2 × 4C3 +7 C 1 × 4C4 =84+7=91
8.How many 6-digit number can be formed from the digits 0,1,3,5,7,9 which are
divisible by 10 and no digit is repeated ?
Sol. Given digits 0,1,3,5,7,9
A number is divisible by 10 if its unit place digit is 0
6-digit number has 6 place, these places can be filling as (no digit repeated)
1 2 3 4 5 1
Total no of 6-digit number which are divisible by 10 =1.2.3.4.5.1=120
MCQs
1. The L.C.M. of 6!, 8!, 9!, 11!. Is
(a) 6! (b) 8! (c) 9! (d) 11!
2. How many three digit numbers are there with all distinct digits ?
(a) 458 (b) 568 (c) 648 (d) 748

37
3. A man has seven friends. In how many ways can he invite one or more of
them to a party ?
(a) 125 (b) 126 (c) 127 (d) 128
4. How many committees of five persons with a chairperson can be selected
from 12 persons ?
(a) 3960 (b) 1260 (c) 4270 (d) 1280
5. A polygon has 35 diagonals. Find the number of its sides.
(a) 12 (b) 16 (c) 10 (d) 20
6. How many six digit telephone numbers be made if each number starts with
35 and no digit appears more than once ?
(a) 1280 (b) 1680 (c) 1380 (d) 1520
𝑛!
7. Evaluate n =6, r=2
𝑟!(𝑛−𝑟)!

(a) 15 (b) 16 (c) 10 (d) 18


8. If nC5 = nC7, find n.
(a) 12 (b) 15 (c) 14 (d) 18
9 A polygon has 44 diagonals, then number of its sides are:

(a) 11 (b) 7 (c) 8 (d) 10


10. The number of ways in which 5 prizes be distributed among 4 boys,
while each boy is capable of having any number of prizes is:

(a) 54 (b) 45 (c) 4! × 24 (d) none of these

11. Number of words from the letters of the words BHARAT in which B and H
will never come together is:

(a) 210 (b) 240 (c) 422 (d) 400


12.There are 10 true-false questions in a examination. Then these questions can
be answered in:
(a) 210 (b) 512 (c) 422 (d) 1024
1-d 2-c 3-c 4-a 5-c 6-b 7-a 8-a 9-a 10-b 11-b 12-d
MISCELLANEOUS EXERCISE
1 1 𝑥
1. 𝐼𝑓 + 9! = 10! Find x
8!
2. Evaluate 5P3
3. If 10Pr=5040 , find the value of r

38
4. If nC8=nC6 , find nC2
5. If 15Cr :15Cr-1=11:5
6. How many 3-digit number can be formed from the digit 1,2,3,4,5 if the digit
can be (i) repeated (ii) not repeated.
7. Eight children are to be seated on a bench, How many arrangements are
possible if the youngest child sits at the left hand end to the bench ?
8. How many words each of 3 vowels and 2 consonants can be formed from the
letters of the word “INVOLUTE” ?
9. How many words can be formed by using the letters of the word ORIENTAL,
so that the vowels always occupy the odd places ?
10. A group consists of 4 girls and 9 boys. In how many ways can a committee of
7 members be selected if the committee has (i) exactly 3 girls ? (ii) at most 3
girl ? (ii) at least 3 girls ?
11. In how many ways can the letter of the word ASSASSINATION be arranged so
that all the S’s are together ?
12. Find the number of integers greater than 7000 that can be formed with the
digits 3, 5, 7, 8 and 9 where no digit are repeated.
13. Case Study 1: A state cricket authority has to choose a team of 11 members,
to do it so the authority ask
2 coaches of a government
academy to select the team
members that have experience
as well as best performer in the
last 15 matches. They can make
up a team of 11 cricketers
amongst 15 possible
candidates in which 5 players
can bowl.
(i) In how many ways can the final eleven be selected from 15 cricket players ?
(ii) In how many ways can the final eleven be selected if exactly 4 bowlers must
be included.
(iii) In how many ways can the final eleven be selected if all bowlers must be
included.
14. Case Study 2: A committee of 5 members is to be formed out of 6 gents and 4
ladies.
In how many ways this can be done, when
i. At least two ladies are included?
ii. At most two ladies are included?
Answers
(1) 100 (2) 60 (3) 4 (4) 91 (5) 5 (6) 125,60 (7) 5040 (8) 2880 (9) 576 (10)
504,1632,588 (11) 151200 (12) 192 (13) 1365,600,210 (14) 186,186

BINOMIAL THEOREM
39
Binomial theorem for positive integral indices:
(a+b)0=1
(a+b)1=a+b
(a+b)0=a2+2ab+b2
(a+b)3=a3+3a2b+3ab2+b3
(a+b)4=a4+4a3b+6a2b2+4ab3+b4
The coefficient in these expansions as follows
Index coefficients
0 1
1 1 1
2 1 2 1
3 1 3 3 1
4 1 4 6 4 1
This array of numbers is known as Pascal’s triangle. This is use to determine coefficient
in the binomial expansions.
Binomial theorem for any positive integer n:
(a + b)n = nC0 an+ nC1 an-1b+ nC2 an-2b2+ nC3 an-3b3+…….+ nCn-1 abn-1+ nCn bn
Observation:
1 The coefficient nCr occurring in the binomial theorem are known as binomial
coefficients.
2.There are (n+1) terms in the expansions of (a+b)n i.e one more than the index
3.The sum of indices of a and b is n in every term of the expansions.
4. (a - b)n = nC0 an- nC1 an-1b+ nC2 an-2b2+-nC3 an-3b3+…….+ (-1) nCn bn
𝑛+1
, 𝑛 𝑖𝑠 𝑜𝑑𝑑
2
5.Number of terms in the expansions of (a+ b)n+(a-b)n={ 𝑛
+1 , 𝑛 𝑖𝑠 𝑒𝑣𝑒𝑛
2
𝑛+1
, 𝑛 𝑖𝑠 𝑜𝑑𝑑
6. Number of terms in the expansions of (a+ b)n-(a-b)n={ 𝑛2
, 𝑛 𝑖𝑠 𝑒𝑣𝑒𝑛
2

Example:
1.Expand (1+2x)5
Sol. By using binomial theorem

40
(1+2x)5 = 5C0 +5C1 (2x)+ 5C2(2x)2+5C3(2x)3+5C4(2x)4+5C5(2x)5
=1+5(2x)+10(4x2)+10(8x3)+5(16x4)+1(32x5)
=1+10x+40x2+80x3+80x4+32x5
2 𝑥
2.Expand (𝑥 − 2)5
2 𝑥
Sol. By using binomial theorem (𝑥 − 2)5

= 5C0 (2/x)5 - 5C1 (2/x)4(x/2)+ 5C2(2/x)3(x/2)2 -5C3(2/x)2(x/2)3 +5C4(2/x)1(x/2)4 -


5C5(x/2)5

32 40 20 5𝑥 5𝑥 3 𝑥5
=𝑥 5 − 𝑥 3 + − + − 32
𝑥 1 8

3.Using binomial theorem evaluate (101)4


Sol. By using binomial theorem
(101)4 =(1+100)4= 4C0 + 4C1(100)+ 4C2(100)2+4C3(100)3+4C4(100)4
=1+400+60000+4000000+100000000
=104060401
4.Using binomial theorem expand (a+ b)5+(a-b)5 and hence find the value of
(√2+1)5+(√2-1)5
Sol. By using binomial theorem
(a+ b)5+(a-b)5= 5C0 a5+ 5C1 a4b+ 5C2 a3b2+ 5C3 a2b3+5C4 ab4+ 5C5 b5
+5C0 a5- 5C1 a4b+ 5C2 a3b2- 5C3 a2b3+5C4 ab4- 5C5 b5
=2[5C0 a5 + 5C2 a3b2 + 5C4 ab4]
=2[a5 +10 a3b2 +5 ab4]

Putting a=√2 , b=1

(√2+1)5+(√2-1)5= 2[(√2)5 +10 (√2)3+5 (√2]=58(√2


5. Using binomial theorem prove that 6n-5n always leaves remainder 1 when divided by
25.
Sol. We have 6n-5n= (1+5)n-5n
⟹ 6n-5n =[ nC0+nC1 (5)+nC2(5)2+ nC3(5)3+ nC4(5)4+ nC5(5)5 +……………+
nCn(5)n]-5n

⟹ 6n-5n =1+5n+ nC2(5)2+ nC3(5)3+ nC4(5)4+ nC5(5)5 +……………+ nCn(5)n]-5n


⟹ 6n-5n =1+52[nC2+ nC3(5)+ nC4(5)2+ nC5(5)3 +……………+ nCn(5)n-2
⟹ 6n-5n =25 [integer]+1

41
⟹ 6n-5n always leaves remainder 1 when divided by 25
6.Find the number of term in the expansion of (1+2x+x2)27
Sol. We have (1+2x+x2)27= [(1+x)2]27=(1+x)54
number of term in the expansion of (1+2x+x2)27
= number of term in the expansion of (1+x)54
=54+1=55
7.Show that nC0 + nC1+ nC2+ nC3+……….+ nCn= 2n
Sol. We have (1+x)n= nC0 + nC1 x + nC2x2+ nC3 x3+……….+ nCn xn
Put x = 1
(1+1)n= nC0 + nC1+ nC2+ nC3+……….+ nCn
2n= nC0 + nC1+ nC2+ nC3+……….+ nCn
MCQs
1. 12th term in the expansion of (𝑥 − 1)11

(a) 1 (b) - 𝑥 (c) -1 (d)none of these

2. Number of terms in the expansion (𝑎 + 𝑏)10 −(𝑎 − 𝑏)10 are

(a) 4 (b 5 (c) 3 (d) 2

3.How many terms in the expansion of (1-2x+x2)50 ?


(a) 50 (b) 51 (c) 100 (d) 101
3
4. Number of terms in the expansion (2 − 𝑥 )99 are
(a) 99 (b) 98 (c) 100 (d) 101
5. The number 6n-5n always leaves remainder 1 when divided by
(a) 24 (b 25 (c) 23 (d) 22

6. The expansion of (1+3x+3x2+x3)n has 100 terms then value of n is


(a) 27 (b 33 (c) 23 (d) 36

7.The value of 5C0+5C1+5C2+5C3+5C4+5C5 is

(a) 16 (b 22 (c) 32 (d) 64

8. Number of terms in the expansion (𝑎 + 𝑏)10 +(𝑎 − 𝑏)10 are

(a) 6 (b 5 (c) 3 (d) 2

42
9. The value of 10C0 -10C1+10C2-10C3+10C4-5C5+…….+(-1)10Cn is

(a) 210 (b 25 (c) 64 (d) 0

10.The coefficient of x10 in the expansion of (1-x)10 is

(a) 1 (b -1 (c) 210 (d) 2

1-c 2-b 3-d 4-c 5-b 6-b 7-c 8-a 9-d 10-a

MISCELLANEOUS EXERCISE
1. Expand (2-3x)6
𝑥 1
2. Expand (3 − 𝑥)5

3. Using binomial theorem evaluate (99)5


4. Using binomial theorem evaluate (102)5
5. Using binomial theorem expand (x+ 1)6+(x-1)6 and hence find the value of
(√2+1)6+(√2-1)6
6. Using binomial theorem prove that 9n+1-8n-9 divided by 64.
7. Show that nC0 - nC1+ nC2- nC3+……….+(-1)n nCn= 0
8. Which is larger (1.01)1000000 or 10000
9. Find the number of rational term in the expansion of (√2+1)6
10. Find an approximation of (0.99)5

ANSWER
1.(64-576x+2160x2-4320x3+4860x4-2916x5+729)
2.(x5/243) +(5x3/81)+(10x/27)+(10/9x)+(5/3x3)+(1/x5)
3. 9509900499
4.11040808032
5.2(x6+15x4+15x2+1) , 198
8. (1.01)1000000
9. 4
10. 0.951

SEQUENCES AND SERIES

43
Sequence
A succession of numbers arranged in a definite order according to a given certain rule is
called sequence. A sequence is either finite or infinite depending upon the number of
terms in a sequence.

Series
If a1, a2, a3,…… an is a sequence, then the expression a1 + a2 + a3 + a4 + … + an is called
series.

Progression
A sequence whose terms follow certain patterns are more often called progression.

Finite Series: A series having finite number of terms is called finite series.
Infinite Series: A series having infinite number of terms is called infinite series.
Arithmetic Progression (AP)
A sequence in which the difference of two consecutive terms is constant, is called
Arithmetic Progression (AP).

Arithmetic Mean
(a) If a, A and b are in AP, then A= (a + b)/2 is called the 2 arithmetic mean of a and b.
(c) If a, A1 , A2 , A3 ,…,An, b are in AP, then A1, A2, A3,…, An are n arithmetic mean
between a and b, where
𝐛−𝐚
d =𝐧+𝟏
𝐧𝐚+𝐛 (𝐧−𝟏)𝐚+𝟐𝐛
A1=a+d= 𝐧+𝟏 , A2=a+2d= 𝐧+𝟏
𝐚+𝐧𝐛
An=a+nd = 𝐧+𝟏

Geometric Progression (GP)


A sequence in which the ratio of two consecutive terms is constant is called GP. The
constant ratio is called common ratio (r).
i.e., ( an+1)/an = r, ∀n≥1
Properties of Geometric Progression (GP)
(i) nth Term of a G, If a is the first term and r is the common ratio

(a) nth term of a GP from the beginning is an = arn-1


(b) nth term of a GP from the end is a’(n )= l/rn-1, l = last term

44
(c) If a is the first term and r is the common ratio of a GP, then the GP can be written as
a, ar, ar2,… , arn-1, …
(ii) If all the terms of GP be multiplied or divided by same non-zero constant, then the
resulting sequence is a GP with the same common ratio.
(iii) If a, b and c are three consecutive terms of a GP, then b2 = ac
Terms in GP
(a) Three terms of a GP can be taken as a/r, a and ar.
(b) Four terms of a GP can be taken as a/r3, a/r, ar and ar3.
(c) Five terms of a GP can be taken as a/r2, a/r,a, ar and ar2.
Sum of n Terms of a GP
Sum of n terms of a GP is given by
𝑎(1 − 𝑟 𝑛 )
, 𝑖𝑓 ⌈𝑟⌉ < 1
1−𝑟
(a) Sum of n terms of a GP is given by 𝑆𝑛 = 𝑎(𝑟 𝑛 − 1)
, 𝑖𝑓 ⌈𝑟⌉ > 1
𝑟−1
2
{𝑎 , 𝑖𝑓 ⌈𝑟⌉ = 1
𝑎
(𝑏) 𝑆∞ = , 𝑖𝑓 ⌈𝑟⌉ < 1
1−𝑟

Geometric Mean (GM)


(a) If a, G, b are in GP, then G is called the geometric mean of a and b and is given by G =
√ab
b) If a, G1, G2, G3, , Gn, b are in GP, then G1, G2, G3,… , Gn, are in GM’s between a and b,
1
b
where r=(a)n+1
1 2
b b
G1=ar=a(a)n+1 , G2==a(a)n+1 ...........................................
n
b
Gn=arn==a(a)n+1

Example:-
𝟐𝐧𝟐 + 𝟓
Q1.Write the first five terms of the sequence whose nth term is an =n .
𝟒

2n2 + 5
sol. an = n( )
4
7 26 69 148 275
⇒ a1 = 4 , a 2 = , a3 = , a4 = , a5 =
4 4 4 4

Q2. For what values of x, the numbers - , x,- are in G.P.?

45
Sol. If - , x,- are in G.P then

x2= ( - )×( - )= 1 ⇒ x = ±1

Q3. Which term of the G.P. 5,10,20,40,… is 5120


Sol. an=arn-1 5120=5.2n-1 ⇒ 2n-1=5120/5=1024=29 ⇒ n-1=10 ⇒ n=11
Q4. Between 1and 31, m numbers have been inserted in such a way that the resulting
sequence is an A.P. and the ratio of the 7 th and (m − 1)th numbers is 5 ∶ 9. Find the
value of m
Sol. Total terms in the AP = (m + 2)
30
31 = 1 + (m + 2 − 1 )d ∴ d = m+1

a8 5 1+7d 5 30 1022
= ⇒ 1+(m−1)d = 9 4 = (5m − 68)d = (5m − 68) m+1 ∴ m = = 14
am 9 73

Q5. If a and b are the roots of x 2 − 3x + p = 0 and c and d roots of x 2 − 12x + q = 0,


where a, b, c, d from G.P. Prove that (q + p): (q - p), =17:15.
Sol. a + b = 3 and ab = p c + d = 12 and cd =q
a, b, c, d are in G.P. i.e. b = ar, c = ar2 , d = ar3
a + b = 3 ⇒ a(1 + r) = 3 ; c + d = 12 ⇒ ar 2 (1 + r) = 12
Find r = ± 2
q+p cd+ab r4 +1 16+1 17
= cd−ab = r4 −1 =16−1 = 15
q−p

Q6. The 5th , 8th and 11th terms of a G.P. are p,q and s ,respectively. Show that q2=ps
Sol. an=arn-1
a5=ar5-1=p...........1 , a8=ar8-1=q............2 , a11=ar11-1=s..........3
Multiply 1& 3 ps=a2r4+10= (ar7)2=q2
Q7. The third term of a G.P. is 4.Find the product of its first five terms
Sol. an=arn-1 a3=ar2=4
let five terms of a GP are be a, ar ,ar2,ar3 and ar4
product of these numbers is a5r1+2+3+4=(ar2)5=45

MCQ,s
Q1 Which statement is correct:
(a) The sum or difference of consecutive terms of two G.P.s, is again a G.P.

46
(b) The sum or difference of consecutive terms of two A.P.s, is again a A.P. .
(c) The product of consecutive terms of two G.P.s, is again a G.P.
(d) The division of consecutive terms of two G.P.s, is again a G.P.

Q2 The lengths of three unequal edges of a rectangular solid block are in G.P. If the
volume of the block is 216 cm3 and the total surface area is 252 cm2, then the
length of the longest edge is
(a) 12 cm (b) 6 cm (c) 18 cm (d) 3 cm

Q3 No term of geometric sequence can be.

(a) 0 (b) 2 (c) 1 (d) 3

Q4 The arithmetic mean between 2+√(2) and 2-√(2) is

(a) 0 (b) √(2) (c) 2 (d) 4

Q5 For a G.P. the ratio of the 7th and the third terms is 16.. What is the common ratio?

a) 2 (b) ±2 (c) 4 (d) ±4


Q6 How many terms of G.P. 3, 32 , 33 , .......... are needed to give the sum 120?
(a) 3 (b) 4 (c) 5 (d) 6
Q7 How many terms of the geometric series 1 + 4 + 16 + 64 + ... will make the sum
5461?
(a) 3 (b) 4 (c) 5 (d) 7
Q8 Next term of the sequence 0.02, 0.006, 0.0018,… is
(a) 0.000054 (b) 0.0054
(c) 0.00054 (d) 0.00036
Q9 ASSERTION-REASON BASED QUESTIONS
In the following questions, a statement of assertion (A) is followed by a statement
of Reason(R).Choose the correct answer out of the following choices.
(a) Both A and R are true and R is the correct explanation of A.
(b) Both A and R are true but R is not the correct explanation of A.
(c) A is true but R is false.
(d) A is false but R is true.
Assertion (A) :Three numbers a, b, c are in A.P., then b is called the AM of a and c.

Reason(R): Three numbers a, b, c are in A.P. iff 2b = a + c.

Q 10 The fourth, seventh and tenth terms of a G.P. are p, q, r respectively, then :

47
(a) p 2 = q 2 + r 2 (b) q 2 = pr (c) p 2 = qr (d) pqr + pq + 1 = 0
Q 11 Which of the following is the geometric mean of 3 and 12

(a) 4 (b) 6 (c) 7.5 (d) 36.

Q 12 4 th term from the end of the G.P. 3, 6, 12, 24., .........., 3072 is
(a) 348 (b) 843 (c) 438 (d) 384
Q 13 11 1
If ax = by = cz, where a, b, c are in G.P. and a,b, c, x, y, z ≠0; thenx y , z, are in:

(a) A.P. (b) G.P. (c) H.P (d) None of these


Q 14 If the arithmetic mean of two numbers be A and geometric mean be G, then the
numbers will be

(a) A ± (A2 – G2 ) (b)√(A ± √A2 − G 2

A ±√(A+G)(A−G)
(c) A ± √(A + G)(A − G) (d) 2

Answers:-1.b 2.a 3.a 4.c 5.b 6.b 7.d 8.b 9.a


10.b 11.b 12.d 13.a 14.c

CASE STUDY : 1

Growth of bacterial cultures is defined as an increase in the number of bacteria in a


population rather than in the size of individual cells.

In growth of a bacterial population each division cycle (generation), one cell gives
rise to 2 cells, then 4 cells, then 8 cells, then 16, then 32, and so forth.

Based on the above case answer these questions:

48
Q1 If there were 30 bacteria present in the culture originally, How many bacteria will
be present at the end of the 4th hour?

Q2 Ratio of no of bacteria present at the end of the 5th hour to bacteria present at the
end of the 7th hour is:

Q3 If there were 30 bacteria present in the culture originally, How many bacteria
increase in first 3hrs?

Q4 How many hours is taken to convert 8 time of original no of bacteria?


Answer:-1.480 2. 1:4 3.550
4.3hrs

CASE STUDY :-2


Q17. Rohan and Sohan are playing cards.These cards are numbered from 1 to 100.

i ) How many numbers between 1 and 100 are divisible by 2?


ii) How many numbers between 1 and 100 are divisible by 5?
iii) How many numbers between 1 and 100 are divisible by 2 or 5?
Iv) Find the sum of all the numbers between 1 and 100, divisible by 2 and 5 both.
v)Find the sum of all the numbers between 1 and 100, divisible by 2 or 5.
ANSWERS
i) 49 II) 19 iii) 59 iv) 450 v) 2950

VERY SHORT ANSWER TYPE QUESTIONS


Q1. Find the 12th term of a G.P. whose 8th term is 192 and the common ratio is 2.
Q2. Find the sum of all integers between 84 and 719, which are multiples of 5.
Q3. In a G.P. the 3rd term is 24 and the 6th term is 192.Find the 10th term.
Q4. If a,b,c are in A.P. and a,b,d are in G.P.,then prove that a,a-b,d-c are in G.P.

49
Q5 Find the two numbers whose A.M.is 25 and G.M. is 20
Q 6. if the sum of n terms of an A.P. is pn+qn2, where p and q are constants, find the
common difference
Q7. The sum of first three terms of G.P. is 13/12 and their product is -1. Find the G.P.
Q 8.How many terms of the series 2+6+18 …must be taken to make the sum equal to
728 ?
Q 9. Insert three arithmetic means between 3 and 19

ANSWERS
Q1. 3072 Q2. 50800 Q3. 3072 Q5. . 40,10 Q6. 2q Q7. 4/3,
-1,3/4 Q8. 6 Q9.7,11,15

LONG ANSWER TYPE QUESTIONS


Q1. The sum of two numbers is 6 times their geometric mean, show that the numbers
are in the ratio (3+2√2):(3-2√2).
Q2. Let S be the sum ,P the product and R the sum of reciprocals of n terms in a G.P.
Prove that P2Rn=Sn.
Q3Find the sum of the series 5+55+555… to n terms

Q4. If a, b, c are in A.P. b, c, d are in G.P. and are in AP, prove that a, c, e are
in G.P
Q 5.If a, b, c are in A.P. and x, y, z are G.P. , then show that xb-c.yc-a.za-b =1

Q 6. Find the value of n so th may be geometric mean between a


and b.
ANSWERS
Q3. 7/81{9n-1+1/10n } Q6. n= -1/2

50
Straight lines

Distance Formula
The distance between two points A (x1, y1) and B (x2, y2) is given by
𝐴𝐵 = √(𝑥2 − 𝑥1 )2 + (𝑦2 − 𝑦1 )2

The distance of a point A(x, y) from the origin 0 (0, 0) is given by OA = √𝑥 2 + 𝑦 2


Section Formula
The coordinates of the point which divides the joint of (x1, y1) and (x2, y2) in the ratio m :
n internally, is
𝑚𝑥2 +𝑛𝑥1 𝑚𝑦2 +𝑛𝑦1 𝑚𝑥2 −𝑛𝑥1 𝑚𝑦2 −𝑛𝑦1
( 𝑚+𝑛
,
𝑚+𝑛
) and externally is ( 𝑚−𝑛
,
𝑚−𝑛
)

𝑥1 +𝑥2 𝑦1 +𝑦2
Mid-point of the joint of (x1, y1) and (x2, y2) is ( , )
2 2

X-axis divides the line segment joining (x1, y1) and (x2, y2) in the ratio -y1 : y2.
Y-axis divides the line segment joining (x1, y1) and (x2, y2) in the ratio -x1 : x2.
The coordinates of the centroid of the triangle whose vertices are (x1, y1), (x2, y2) and
𝑥1 +𝑥2 +𝑥3 𝑦1 +𝑦2 +𝑦3
(x3, y3) is ( , )
3 3

Area of Triangle
The area of the triangle, the coordinates of whose vertices are (x1, y1), (x2, y2) and (x3,
y3) is the absolute value of

If the points (x1, y1), (x2, y2) and (x3, y3) are collinear, then x1 (y2 – y3) + x2 (y3 – y1) + x3
(y1 – y2) = 0.

Straight Line
A curve is said to be a straight line if two points are taken on the curve such that every
point on the line segment joining any two points on it lies on the curve. General
equation of a line is ax + by + c = 0.

51
Slope or Gradient of Line
The inclination of angle θ to a line with a positive direction of X-axis in the anti-
clockwise direction, the tangent of angle θ is said to be slope or gradient of the line and
is denoted by m. i.e. m = tan θ
The slope of a line passing through points P(x1, y1) and Q(x2, y2) is given by 𝑚 =
𝑦 𝑦
tan 𝜃 = 𝑥2−𝑥1
2− 1

Note: Slope of a line parallel to X-axis is zero and slope of a line parallel to Y-axis is not
defined.

Angle between Two Lines

𝑚 𝑚
The angle θ between two lines having slope m1 and m2 is tan 𝜃 = |1+𝑚
2− 1
|
𝑚 1 2

• If two lines are parallel, their slopes are equal i.e. m1 = m2.
• If two lines are perpendicular to each other, then their product of slopes is -1 i.e.
m1m2 = -1.
• Three points are collinear if slopes of line segments joining them are equal.

Various Forms of the Equation of a Line


If a line is at a distance k and parallel to X-axis, then the equation of the line is y = ± k.

If a line is parallel to Y-axis at a distance c from Y-axis, then its equation is x = ± c.

One point-slope form: The equation of a line which passes through the point (x1, y1)
and has the slope of m is given by y – y1 = m (x – x1).

Two points form: The equation of a line passing through the points (x1, y1) and (x2, y2)
is given by
𝑦2 −𝑦1
𝑦 − 𝑦1 = ( ) (𝑥 − 𝑥1 )
𝑥2 −𝑥1

Slope-intercept form: The equation of line with slope m and making an intercept ‘c’ on
the y-axis, is y = mx + c.

52
The Intercept form: The equation of a line which cuts off intercepts a and b
𝑥 𝑦
respectively on the x and y-axes is given by + 𝑏 =1
𝑎

Position of Points is Relative to a Given Line


Let the equation of the given line be ax + by + c = 0 and let the coordinates of the two
given points be P(x1, y1) and Q(x2, y2).

The two points are on the same side of the straight-line ax + by + c = 0, If ax1 + by1 + c
and ax2 + by2 + c have the same sign.

The two points are on the opposite sides of the straight-line ax + by + c = 0, If ax1 + by1 +
c and ax2 + by2 + c have opposite sign.

Distance of a Point from a Line


The perpendicular distanced of a point P(x1, y1)from the line Ax + By + C = 0 is given by
𝐴𝑥1 +𝐵𝑦1 +𝐶
𝑑=| |
√𝐴2 +𝐵 2

Distance between Two Parallel Lines


The distance d between two parallel lines y = mx + c1 and y = mx + c2 is given by

𝑑 = | 𝑐1−𝑐2 |
√1+𝑚2

Examples :

1. By using the slope method prove that the point (4,8), (5,12) and (9,28) are
collinear.

12−8 28−12
Sol: Let A (4, 8) B(5, 12) and C ( 9. 28) Slope of AB = = 4, slope of BC = =
5−4 9−5

4, as slope of AB = Slope of BC, therefore points A, B and C collinear.

2. Two lines passing through the point (2, 3) make an angle of 450. If the slope of
one of the line is 2, find the slope of other line.
𝑚 𝑚
Sol: Using tan 𝜃 = |1+𝑚
2− 1
|,
𝑚 1 2

2−𝑚1
1=| |
1+2𝑚1

Removing mode sign and solving by cross multiplication we get m = 3 and 1/3

53
3. Find the equation of line passing through (1, 2) and making an angle 300 with y
axis.
Sol: if line makes angle 300 with y axis, it means it makes an angle 600 with x axis
So slope of line = tan 60° = √3
Using slope point from equation of line is (y -2) = √3(𝑥 − 1)
4. A straight line moves so that the sum of reciprocal of its intercepts made on axes
is constant. Show that it passes through a fixed point.
Sol:
𝑥 𝑦 1 1
Consider intercept from of equation of line + 𝑏 = 1, given that 𝑎 + 𝑏 =
𝑎
1 𝑘 𝑘
(𝑠𝑜𝑚𝑒 𝑐𝑜𝑛𝑠𝑡𝑎𝑛𝑡) => 𝑎 + 𝑏 = 1, This proves that the line passes through a
𝑘

fixed point (k, k)


5. Show that the point (3, 4) and (2, -6) are situated in opposite side of the line 3x –
4y – 8 = 0.
Solution: substitute (3, 4) in 3x – 4y – 8 ⇒ 9 – 16 – 8 = - 15
Substitute (2, - 6) in 3x – 4y – 8 ⇒6 + 24 – 8 = 22
We see that sign are opposite so these number lies in opposite sign of given line.

Very short answer type questions


1. The equation of a straight line which cuts off an intercept of 5 units on negative
direction of y-axis and makes an angle of 120º with the positive direction of x-
axis is
(a) 3x + y + 5 = 0 (b) 3x + y - 5 = 0 (c) 3x - y - 5 = 0 (d) 3x - y + 5 = 0
Answer : A
2. The equation of the straight line that passes through the point (3, 4) and
perpendicular to the line 3x + 2y + 5 = 0 is
(a) 2x + 3y + 6 = 0 (b) 2x – 3y – 6 = 0
(c) 2x – 3y + 6 = 0 (d) 2x + 3y – 6 = 0 Answer : C
3. If a, b, c are in A.P., then the straight lines ax + by + c = 0 will always pass
through
(a) (1, – 2) (b) (1, 2) (c) (– 1, 2) (d) (– 1, – 2) Answe; A

54
Which one of the following is the nearest point on the line 3x– 4y = 25 from the
origin?
(a) ( –1, –7) (b) (3, –4) (c) ( –5, –8) (b) (3, 4) Answer : B
4. If the midpoint of the section of a straight line intercepted between the axes is (1,
1), then what is the equation of this line?
(a) 2x + y = 3 (b) 2x – y = 1 (c) x – y = 0 (d) x + y = 2
Answer : D
5. What is the equation of the line which passes through (4, – 5) and is parallel to
the line 3x + 4y +5 = 0?
(a) 3x – 4y – 32 = 0 (b) 3x + 4y + 8 = 0
(c) 4x – 3y – 31 = 0 (d) 3x + 4y – 8 = 0 Answer : B

Short answer type questions

1. Determine angle B in triangle ABC whose vertices are A(-2,1), B(2,3) and C(-2,-
2
4). Ans 𝑡𝑎𝑛−1 (3)

2. Which point on the x axis is equidistant from (7, 6) and (-3,4) Ans : (3,0)
3. Find the equation of straight line which passes through the point (-1, 2) and
makes equal intercept with coordinate axes. Ans: x + y = 1
4. Show that if a line ax + by + c = 0 passes through point (1, -2), then a, b, c are in
A.P.
5. If kx +2y -1 = 0 and 6x – 4y +2 = 0 are identical lines find the value of k.
Ans: k = - 3
6. Find the value of p so that the three lines 3x + y – 2 = 0, px + 2 y – 3 = 0 and 2x – y
– 3 = 0 may intersect at one point.
Ans p = 5
7. Find angles between the lines √3𝑥 + 𝑦 = 1 𝑎𝑛𝑑 𝑥 + √3𝑦 = 1 .
Ans right angle
8. The perpendicular from the origin to the line y = mx + c meets it at the point (–1,
2). Find the values of m and c.
Ans m = ½ and c = 5/2
9. In the triangle ABC with vertices A (2, 3), B (4, –1) and C (1, 2), find the equation
and length of altitude from the vertex A. Ans: x – y +1 = 0 and length = √2 unit

10. If p is the length of perpendicular from the origin to the line whose intercepts on

55
𝟏 𝟏 𝟏
the axes are a and b, then show that = 𝒂𝟐 + 𝒃𝟐
𝒑𝟐

3 Marks Questions
𝑎 𝑏
1. If the three points (h, 0), (a, b) and (0, k) lies on a line show that +𝑘 = 1.

2. Find the points on x-axis whose distance from line 4x + 3y = 12. Ans : (8, 0) and (
-2, 0)
3. The line through the points (h, 3) and (4, 1) intersects the line 7x -9y -19 = 0 at
right angle. Find the value of h.
Ans: 22/9
4. Find the equation of a line passing through the point (-3, 5) and perpendicular to
line through the points (2, 5) and (-3, 6).
Ans: y = 5x + 20
5. Find the value of k for which the line (k-3)x – (4 – k2)y + k2 – 7k + 6 = 0 is

(i) parallel to x-axis (ii) parallel to y-axis (iii) passing through origin
Ans: (i) k = 3 (ii) k = ±2 (iii) k = 1 or 6
6. Find the angles between the lines y = (2 − √3)(𝑥 + 5)𝑎𝑛𝑑 𝑦 = (2 + √3)(𝑥 − 7) .
Ans : 60°
5 marks questions
1. Find the perpendicular distance from the origin to the line joining the points
(cos 𝜃, sin 𝜃) 𝑎𝑛𝑑 (cos 𝜑, sin 𝜑)
sin(𝜑−𝜃)
Ans: | (𝜑−𝜃) |.
2 sin
2

2. Find the equations of the line which is equidistant from parallel lines 9x + 6y – 7
= 0 and 3x + 2y + 6 = 0.
Ans: 18x + 12y + 11 = 0
3. In what direction should a line be drawn through the point (1, 2), so that its point
√6
of intersection with the line x +y =4 is at a distance from the given point. Ans:
3

making angle of 75° 𝑜𝑟 15° with positive direction of x axis.


4. A variable line passes through a fixed-point P. if the algebraic sum of
perpendicular distances from the points (2, 0), (0, 2) and (1, 1) from the line is 0.

56
Find the coordinates of point P.
Ans : P(1, 1)
5. Find the equation of line which passes through the point (3, 4) and cuts off
intercepts from the coordinate axes such that their sum is 14.
Ans : x + y = 7 and 4x + 3y = 24
6. Find the image of the point (3, 8) with respect to the line x + 3y = 7 assuming the
line to be a plane mirror.
Ans : (-1,-4)
7. Find the equations of the lines, which cut-off intercepts on the axes whose sum
and product are 1 and – 6, respectively.
ANS. 2x – 3y = 6 , -3x + 2y = 6
8. Find the equation of the line passing through the point (4,5) and equally inclined
to the line 3x = 4y + 7 and 5y = 12x + 6
9 −7
Ans : 𝑦 − 5 = 7 (𝑥 − 4) 𝑎𝑛𝑑 𝑦 − 5 = (𝑥 − 4)
9

Case Based questions:

1. Three friends are standing in a ground. Their position is A (0, 0), B (4, 0) and C
(2, 2) with respect to a mango tree in the ground. Based on this information
answer the following questions:
(i) What is the slope of line segment joining AB?
(ii) What is the slope of line segment joining BC?
(iii) Find the equation of line AB and BC.
0−0
Solution: (i) Slope of AB = 4−0 = 0

57
2−0 1
(ii) Slope of BC = 2−4= − 2

(ii) Equation of AB : (y -0) = 0(x -0) or y = 0


Equation of BC : (y – 0) = - ½ (x – 4) or 2y = 4 – x
2. A person standing at the junction of two straight roads represented by equations
2x – 3y +4 = 0 and 3x + 4y – 5 = 0. He wants to reach the path whose equation is
6x – 7y + 8 = 0 in the least time.

Based on this information answer the following questions:


(i) Find the location of the point the man is standing.
(ii) What is the slope of path he should follow?
(iii) Write the equation of path to reach in least time.
Sol: (i) Solving the given equations 2x – 3y +4 = 0 and 3x + 4y – 5 = 0 we get 𝑥 =
1 22
− 17 𝑎𝑛𝑑 𝑦 = 17
6 7
(ii) Slope of line 6x – 7y + 8 = 0 is 7 , Therefore the slope of his path = − 6.
22 7 1
(iii)Equation of path he should follow (𝑦 − 17) = − 6 (𝑥 + 17) ➔ 119x +102y = 125

58
CONIC SECTIONS

Sections of Cone:

The conic sections are the non-


degenerate curves generated by the
intersections of a plane with one or two
nappes of a cone. For a plane
perpendicular to the axis of the cone, a
circle is produced. For a plane that is
not perpendicular to the axis and that
intersects only a single nappe, the curve
produced is either an ellipse or a
parabola. The curve produced by a
plane intersecting both nappes is a
hyperbola.

The ellipse and hyperbola are known as central conics.

Circle
A circle is the set of all points in a plane, which are at a fixed distance from a fixed point
in the plane. The fixed point is called the centre of the circle and the distance from centre
to any point on the circle is called the radius of the circle.
The equation of a circle with radius r having centre (h, k) is given by (x – h)2 + (y – k)2 =
r2.

The general equation of the circle is given by x2 + y2 + 2gx + 2fy + c = 0 , where, g, f and c
are constants.

• The centre of the circle is (-g, -f).


• The radius of the circle is r =√𝑔2 + 𝑓 2 − 𝑐

The general equation of the circle passing through origin is x2 + y2 = r2.

The parametric equation of the circle x2 + y2 = r2 are given by x = r cos θ,

y = r sin θ, where θ is the parameter and the parametric equation of the circle (x – h)2 +
(y – k)2 = r2 are given by x = h + r cos θ, y = k + r sin θ.

The general equation of the circle involves three constants which implies that at
least three conditions are required to determine a circle uniquely.

Parabola
A parabola is the set of points P whose distances from a fixed point F in the plane are
equal to their distance from a fixed line l in the plane. The fixed point F is called focus and
the fixed line l is the directrix of the parabola.

59
Main Facts about the Parabola

Forms of 2 y2 = - x2 = x2 = -
y = 4ax
parabola 4ax 4ay 4ay
Axis of
y=0 y=0 x=0 x=0
parabola
Directrix
x=-
of x = a y = -a y=a
a
parabola
(0,
Vertex (0, 0) (0, 0) (0, 0)
0)
(a, (-a,
Focus (0, a) (0, -a)
0) 0)
Length
of latus 4a 4a 4a 4a
rectum
Focal |x +
|x – a| |y + a| |y – a|
length a|

Ellipse
An ellipse is the set of all points in a plane such that the sum of whose distances from
two fixed points is constant. OR
An ellipse is the set of all points in the plane whose distances from a fixed point in the
plane bears a constant ratio, less than to their distance from a fixed point in the plane.
The fixed point is called focus, the fixed line a directrix and the constant ratio (e) the
eccentricity of the ellipse. We have two standard forms of ellipse i.e.
𝑥2 𝑦2 𝑥2 𝑦2
(i)𝑎2 + = 1 (ii) + = 1 in both the cases a > b and 𝑏 2 = 𝑎2 (1 − 𝑒 2 ), e<1
𝑏2 𝑏2 𝑎2

Main Facts about the Ellipse

60
Hyperbola
A hyperbola is the locus of a point in a plane which moves in such a way that the ratio of
its distance from a fixed point in the same plane to its distance from a fixed line is always
constant which is always greater than unity. The fixed point is called the focus, the fixed
line is called the directrix and the constant ratio, generally denoted bye, is known as the
eccentricity of the hyperbola.

We have two standard forms of hyperbola i.e.

Main Facts about Hyperbola

Examples:

1. Find the equation of a circle with centre (0,2) and radius =2

Solution:

61
We have equation of a circle as (𝑥 − ℎ)2 + (𝑦 − 𝑘)2 = 𝑟 2
Put h = 0, k = 2 and r = 2 we have (𝑥 − 0)2 + (𝑦 − 2)2 = 22 or X2 + y2 - 4y = 0

2. Find the centre and radius of the circle given by x2 + y2 -2x +4y = 8.
Sol: The given equation of circle may be written as (𝑥 − 1)2 + (𝑦 + 2)2 = 3
Hence centre is (1, -2) and radius = √3 units
3. Find the equation of a circle whose end points of one diameter are ( -2,3) and (0,
-1)
Sol: the equation of such circle is given by (x –x1)(x- x2) + (y –y1) (y – y2) = 0
Put the coordinates of ends of diameter (x +2)(x-0) +(y- 3)(y+1) = 0
X2 + 2x + y2 -2y – 3 = 0 or X2 + y2 + 2x -2y – 3 = 0 is the required equation of circle
4. Find the equation of circle having centre (3, -4) and touching the line 5x + 12y –
19 = 0
Sol: Given centre of circle is (3, -4) since it is touching the line 5x + 12y – 19 = 0
Therefore the radius of circle is perpendicular distance of line from centre of
5.3+12.(−4)−19
circle = | √52+ 2 | = 4
12
Hence the equation of circle is (𝑥 − 3)2 + (𝑦 + 4)2 = 42
x2 +y2 -6x + 8y +9 = 0
5. Find the coordinates of points on the parabola y2 = 8x whose focal distance is 4.
Sol:
Given equation of parabola is y2 = 8x ⇒ y2 = 4 .2.x hence a = 2 and focus (2, 0)
Now let P(x, y) be any point on the parabola then PF = 4 ⇒ √(𝑥 − 2)2 + (𝑦 − 0)2
=4

⇒x2 – 4x +4 + 8x = 16 ⇒ x2 + 4x – 12 = 0 ⇒ (x+6)(x-2) = 0

⇒x = -6 and 2 from x = 2 we get y2 = 16 or 𝑦 = ±4

The required points are (2, ±4)


6. Find the equation of parabola with vertex at (0, 0) and focus (0, 2).
Equation of parabola is x2 = 4ay put a =2 we get x2 = 8y
7. Find the coordinates of the vertices, foci, eccentricity and length of latus rectum
𝑥2 𝑦2
of the hyperbola − =1
25 4
Sol: By comparison with standard form we have a= 5 and b =2 → c = √52+ 22 =
√29
√29
Vertices are(±𝑎, 0) = (±5,0), foci = (±𝑐, 0) = (±√29, 0), e = c/a = and LLR=
5
2𝑏 2
= 8/5
𝑎
8. Find the area of the triangle formed by the lines joining the vertices of the
parabola x2 = -36y to ends of the latus rectum.
Sol: given equation of parabola is x2 = -36y which gives a = 9 hence focus is (0, -9)
we know that latus rectum passes through focus therefore ordinate of ends of
LLR and focus are same -> x2 = -36(-9) -> x2 = 324 → x = ±18 → ends of LLR (18,
-9), (-18, -9)
Area of triangle = ½ x 36 x 9 = 162 sq unit.

MCQs

62
1. The area of a circle cantered at (1, 2) and passing through (4, 6) is
(a) 5𝜋 (b) 10𝜋 (c) 25𝜋 (d) none of these Ans : C
2. Equation of a circle with centre on Y axis and passing through the origin and the
point (2,3) is
(a) 𝑥 2 + 𝑦 2 + 13𝑦 = 0 (b) 3𝑥 2 + 3𝑦 2 + 13𝑥 + 3 = 0
(c) 6 𝑥 2 + 6𝑦 2 − 13𝑦 = 0 (d) 𝑥 2 + 𝑦 2 + 13𝑥 + 3 = 0 Ans : C
3. If the parabola 𝑦 = 4𝑎𝑥 passes through the point (3,2) then the length of latus
2

rectum is (a) 2/3 (b) 4/3 (c) 1/3 (d) 4 Ans: b


𝑥2 𝑦2
4. Length of latus rectum of ellipse + 12 = 1 is
4
4
5. (a) 4 (b) 3 (c) 8 (d) Ans: d
√3
6. Where does the point (0, 0) lies with reference to the circle 𝑥 2 + 𝑦 2 − 𝑥 + 3 = 0 (
a) interior (b) exterior (c) on the circle (d) none of these

2 Marks Questions
1. The coordinates of focus, and eccentricity of the parabola 3y2 = 8x. Ans (2/3, 0)
and 1
2. Show that the line 3x + 4y +7=0 touches the circle x2 + y2 – 4x - 6y -12 = 0
3. Find the equation of the circles passing through the points (1, 1) and (2, 2) with
its radius unity. Ans: x2 + y2 – 4x - 2y + 4 = 0, x2 + y2 – 2x
- 4y + 4 = 0
4. Find the equation of ellipse whose foci are at (±5, 0) and length of semi major axis
𝑥2 𝑦2
is 6 units. Ans: 36 + 11 = 1
5. Find the equation of parabola with focus at (1, - 1) and vertices at (2,1) Ans: 4x2 +
y2 – 4xy +8x + 46y -71 = 0
6. Find the equation of a circle passing through the origin and having intercepts ‘a’
and ‘b’ on the coordinate axes. Ans : x2 + y2 – ax - by = 0
7. What is the length of latus rectum of ellipse 16x2 + y2 = 16? Ans: ½
8. Find the equation of hyperbola, the length of whose latus rectum is 8 and the
3
eccentricity is . Ans : 4x2 – 5y2 = 100
√5
9. Find the equation of hyperbola, with vertices at (±4, 0) and foci at (±6, 0). Ans :
5x2 – 4y2 = 80.
10. Prove that the points ( -1, 6), (5, 2), (7,0) and (-1, -4) are not concyclic.
3 marks questions
1. If line y = mx + c touches the parabola y2 = 4x, then find the value of m. Ans : m =
1
2. Find the coordinates of the points on parabola y2 = 8x whose focal distance is 4.
Ans : (2, ±4)
3. Find the equation of the set of all points such that the difference of their distance
from (4, 0) and (-4, 0) is always 2 units. Write the name of curve. Ans : 15x2 – y2 =
15

63
4. Find the equation of the set of all points such that the sum of their distance from
(3, 0) and (9, 0) is always 12 units. Write the name of curve.
5. Find the equation of hyperbola whose conjugate axis is 5 and distances between
𝑥2 4𝑦 2
foci is 13 unit. Ans : 36 − =1
25
5 Marks questions
1. Find the equation of ellipse in which length of minor axis is equal to distance
between foci, given length of latus rectum is 10 units and major axis is along X
axis. Ans : x2 + 2y2 =100
2. Find the area of the triangle formed by lines joining the vertices of parabola x2 = -
36y to the ends of the latus rectum. Ans : 162
square units.
3. Prove that the line lx + my + n = 0 will touch the parabola y2 = 4ax, if ln = am2.
4. If the lines 2x – 3y = 5 and 3x – 4y = 7 are the diameter of a circle with area 154
units, then obtain the equation of circle. Ans: x2 + y2 – 2x +
2y = 47
5. Find the equation of a circle of radius 5 units and touching another circle x2 + y2 –
2x - 4y = 20 Ans : x2 + y2 – 18x - 16y
+ 120 =0

Case Study Questions:


1. The cable of a uniformly loaded suspension bridge hangs in the form of a parabola. The
roadway which is horizontal and 100m long is supported by vertical wires attached to
the cable, the longest wire being 30m and the shortest being 6m. Based on this
information answer the following questions.

(i) What may be the equation of parabola?


(ii) What will be the x- coordinate of the top of a supporting wire attached to
the roadway 18m from middle?

64
(iii) What will be the length of a supporting wire attached to the roadway 18m
from middle?
Ans: (i) Equation of parabola is 6x2 = 25y (ii) x coordinate of top is 18
(III) The length of required wire is 9.11 m
2. A man running a racecourse note that the sum of the distances from the two flag
posts from him is always 10 m and the distance between flag posts is 8m. Based
on this information answer the following questions.

(i)What is an ellipse?
(ii)According the given situation what is the value of lengths of semi
major and semi minor axis?
(ii)Write the name and equation of the path traced by the runner.

Ans: (i) An ellipse the locus of a moving point which moves such that sum of its
distances from two fixed points is always constant.

(ii) Length of semi major axis is 5 and that of semi minor axis is 3 units

𝑥2 𝑦2
(III) The path is an ellipse and equation is 25 + =1
9

65
Introduction to 3D geometry

Coordinate Axes
In three dimensions, the coordinate axes of a rectangular Cartesian coordinate system
are three mutually perpendicular lines. These axes are called the X, Y and Z axes.

Coordinate Planes
The three planes determined by the pair of axes are the coordinate planes. These planes
are called XY, YZ and ZX plane and they divide the space into eight regions known as
octants.

Coordinates of a Point in Space


The coordinates of a point in the space
are the perpendicular distances from P
on three mutually perpendicular
coordinate planes YZ, ZX, and XY
respectively. The coordinates of a point
P are written in the form of triplet like
(x, y, z).
The coordinates of any point on

• X-axis is of the form (x, 0,0)


• Y-axis is of the form (0, y, 0)
• Z-axis is of the form (0, 0, z)
• XY-plane are of the form (x, y, 0)
• YZ-plane is of the form (0, y, z)
• ZX-plane are of the form (x, 0, z)

Sign of coordinates of a point in octant in which space is divided

66
Octant I II III IV V VI VII VIII
X + - - + + - - +
Y + + - - + + - -
Z + + + + - - - -
Distance Formula
The distance between two points P(x1, y1, z1) and Q(x2, y2, z2) is given by
PQ= √(𝑥2 − 𝑥1 )2 + (𝑦2 − 𝑦1 )2 + (𝑧2 − 𝑧1 )2

The distance of a point P(x, y, z) from the origin O (0, 0, 0) is given by OP =


√𝑥 2 + 𝑦 2 + 𝑧 2

Three points A, B and C are collinear if sum of two small distances is equal to largest
length.

Properties of some geometrical shapes:

1. Scalene Triangle – all 3 sides are un equal


2. Isosceles Triangle – any 2 sides are equal in length
3. Equilateral Triangle – All 3 sides are equal
4. Right angled triangle – Help of Pythagoras theorem
5. Opposite sides of parallelogram are equal and diagonals are not equal
6. All sides of rhombus are equal and diagonals are not equal
7. Opposite sides and diagonals of rectangle are equal.
8. All sides and diagonals of square are equal.

Examples:

1. If the distance between the points (a, 2, 1) and (-1, 1, 1) is 5, then find the values
of a.
Solution: Given 52 = (𝑎 − 1)2 + (2 + 1)2 + (1 − 1)2
= a2 - 2a – 15= 0 ⇒ (a-5) (a+3) =0 ⇒ a = 5 and -3
2. What is the distance of the point (3, 4, 5) from the YZ Plane?
Sol:
For perpendicular distance of this point in YZ plane we need a point just opposite
to it.
It is (0, 4, 5) hence its distance will be √(3 − 0)2 + (4 − 4)2 + (5 − 5)2 = 3 units
3. Using the distance formula prove that the points P (2, 4, 6), Q (-2,-2,-2) and R (6,
10, 14) are collinear.
Solution: PQ= √(2 + 2)2 + (4 + 2)2 + (6 + 2)2 = √ 16 + 36 + 64
= √116 = 2√29
QR= √(6 + 2)2 + (10 + 2)2 + (14 + 2)2 = √64 + 144 + 256 = √464 =4√29

67
PR = √(6 − 2)2 + (10 − 4)2 + (14 − 6)2 = √16 + 36 + 64 = √116 = 2√29
It can be seen that PQ+ PR = QR hence points are collinear.
4. What is the length of foot of perpendicular from the point P (3, 4, 5) on the y-
axis?
Solution: Let M be the foot of perpendicular of the point P on y –axis. Its
coordinates will be (0, 4, 0). Therefore using distance formula length of
perpendicular MP is √34 𝑢𝑛𝑖𝑡𝑠
5. Show that the triangle formed by points A( 0,4,1), B(2,3, -1) and C(4,5,0) forms a
right triangle.
Sol: AB2 = (2-0)2 + (3 -4)2 + (-1 -1)2 = 4 + 1 + 4 = 9
BC2 = (4 -2)2 + (5 -3)2 + (0 +1)2 = 4 + 4 + 1 = 9 and
AC2 = (4-0)2 + (5 -4)2 + (0-1)2 = 16 + 1+ 1 = 18 it’s obvious that AB2 + BC2 = AC2
therefore ABC is a right angled triangle.
MCQ’s

1. A plane is parallel XY-plane, so it is perpendicular to


(a) z-axis (b) y-axis (c) x-axis (d) None of these
Answer : A
2. The locus of a point for which y = 0, z = 0 is
(a) equation of x-axis (b) equation of y-axis
(c) equation of z-axis (d) None of these
Answer: A
3. The points (5, 2, 4), (6, -1, 2) and (8, -7, k) are collinear, if k is equal to
(a) -2 (b) 2 (c) 3 (d) -1
Answer : A
4. The point A(1, -1, 3), B(2, -4, 5) and C(5, -13, 11) are
(a) collinear (b) non-collinear
(c) Do not say anything (d) None of these
Answer : A
5. Question. The points (5, – 4, 2),(4,– 3, 1), (7, -6, 4) and (8, – 7, 5) are the vertices
of
(a) a rectangle (b) a square (c) a parallelogram (d) None of these
Answer : C
6. If the coordinates of the vertices of a ΔABC are A(-1, 3, 2), B(2, 3, 5) and C(3, 5, -
2), then ∠A is equal to

(a) 45° (b) 60° (c) 90° (d) 30°


Answer : A
7. The points A(5, -1, 1), B(7, -4, 7), C(1, -6, 10) and D(-1, -3, 4) are vertices of a
(a) square (b) rhombus (c) rectangle (d) None of these
Answer : B
Extra questions:

68
1. Name the octant in which the following points lie: (-3, 4, 3), (4, -1, 4) and (-1, 2,
3). Ans: II and IV and VI octant
2. Find the image of (-5, 4, -3) in XZ plane. Ans: (-5, -4, -3)
3. Find the distance between the pairs of points (2, 3, 5) and (4, 3, 1). Ans : 2√5
units
4. Verify that (0, 7, -10), (1, 6, -6) and (4, 9, -6) are vertices of an isosceles triangle.
5. Verify that (0, 7, 10), (-1, 6, 6) and (-4, 9, 6) are the vertices of a right angled
triangle.
6. Verify that ( -1,2,1), (1, -2, 5), (4, 07, 8) and (2, -3, 4) are vertices of a
parallelogram.
7. Find the equation of the set of points P, which are equidistant from (1, 2, 3) and
(3, 2,-1). Ans : x-2z = 0
8. Find the length of medians of triangle formed by vertices A(0,0,6), B(0,4,0) and
C( 6,0,0).
9. Find the coordinates of vertices of a cube of edge 2 units. One of its vertex is at
origin and 3 edges along coordinate axes.
Ans: (0, 0, 0), (0, 2, 0), (0, 0, 2), (2,0,0), (2,2,0), (2,0,2), (0,2,2), (2,2,2)
10. Show that the distances of the point (1, 2, 3) from coordinate axes are
√13, √10, √5 units.
11. Show that the points (0, 4, 1), (2, 3, -1), (4, 5, 0) and (2, 6, 2) forms a square.
12. Show that the points (1,3,4), (-1,6,10), (-7,4,7) and (-5,1,1) forms a rhombus.
13. Show that the points A(1,3,0) B( -5, 5, 2), C( -9, -1, -2) and D (-3, -3, 0) are the
vertices of a parallelogram but not a rectangle.
14. Find the coordinates of a point on y axis that is at a distance of 5√2 units from
the point P( 3, -2, -5).

69
LIMITS AND DERIVATIVES
LIMITS
1. Limit of a function
Let f be a function defined on an open interval (a-h,a+h ) , h>0 ,then
(i) Left hand limit :-A real number k is said to be left hand limit of f(x) at a , if
for all x sufficiently close to a , on the left of a but not equal to a , the value
of f(x) can made as close as desired to the number k . we write lim− f(x) =
x→a
k.
(ii) Right hand limit :-A real number k’ is said to be right hand limit of f(x) at
x=a , if for all x sufficiently close to a , on the right of a but not equal to a ,
the value of f(x) can made as close as desired to the number k’ . we write
lim+ f(x) = k′ .
x→a
(iii) The limit of f(x) at x-a will exist if k=k’ i.e lim− f(x) = lim+ f(x) .
x→a x→a
(iv) If L.H.L ≠ R.H.L then we say that lim f(x) does not exist.
x→a
2. Evaluation of Right hand and Left hand limits
For R.H.L
Put x=a+h and evaluate lim f(a + h)
x→0
For L.H.L
Put x=a-h and evaluate lim f(a − h) .
x→0
3. Expected learning outcome :
i) Students will imagine and observe the intuitive idea of limits through graphs.

ii ) Students will comprehend one sided limits both graphically and through the
definition.

iii )Students will apply the concept of one sided limits to find the existence of
the limit.

iv) They observe the nature of the function involving the limit.

4. Concept mapping :-

LIMITS

ONE SIDED
ALGEBRA OF LIMITS
LIMITS

STANDARD
70 FORMULAE
4.
Algebra of limits :-
(a) lim[f(x) + g(x)] = lim f(x) + lim g(x) b) lim[f(x) − g(x)] = lim f(x) -
x→a x→a x→a x→a x→a
lim g(x)
x→a
f(x) lim f(x)
(b) lim[f(x). g(x)] = lim f(x). lim g(x) d) lim g(x) = lim
x→a
, . lim g(x) ≠ 0
x→a x→a x→a x→a g(x) x→a
x→a
(c) lim[(λf)(x)] = λ lim f(x)
x→a x→a
5. Important Limits formulas
xn −an Sinx
(a) lim = nan−1 b) lim = 1
x→a x−a x→0 x

tanx ax −1
c) lim = 1 d) lim = log e a
x→0 x x→0 x

ex −1 log(1+x)
e) lim = 1 f) lim = 1
x→0 x x→0 x
1
1 x
g) lim(1 + x)x = e h) lim (1 + x) = e
x→0 x→∞

Problems based on limits

Q:-1
.

Q:-2
Soloution:- Let put x+1 = y Then y→ 1 as x → 0
(𝑥+1)5 −1 𝑦 5 −1 𝑦 5 −15
Accordingly lim = lim = lim = 5(1)5−1 = 5
𝑥⟶∞ 𝑥 𝑥⟶1 𝑦−1 𝑥⟶1 𝑦−1

Q:-3

71
𝑧⃓ 1/3 −1
Q:- Evaluate:- lim 𝑧⃓ 1/6 −1
𝑧⃓⟶1

Solution : At z=1, the value of the given function takes the form 0/0
Put z1/6 = x sothat z→1 as x→1
1
𝑧⃓ 3 −1 𝑥 2 −1
lim 1 = lim = 2 (1)2-1 = 2
𝑧⃓⟶1 𝑧⃓ 6 −1 𝑥⟶1 𝑥−1

cos 2𝑥−1
Q:- Evaluate lim
𝑥⟶0 𝑐𝑜𝑠𝑥−1

Solution:-

72
1 Marks questions
Evaluate :-
sin2x Sinx0
Q:-1 lim {( )} Q:-2 lim
x→0 x x→0 x

ax −bx x4 −16
Q:-3 lim Q:-4 lim
x→0 x x→2 x−2
22 4x+3
Q:-5 lim (x − ) Q:- 6 lim
x→π 7 x→4 x−2
sin(π−x)
Q:-7 lim {( )}
x→π x(π−x)

2 marks questions
x2 −9x+20 1−Cos2x
Q:-8 lim Q:-9 lim
x→5 x2 −6x+5 x→0 x2

xn −3n Sin5x−Sin3x
Q:-10 Find n if lim = 108 , n ϵ N Q:-11 lim
x→2 x−3 x→0 Sinx
1

1−x 3
Q:-12 lim 2
x→1 1−x− 3

3 marks questions
Sinx−Cosx tanx−Sinx
Q:-13 limπ π Q:-14 lim
x→ x− x→0 x3
4 4

Q:- 15

and if lim f(x) = f (1), Find the possible value of


x→1
a and b? +
(x+y)Sec(x+y)−xSecx log(1+ x3 )
Q:-16 lim Q:-17 lim
x→0 y x→0 Sin3 x

x2 −xlogx+2logx−4 1−Cosx √Cos2x


Q:-18 lim Q:-19 lim
x→0 x−2 x→0 x2

Derivatives
First Principle /ab- initio method: - Let y = f(x) be a real valued function then
dy f(x+h)−f(x)
= lim .
dx h→0 h

Geometrical Significance: - The derivative f ’(a) represents the slope of the tangent to
the curve y= f(x) at the point where x=a.

73
Physical significance:- The derivative f ’(x) represents rate of change in y with respect
to x .
d dv du
Product rule :- (u. v) = u dx + v dx
dx
du dv
d u v −u
Quotient rule:- ( )= dx dx
dx v v2

Important formulae for differentiation


d d
1. (x n ) = n x n−1 . 2. (ax ) = ax loga
dx dx
d d 1
3. dx (ex ) = ex 4. (log e x) =
dx x

d d
5. dx (Sinx ) = Cosx 6 . (Cosx ) = Sinx
dx
d d
7. dx (tanx ) = Sec2x 8. (Cotx ) = - Cosec2x
dx
d d
9. dx (Secx ) = Secx tanx 10. dx (Cosecx ) = - Cosecx .Cotx

Solved problems
1. Find the derivative of f(x) = x3- 27 by using first principle ?
Solution:-

.
x+1
2. Find the derivative of f(x) = by using first Principle ?
x−1

74
ax+b dy
3. If y = , Find dx
cx+d

4. If f(x) = (5x3 +3x-1)(x-1)

MCQs
x−4
1. If f(x) = 2 x, then f ‘(1) is equal to

5 4
(a) (b) (c) 1 (d) 0
4 5

xn −an
2. If f(x) = for some constant ‘a’, then f ‘(a) is equal to
x−a

1
(a) 1 (b) 0 (c) does not exist (d) 2

1
1+ 2 dy
3. If y = x
1 , thendx is equal to
1− 2
x

−4x −4x 1−x2 4x


(a) (b) (c) (d) x2−1
(x2 −1)2 x2 −1 4x

Sin x+cos x dy
4. If y= , then value of dx at x = 0 is
Sin x−cos x

75
1
(a) -2 (b) 0 (c) (d) does not exist.
2

5. If f(x) f(y) = f(x+ y) for all x, y f(5) = 2 f ‘(0)=3 then f ‘(5) = ………….

(a) 5 (b) 6 (c) 0 (d) 1

sin(x+9) dy
6. If y= , then dx at x = 0 is …………..
cos x

(a) Cos 9 (b) sin 9 (c) 0 (d) 1

7. If f(x) = 1-x + x2- x3 + ……………. –x99 + x100 then f ‘(1) =…………………….

(a) 150 (b) -50 (c) -150 (d) 50


1
8. If f(x) = √x + , then f ‘(x) at x = 1 is………
√x

1
(a) 1 (b) ½ (c) (d) 0
√2

x2 x100
9. If f(x) = 1 + x + + ------------ + then f ‘(1) = --------------
2 100

(a) 0.01 (b) 100 (c) 0 (d) does not exist

10. The derivative of x sin x is ………………

(a) x sin x + cos x (b) x cos x + sin x (c) x sin x- cos x (d) x cos x-sin x

Q.11. Differentiate 5sinx - 6cosx + 7 w.r.t.x


1−cosx
Q 12. Find the derivative of function 1+cosx w.r.t.x

Q 13. Compute the derivative of 6x100 – x55 + x

dy
Q 14. If y = sin(x + 1), find dx

Q15. Differentiate w.r.t. x : x cos x

Q 16. Find derivative of f(x) = sin2x


π
Q 17. If f(x) = x sin x , prove that f ‘(2 ) = 1

Q 18. Differentiate log(log x) w.r.t. x


1
Q 19. Differentiate w.r.t .x : (√x + )2
√x

76
Q 20. Find the derivative of (7x4 + 3x2 - 2x + 1) (x – 1)

ex
Q 21. Differentiate 1+sinx w.r.t.x

secx−1
Q22. Find derivative of .
secx+1

secx+tanx
Q 23. Differentiate w.r.t.x : secx−tanx

1−sinx π
Q 24. Find the derivative of at x = 2
1+cosx

1
Q 25. Differentiate w.r.t.x (x2 + x2 )3

dy 1
Q26. Find , when y = 3tanx + 5logax + √x - 3ex + x
dx

f ′ (1)
Q 27. If f(x) = α xn, prove that α = n

Q28. If f(x) = mx + c and f(0) = f ‘(0) = 1 what is f(2) ?

Q29. Find the derivative of x. sin x. log x .

d ax + b
Q30. Evaluate: ( )
dx px2 +qx+r
dy Sinx−cosx
Q:-31 Find dx ,if y= Sinx+Cosx

Q:-31 find the derivative of following function w.r.t .x by using first principle.
3 1
1 ) √cosecx . 2) e√tanx 3) √sinx . 4) x + x

Answers (Limit)

1 -2 11 2
2 π 12 1
180 2
3 a
log(b) 13 √2
4 32 14 1
2
5 22 15 b=4,a=0
π−
7
6 19 16 X tan x Sec x + Sec x
2
7 1 17 1
π

77
8 1 18 2-log2
π
9 2 19 3
2
10 4

Derivatives :-
1.(a) 2.(c) 3.(a) 4.(a) 5.(b) 6.(a) 7.(d) 8. (d) 9. (b) 10.(b)

2sinx
7. 5cosx + 6sinx 12. (1+cosx)2

13 600x99 – 55x54 + 1 14 Cos(x + 1)

15 -x sin x + cos x 16 2 sin x cos x

1
17 Proof 18
xlogx

1
19 1- x2 20 35x4-28x3 + 9x2 -10x + 3

ex (1+sinx−cosx) 2sinx
21 22
(1+sinx)2 (1+cosx)2

2cosx
23 24 0
(1−sinx)2

6 6
25 6x5 + 6x - -
x3 x7

5
26 3sec2x + xloga + ½ x – ½ -3ex + (- 1)x- 2

27 Proof

28 3

29 sin x log x + x cos .log x + sin x

−apx2 −2bpx+ar−bq
30 ( px2 + qx+r)2
1 1
31 f ‘(x) = 3x2/3 , f ‘(0) = 3(0)2/3 , which does not exists

f ‘(1) = 1/3

78
STATISTICS
∑ 𝑥𝑖
Mean for raw data 𝑥̅ = 𝑛
∑ 𝑓 𝑖 𝑥𝑖
Mean for discrete data 𝑥̅ = ∑ 𝑥𝑖

∑ 𝑓𝑖 𝑑𝑖 𝑥𝑖 −𝑎
Mean for grouped data 𝑥̅ = 𝑎+ = ∑ 𝑥𝑖
X h ; where 𝑑𝑖 = ℎ

Measure of Dispersion
The dispersion is the measure of variations in the values of the variable. It measures
the degree of scatteredness of the observation in a distribution around the central
value.

Range
the measure of dispersion which is easiest to understand and easiest to calculate is
the range.
Range is defined as the difference between two extreme observations of the
distribution.
Range of distribution = Largest observation – Smallest observation.

Mean Deviation
Mean deviation for ungrouped data
For n observations x1, x2, x3,…, xn, the mean deviation about their mean x¯ is given by

∑|𝑥𝑖 − 𝑥̅ |
𝑀𝐷(𝑥̅ ) =
𝑁
Mean deviation about their median M is given by

∑|𝑥𝑖 − 𝑀|
𝑀𝐷(𝑀) =
𝑁
Mean deviation for discrete frequency distribution
Let the given data consist of discrete observations x1, x2, x3,……., xn occurring with
frequencies f1, f2, f3,……., fn respectively in case

∑ 𝑓𝑖 |𝑥𝑖 − 𝑥̅ | ∑ 𝑓𝑖 |𝑥𝑖 − 𝑥̅ |
𝑀𝐷(𝑥̅ ) = =
∑ 𝑓𝑖 𝑁

Mean deviation about their Median M is given by

∑ 𝑓𝑖 |𝑥𝑖 − 𝑀| ∑ 𝑓𝑖 |𝑥𝑖 − 𝑀|
𝑀𝐷(𝑀) = =
∑ 𝑓𝑖 𝑁

Mean deviation for continuous frequency distribution

79
∑ 𝑓𝑖 |𝑥𝑖 − 𝑥̅ |
𝑀𝐷(𝑥̅ ) =
𝑁

∑ 𝑓𝑖 |𝑥𝑖 − 𝑀|
𝑀𝐷(𝑀) =
𝑁
where xi are the mid-points of the classes, x¯ and M are respectively, the mean and
median of the distribution.

Variance
Variance is the arithmetic mean of the square of the deviation about mean x¯.
Let x1, x2, ……xn be n observations with x¯ as the mean, then the variance denoted by
σ2, is given by

Standard deviation
If σ2 is the variance, then σ is called the standard deviation is given by

Standard deviation of a discrete frequency distribution is given by

Standard deviation of a continuous frequency distribution is given by

Solved questions

1. Find the mean deviation about the mean for the following data 6,7,10,12,13,4,8,12.

∑ 𝑥𝑖 6+7+10+12+13+4+8+12 72
Solution:- Mean 𝑥̅ = = = =9
𝑛 8 8

80
𝑥𝑖 𝑥𝑖 − 𝑥̅ |𝑥𝑖 − 𝑥̅ |
6 -3 3
7 -2 2
10 1 1
12 3 3
13 4 4
4 -5 5
8 -1 1
12 3 3
∑|𝑥𝑖 − 𝑥̅ | = 22

22
M.D = = 2.75
8

2. Find the mean deviation about the meadian for the following data
3,9,5,3,12,10,18,4,7,19,21.

Soloution :- Arrange the data into ascending order.

3,3,4,5,7,9,10,12,18,19,21

11+1 𝑡ℎ
N = no o term = 11 (odd) , median = ( ) = 6th observation = 9
2

𝑥𝑖 𝑥𝑖 − 𝑀 |𝑥𝑖 − 𝑀|
3 -6 6
3 -6 6
4 -5 5
5 -4 4
7 -2 2
9 0 0
10 1 1
12 3 3
18 9 9
19 10 10
21 12 12
∑|𝑥𝑖 − 𝑀| = 58

58
𝑀. 𝐷 = 11 = 5.27 .

3. Find mean deviation about the mean for the following data :

81
x 2 5 6 8 10 12
f 2 8 10 7 8 5
Solution :-

𝑥𝑖 𝑓𝑖 𝑓𝑖 𝑥𝑖 |𝑥𝑖 − 𝑥̅ | 𝑓𝑖 |𝑥𝑖 − 𝑥̅ |
2 2 4 5.5 11
5 8 40 2.5 20
6 10 60 1.5 15
8 7 56 0.5 3.5
10 8 80 2.5 20
12 5 60 4.5 22.5
40 ∑ 𝑓𝑖 𝑥𝑖 = 300 92
N = ∑ 𝑓𝑖 = 40 , ∑ 𝑓𝑖 𝑥𝑖 = 300 ∑ 𝑓𝑖 |𝑥𝑖 − 𝑥̅ | = 300

∑ 𝑓 𝑖 𝑥𝑖 300
𝑥̅ = ∑ 𝑥𝑖
= = 7.5
40

1 92
M.D = ∑ 𝑓𝑖 |𝑥𝑖 − 𝑥̅ | = = 2.3
𝑁 40

Q:-4 Find the mean , variance and the standard deviation for the following
frequency distribution :
xi 0-10 10- 20- 30- 40-
20 30 40 50
fi 5 8 15 16 6
Solution:-

C.I 𝑥𝑖 𝑓𝑖 𝑑𝑖 𝑑𝑖2 𝑓𝑖 𝑑𝑖 𝑓𝑖 𝑑𝑖2


0-10 5 5 -2 4 -10 20
10-20 8 15 -1 1 -8 8
20-30 15 25 0 0 0 0
30-40 16 35 1 1 16 16
40-50 6 45 2 4 12 24-
N= 50 ∑ 𝑓𝑖 𝑑𝑖 = 10 ∑ 𝑓𝑖 𝑑𝑖2 = 68
Le t a= 25 (Assumed mean)

∑ 𝑓𝑖 𝑑𝑖 𝑥𝑖 −𝑎
1. Mean 𝑥̅ = 𝑎+ = ∑ 𝑥𝑖
X h ; where 𝑑𝑖 = ℎ

10
= 25 + X 10 = 27 .
50

82
ℎ2
2. Variance = ( 𝑁 ∑ 𝑓𝑖 𝑑𝑖2 − (∑ 𝑓𝑖 𝑑𝑖 )2 )
𝑁2

100 1 3300
= 2500 (50 × 68 − (10)2 ) = 25 (3400 − 100) = = 132.
25

3. Standard Deviation = √132 = 11.48 (approx.)

Q:- 5. The mean and standard deviation of a group of 100 observations were found
to be 20 and 3, respectively. Later on it was found that three observations were
incorrect, which were recorded as 21, 21 and 18. Find the mean and standard
deviation if the incorrect observations are omitted?

Solution:- Number of observations (n) = 100 Incorrect mean


Incorrect standard deviation (σ) = 3

∴ Incorrect sum of observations = 2000


⇒ Correct sum of observations = 2000 – 21 – 21 – 18 = 2000 – 60 = 1940

Correct ∑ 𝑥𝑖2 = 40900-441-441-324 = 39694


∴ Correct Standard Deviation =

MCQs

Q:-1 Standard deviation of first 10 natural numbers is


(a) 5.5 (b)3.87 (c) 2.97 (d)2.87

Q:-2 The range of the data 35,50,48,62,27,39,43,72,56,68 is

83
(a) 25 (b)45 (c) 35 (d)15

Q:-3 If the variance of the numbers 2,4,5,6,8,17 is 23.33, then the variance of
4,8,10,12,16,34 will be
(a) 23.33 (b)46.66 (c) 93.32 (d)none of these
Q:-4 Which of the following is one of the measures of dispersion?
(a) Mean (b)Range (c) Median (d) Mode
Q:-5 The following information relates to a sample of size 60; ∑ 𝑥 2 =18000,∑ 𝑥 =960,
The variance of the data is
(a) 6.63 (b)16 (c) 22 (d)44
Q:-6 Mean deviation of n observations x1,x2,x3,…,xn from their mean 𝑥̅ is given by
∑𝑛
𝑖=1(𝑥𝑖 − 𝑥̅ ) ∑𝑛
𝑖=1(𝑥𝑖 − 𝑥̅ )
(a) ∑𝑛𝑖=1(𝑥𝑖 − 𝑥̅ ) (b) (c) ∑𝑛𝑖=1(𝑥𝑖 − 𝑥̅ ) 2 (d)
𝑛 𝑛

Q:-7 The mean and standard deviation of 6 observations are 8 and 4 respectively. If
each observation is multiplied by 3, the new S.D. is
(a)10 (b)11 (c) 12 (d)13
Q:-8 The variance of the data is 729 , then the standard deviation of the data is
(a) 21 (b) 23 (c) 27 (d) 37
Section- B (3/5 marks)
Q:-9 Find the variance of first n natural numbers.
Q:-10 Find the mean deviation of the data 3,10,10,4,7,10,5 from the mean.
Q :-11Find the mean deviation about the median for the
data3,9,5,3,12,10,18,4,7,19,21(5.27)
Q12.. The mean and variance of 8 observations are 9 and 9.25 respectively. If six of
the observations are 6, 7, 10,12,12, and 13 find the remaining two observations.
Q13. The mean and standard deviation of 100 observations were calculated as 40
and 5.1 respectively by a student who took by mistake 50 instead of 40 for one
observations . What are the correct mean and standard deviation.
Q:-14 The mean and standard deviation of 20 observations are found to be 10 and 2
respectively . on rechecking it was found that an observation 8 was incorrect .
Calculate the correct mean and standard deviation in each of the following cases
I) if wrong item is omitted.
Ii) if it is replaced by 12
Q:-15 . Find the mean deviation about mean for the following data

84
xi 5 7 9 11 13 15
fi 8 6 2 2 2 6
Q16. Find the mean deviation about median for the following data
xi 15 21 27 30
fi 3 5 6 7

Q17. Calculate mean deviation about median of the following frequency distribution.
class 0-6 6-12 12-18 18-24 24-30
frequency 8 10 12 9 5

Q18 Calculate mean deviation about mean of the following frequency distribution.
class 0-10 10-20 20-30 30-40 40-50-
frequency 5 10 20 5 10
Q19. . Calculate mean and standard deviation of the following frequency distribution.
class 20- 30- 40- 50- 60- 70- 80-
30 40 50 60 70 80 90
frequency 3 6 13 15 14 5 4

Q20. Calculate standard deviation of the following frequency distribution.


x 3 8 13 18 23
f 7 10 15 10 6

Q21 CASE STUDY :- Let x1, x2, x3, …, xn be n observations. If each observation is
increased, decreased, multiplied or divided by a non-zero constant a, then the mean
is also increased , decreased ,multiplied or divided by the same non-zero constant a.
In case of variance, if each observation is increased or decreased by the same
constant a, then the variance remains unchanged. But on multiplying or dividing each
𝜎2
observation by same non- zero constant a, the variance 𝜎 2 becomes 𝑎2 𝜎 2 or 𝑎2

respectively. So, we can say that variance is independent of change of origin but not
of the scale.

85
Based on the above information answer the following questions:

I) I f the mean of 10 observations is 18. If each observation is increased by 2, then


find the new mean ?
II) The mean of n observations is 𝑥̅ . If each observation is multiplied by same non-
zero constant k, then what will be new mean?
III) The mean of 7 observations is 25. If 3 is subtracted from each observation, then
calculate the new mean ?

Answer i) 20 ii) k𝑥̅ iii) 22

Answers

1 D 7 C 13 39.9,5 19 55.33,14.94
2.87
2 B 45 8 C 14 I) 10, 1.9697 20 6.12
ii) 10.2, 1.9899
3 c 9 𝑛2 − 1 15 2.72
12
4 B 10 2.57 16 5.93
5 D 11 5.27 17 6.318
6 D 12 6,8 18 9

86
PROBABILITY
Random experiment :- An experiment whose outcome cannot be predicted in
advance or determined in advance is called a random experiment.
Sample space: The set of all possible outcomes
Sample points: Elements of sample space.
Event: A subset of the sample space.
Impossible event : The empty set .
Sure event: The event which has to occur definitely is called a sure event.
Complementary event or ‘not event’ : The set A′ or S – A
Event A or B: The set A ∪ B
Event A and B: The set A ∩ B
Event A and not B: The set A – B
Mutually exclusive event: A and B are mutually exclusive if the occurrence of one
excludes the occurrence of the other . i.e A ∩ B = φ
Independent event:- Two events are said to be independent if occurrence of one
does not depend upon the occurrence or non occurrence of the other.
Equally likely events:- If they have the same chance of occurrence .
Favourable outcome:- Outcomes of an event which ensure the happening of the
event
Exhaustive and mutually exclusive events: Events E1 , E2 ,..., En are mutually exclusive
and exhaustive if E1 ∪ E2 ∪ ...∪ En = S and Ei ∩ Ej = φ, for all i ≠ j
(i) Probability of an event: For a finite sample space with equally likely outcomes
Probability of an event (A)
𝑛(𝐴)
(ii) P(A) = 𝑛(𝑆) , where n(A) = number of elements in the set A, n(S) = number of
elements in the sample space
(iii) If A and B are any two events, then P(A or B) = P(A) + P(B) – P(A and B)
(iv) equivalently, P(A ∪ B) = P(A) + P(B) – P(A ∩ B)
(v) If A and B are mutually exclusive, then P(A or B) = P(A) + P(B)
(vi) If A is any event, then P(not A) = 1 – P(A)
Solved questions
1 7
Q:-1 Let A and B be two independent events such that P(A) = 5 ,P(AUB) = 10 . Then
Find P(𝐵̅)?
Solution :- If A and B are Independent events then P(A∩ 𝐵) = P(A) P(B)
P(AUB) = P(A) + P(B) – P(A∩ 𝐵)

87
= P(A) + P(B) - P(A) P(B)
7 1 1 5 4
= + P(B) - 5 P(B) . = = P(B)
10 5 10 5
5 5 3
P(B) = 8 ⇒ P(𝐵̅) = 1- P(B) = 1- 8 = 8 .

Q:-2 If the letters of the word ‘ATTEMPT’ are written down at random, Find the
chance that all T’s are consecutive .
Solution:- The word we have ‘ATTEMPT’
No of letters used = 07
T= 3 times
7!
No of total word can be formed = 3!

No of words in which all T’s are consecutive = 5!


5! 1
Probability of getting the word in which all T’s are consecutive is = 7! = 7 .
3!

Q:-3 What is the Probability that the birthdays of six different persons will fall in
exactly two calendar months ?
Solution :- No. of total ways = 126
Favourable number of ways = 12C2 X (26 − 2)
Probability that the birthdays of six different persons will fall in exactly two calendar
341
months = 12C2 X (26 − 2) / 126 = 125 .

Q:-4 Find the probability that when a hand of 7 cards is drawn from a well shuffled
deck of 52 cards ,It contains (i) all Kings (ii) 3 Kings (iii) at least 3 Kings ?
Solution :- Total no of possible hands = 52C7 .
(i) Number of hands with 4 kings = 4 C4 X 48C4 ( 4 king cards and 3 other cards ) .
9
P( A hand will have 4 Kings ) = 4 C4 X 48C4 / 52C7 =
1547

Section – 1(I MARK)


Describe the sample space for the indicated experiment 1 to 5
Q. 1. A coin is tossed three times.
Q 2. A die is thrown two times.
Q.3. A coin is tossed four times.
Q 4. A coin is tossed and a die is thrown.
Q5. A coin is tossed and then a die is rolled only in case a head is shown on the coin.

88
Q6. . The numbers 1, 2, 3 and 4 are written separately on four slips of paper. The slips
are put in a box and mixed thoroughly. A person draws two slips from the box, one
after the other, without replacement. Describe the sample space for the experiment.
Q.7. An experiment consists of rolling a die and then tossing a coin once if the number
on the die is even. If the number on the die is odd, the coin is tossed twice. Write the
sample space for this experiment.
Q.8. A box contains 1 red and 3 identical white balls. Two balls are drawn at random
in succession without replacement. Write the sample space for this experiment.
Q.9. . If 3/4 is the probability of an event, what is the probability of the event ‘not A’?
Q 10 If A and B are two events such that P (A) = 0.54, P (B) = 0.69 and P (A ∩ B) =
0.3.Find
1) P (A ∪ B) ii) P (A´ ∩ B´) iii) P (A ∩ B´) iv) P (B ∩ A´)
Q11 Three coins are tossed once. Find the probability of getting
i) 3 heads ii) at least 2 heads iii) at most 2 heads iv) 3 tails
SECTION – B (2 /3 MARKS)

Q.12 One card is drawn from a well shuffled deck of 52 cards. If each outcome is
equally likely, calculate the probability that the card will be (i) a diamond (ii) a black
card (i.e., a club or, a spade).

Q.13 Events E and F are such that P(not E or not F) = 0.25, State whether E and F are
mutually exclusive.
1 1
Q14. Given P(A) = and P(B) = 3 . Find P(A or B), if A and B are mutually exclusive
5
events

Q. 15.. A letter is chosen at random from the word ‘ASSASSINATION’. Find the
probability that letter is (i) a vowel (ii) a consonant ?

Q.16. In Class XI of a school 40% of the students study Mathematics and 30% study
Biology. 10% of the class study both Mathematics and Biology. If a student is selected
at random from the class, find the probability that he will be studying Mathematics or
Biology ?

Q.17 A die has two faces each with number ‘1’, three faces each with number ‘2’ and
one face with number ‘3’. If die is rolled once, determine (i) P(2) (ii) P(1 or 3) (iii)
P(not 3)

SECTION – C (5 MARKS)

Q18. In a class of 60 students, 30 opted for NCC, 32 opted for NSS and 24 opted for
both NCC and NSS. If one of these students is selected at random, find the probability
that

89
(i) The student opted for NCC or NSS.
(ii) The student has opted neither NCC nor NSS.
(iii) The student has opted NSS but not NCC.

Q.19 Out of 100 students, two sections of 40 and 60 are formed. If you and your
friend are among the 100 students, what is the probability that (a) you both enter the
same section? (b) you both enter the different sections?

Q.20 If 4 digit numbers greater than 5000 are randomly formed from the digits
0,1,3,5,7, What is the probability of forming a number divisible by 5 when

i) The digits are repeated ii) The repetition of digits is not allowed
Q:-21 Two students Anil and Ashima appeared in an examination. The probability
that Anil will qualify the examination is 0.05 and that Ashima will qualify the
examination is 0.10. The probability that both will qualify the examination is 0.02.
Find the probability that
(i) Both Anil and Ashima will not qualify the examination.
(ii) At least one of them will not qualify the examination.

iii)Only one of them will qualify the examination

(Case Study)

Q.1 If three coins are tossed, all possible outcomes are

{HHH, HHT, HTH, THH, TTH, HTT, THT, TTT},

if E1 , E2 , ..., En are n events of a sample space S and if E1 UE2 UE3….. U En = S , then E1 ,


E2 , ...., En are called exhaustive events. Further, if Ei ∩ Ej = φ for i ≠ j i.e. events Ei and
Ej are pairwise disjoint and UEi=S , then events E1 , E2 , ..., En are called mutually
exclusive and exhaustive events, now answers the following

(i) Two events which are mutually exclusive.

(ii) Three events which are mutually exclusive and exhaustive.

(iii) Two events, which are not mutually exclusive.

(iv)Two events which are mutually exclusive but not exhaustive.

(v) Three events which are mutually exclusive but not exhaustive.

90
Q.2. Grandmother of Reena, Simi and Aryan has a bag, which contain 9 discs of
different beautiful colors, of which 4 are red, 3 are blue and 2 are yellow. The discs

are similar in shape and size. Children asked grandmother to give them discs and
grandmother draws a disc at random from the bag.

Calculate the probability that it will be (i) red, (ii) yellow, (iii) blue, (iv) not blue, (v)
either red or yellow.

ANSWERS
1. {HHH, HHT, HTH, THH, TTH, HTT, THT, TTT}
2. {(x, y) : x, y = 1,2,3,4,5,6} or {(1,1), (1,2), (1,3), ..., (1,6), (2,1), (2,2), ..., (2,6), ..., (6, 1),
(6, 2), ..., (6,6)}
3. {HHHH, HHHT, HHTH, HTHH, THHH, HHTT, HTHT, HTTH, THHT, THTH, TTHH,
HTTT, THTT, TTHT, TTTH, TTTT}
4. {H1, H2, H3, H4, H5, H6, T1, T2, T3, T4, T5, T6}
5. {H1, H2, H3, H4, H5, H6, T}.
6. {(1,2), (1,3), (1,4), (2,1), (2,3), (2,4), (3,1), (3,2), (3,4), (4,1), (4,2), (4,3)}
7. {1HH, 1HT, 1TH, 1TT, 2H, 2T, 3HH, 3HT, 3TH, 3TT, 4H, 4T, 5HH, 5HT, 5TH, 5TT, 6H,
6T}
1 1 1 7
8.. {RW, WR, WW} 9. 4 10. (i) 0.93 (ii) 0.07 (iii) 0.24 (iv) 0.39 11. (i)8 (ii) 2(iii) 8
1
(iv)8
1 1 8 6 7 1 1 5
12 . 4 , 2 13. It is not mutually exclusive. 14. 15 15. 13 , 13 16. 0.6 17. , , .
3 2 6

18. (i) 19/30 (ii ) 11/30 ( iii ) 2/15 19 (i) 17/33 (ii) 16/33 20. (1) 2/5
ii) 3/8
21 . i) .0.87 (ii) 0.98 (iii)0.11 .

91
Sample Question Papers
(Blue Print)
CLASS: XI
Session: 2022-23
Mathematics (Code-041)
Time Allowed: 3 Hours Maximum Marks: 80
MCQs Case
& AR LA(5 Study Unit
Name of the VSA (2 SA (3
S. Qs mark based Total Wise
Chapter marks) marks)
No (1 s) Questi Total
mark) ons
1 Sets
2 Relations &
--- --- --- 1*(5) --- 1(5)
Functions
3(8)
3 Trigonometric 1(ASR)
1*(2) --- --- --- 2(3)
Functions (1)
4 Complex
numbers and
1(1) --- --- --- --- 1(1)
quadratic
6(10)
equations
5 Linear
4(4) --- --- 1(5) --- 5(9)
inequations
6 Permutations
and 2(2) 1(2) --- --- --- 3(4)
combinations
7 Binomial
--- 1(2) --- --- 2(8) 3(10)
theorems 15(35)
8 Sequences and
2(2) --- 2+1*(9) --- --- 5(11)
series
9 Straight lines --- --- --- 1(5) --- 1(5)
10 Conic sections 2(2) --- 1*(3) --- --- 3(5)
11 Introduction to
Three
3(3) 1+1*(4) --- --- --- 5(7)
Dimensional
Geometry 8(14)
12 Limits and 1(1)+1
derivatives (ASR) --- --- 1*(5) --- 3(7)
(1)
13 Statistics 2(2) --- 1(3) --- --- 3(5)
14 Probability 6(13)
1(1) --- 1*(3) --- 1(4) 3(8)
Total No of Questions 20(20) 5(10) 6(18) 4(20) 3(12) 38(80) 38(80)
# No. of questions (Marks) * Internal Choice Questions, ASR=Assertion Reason Based

92
Sample Question Paper -1
CLASS: XI
Session: 2022-23
Mathematics (Code-041)

Time Allowed: 3 hours


Maximum Marks: 80
General Instructions:
1. This Question paper contains - five sections A, B, C, D and E. Each section is
compulsory. However, there are internal choices in some questions.
2. Section A has 18 MCQ’s and 02 Assertion-Reason based questions of 1 mark
each.
3. Section B has 5 Very Short Answer (VSA)-type questions of 2 marks each.
4. Section C has 6 Short Answer (SA)-type questions of 3 marks each.
5. Section D has 4 Long Answer (LA)-type questions of 5 marks each.
6. Section E has 3 source based/case based/passage based/integrated units of
assessment
(4 marks each) with sub parts.

SECTION A
(Multiple Choice Questions)
Each question carries 1 mark
1 Let A = { 1, 2, 3, 4, 5, 6}, B = { 2, 4, 6, 8 }. Find B – A
(a) {4,6} (b) {6}
(b) {2,4,6,8} (d) {8}
2 If set 𝐴 = {a, b, c, d, e} then number of proper subsets are
(a) 3 2 (b) 1 (c) 3 0 (d) 3 1
3 If (x + 1, y – 2) = (-3, -1), find the values of x and y.
(a) x = 2 and y = 3 (b) x = 2 and y =- 3
(c) x = -4 and y = 1 (d) x = -4 and y = -1
4 If 4x + i(3x – y) = 3 + √(– 4), where x and y are real numbers, then find the
values of x and y.
3 1 3 1
(a) x = and y = - (b) x = and y =
4 4 4 4
1 1 3 5
(c) x = and y = (d) x = 4 and y = 4
4 4

5 Express the complex number z = (2 + i) / [(1 + i) (1 - i)] in ( x + i y) form.


1 3
(a)1 + 2 𝑖 (b) 2 + 2 𝑖
(c)1 + 𝑖 (d) 1 − 𝑖
6 Solution of the equation is 2(2𝑥 + 3) − 10 ≤ 6(𝑥 − 2)
(a) x ∈ [ 4 , ∞) (b) x ∈ [ - 4 , ∞) (c) x ∈ (4 , ∞) (d) x ∈ (- ∞, 4]
7 Solve : −11 ≤ 4𝑥 − 3 ≤ 13
(a) x ∈ [2, 4) (b) x ∈ [ - 2 , 4] (c) x ∈ (2, 4) (d) x ∈ (2, 4]
8 1 1 𝑥
Find x if, 4! + 5! = 6!

93
(a) 42 (b) 24 (c) 30 (d) 36
9 𝑤ℎ𝑎𝑡 𝑖𝑠 𝑡ℎ𝑒 𝑛𝑢𝑚𝑏𝑒𝑟 𝑜𝑓 𝑡𝑒𝑟𝑚𝑠 𝑖𝑛 𝑒𝑥𝑝𝑒𝑛𝑠𝑖𝑜𝑛 𝑜𝑓 (𝑥 + 3𝑦) − (𝑥 − 3𝑦)8
8

(a) 8 (b) 16 (c) 18 (d) 4


10 9𝑛+1
− 8𝑛 − 9 𝑖𝑠 𝑑𝑖𝑣𝑖𝑠𝑖𝑏𝑙𝑒 𝑏𝑦
(b) 16 (b) 64 (c) 32 (d) 8
11 What is the equation of line which cuts of intercepts 3 and 2 on X and Y- axes.
(a) 2x+3y-6=0 (𝑏)2x + 3y + 6 = 0
(c) 2x - 3y-6=0 (𝑑) 2x − 3y + 6 = 0
12 Find the slope of the lines: (a) Passing through the points (3, – 2) and (–1, 4)
3 1 2 3
(a)− 2 (b) − 2 (c) − 3 (d) 2
13 x +1 −1
What is the limit: lim
x→0 x
1 1
(a) 2 (b) − 2
1
(c ) 3 (d) 0
14 Find the value of lim sin 𝑎𝑥
𝑠𝑖𝑛𝑏𝑥 𝑥→0
𝑎 1 𝑎 𝑏
(a) (b) (c) − 𝑏 (d)
𝑏 𝑎𝑏 𝑎
15 Find the derivative of f(x) =
𝑥+1
𝑥
1 1
(a) (b)− 𝑥 2
𝑥2
1 1
(c) − 𝑥 (d) 𝑥
16 Find the value of x if mean of the 3, 8, x, 5, 10 is 6
(a) 4 (b) 5 (c) 6 (d) 3
17 The variance of the data is 121 , then the standard deviation of the data is
(a) 12 (b) 11 (c) 120 (d)110
18 A bag contains 3 red balls, 5 white balls and 7 black balls. What is the
probability that a ball drawn from the bag at random will be neither red nor
black?
(a) 1/5 (b) 1/3 (c ) 7/15 (d) 8/15
ASSERTION-REASON BASED QUESTIONS
In the following questions, a statement of assertion (A) is followed by a
statement of Reason (R). Choose the correct answer out of the following
choices.
(e) Both A and R are true and R is the correct explanation of A.
(f) Both A and R are true but R is not the correct explanation of A.
(g) A is true but R is false.
(h) A is false but R is true.
19 Assertion (A): If (4x+3,y) = (3x+5 , -2) then x=2 and y = -2
Reason (R) : If A= {-1,3,4} then AXA = {(-1,-1), (-1,3), (-1,4), (3,-1), (4,-1),
(3,4)}
20 Assertion (A) : the value of sin (-6900) cos(-3000 ) +cos(-7500 ) sin (-2400 ) = 1
Reason (R): The values if sine and cosine is negative in third and fourth
quadrant respectively

94
SECTION B
This section comprises of very short answer type-questions (VSA) of 2
marks each
21 If A= {1,3,5,7,11,13,15,17} , B= {2,4,6,……,18} and N is the universal set, then
find (A’U(AUB)∩B’)
22 Find the domain for which the functions f(x)= 2x2-1 and g(x) = 1-3x are equal.

OR

Find the domain of the function 𝑓(𝑥) = √9 𝑥 − 𝑥 2


23 Prove that: tan20° tan40° tan80° = tan60°

24 If the end points of a diagonal of a square are (1, -2, 3) and (2, -3, 5) then find
the length of the side of square?

OR
Find the points on the y- axis which are at a distance of 3 units from the point
(2, 3, -1)

25  2 −
1 
Evaluate lim  1− x2
x→1  1− x 
SECTION C
(This section comprises of short answer type questions (SA) of 3 marks
each)
26 Define a relation R on the set N of natural numbers by R = {(x, y) : y = x + 5, x is
a natural number less than 4; x, y ∈N}. Depict this relationship using roster
form. Write down the domain and the range.

27 What is value of 𝑥 If (1+𝑖)𝑥 = 1


1−𝑖
OR
𝑎+𝑖𝑏
If x + iy =𝑎−𝑖𝑏 , prove that x 2 + y 2 = 1.
28 How many litres of water will have to be added to 1125 litres of the 45%
solution of acid so that the resulting mixture will contain more than 25% but
less than 30% acid content?

29 Find (x + 1)6 + (x – 1)6 . Hence or otherwise evaluate ( √2 + 1)6 + ( √2 – 1)6

30 Find the equation of straight line passing through the point (1, 2) and parallel
to the line y = 3x + 1.
OR
Find the value of k if the lines x + (k-1)y + 1 = 0 and 2x + k2y – 1 = 0 are
perpendicular to each other.
31 𝑥 2 𝑐𝑜𝑠 𝜋/6
Find the derivative of with respect to x.
𝑠𝑖𝑛𝑥
OR

95
𝑥+1
Find the derivative of from first principle.
𝑥−1

SECTION D
(This section comprises of long answer-type questions (LA) of 5 marks
each)
32 Find sin , cos 𝑎𝑛𝑑 tan if tan x = , x lies in the third quadrant.
𝑥 𝑥 𝑥 3
2 2 2 4

33 If a, b, c are in A.P. b, c, d are in G.P. and 1, 1 , 1 are in AP, prove that a, c, e are in
c 𝑑 𝑒
G.P.
OR
If a, b, c and d are in G.P. show that (a + b 2 + c 2 ) (b 2 + c 2 + d 2 ) = (ab + bc +
2

cd) 2

34 Find the length of major axis, coordinates of vertices, the foci, the eccentricity &
length of latus rectum of the ellipse 16x2 + y2 =16
OR
Find the lengths of the axis, the coordinates of the vertices , the foci, the eccentricity &
length of the latus rectum of the hyperbola 25x2 -36 y2 =225

35 The mean and standard deviation of 100 observations are found to be 20 and
3 respectively, on rechecking it was found that 3 observations 21, 21, 18 were
incorrect. calculate the correct mean and standard deviation if the wrong items
were omitted.
SECTION E
This section comprises of 3 case-study/passage-based questions of 4
marks each with sub part. First two case study questions have three
sub parts (i), (ii), (iii) of marks 1, 1, 2 respectively. The third case study
question has two sub parts of 2 marks each.)
36 Case-Study 1: Read the following passage and answer the questions given
below.

In a Sport club, 100 persons played the different games. Out of them, 10
persons played cricket only, 15 persons played football only, 10 persons
played volleyball only, 30 persons played cricket and volleyball, 25 persons

96
played football and volleyball, 15 persons played cricket and football, 60
persons played volleyball.
(i) The number of persons who played all three Games.
(ii) The number of persons who played cricket.
(iii) The number of persons who did not played any of the above
three games.
OR
The number of persons who played football.

37 Case-Study 2: Read the following passage and answer the questions given
below.

Find the number of arrangements of the letters of the word


INDEPENDENCE. In how many of these arrangements,
(i) Do the words start with P
(ii) Do all the vowels always occur together
(iii) Do the vowels never occur together
OR
Do the words begin with I and end in P?

38 Case-Study 3: Read the following passage and answer the questions given
below.

Two students Anil and Ashima appeared in an examination. The probability


that Anil will qualify the examination is 0.05 and that Ashima will qualify the
examination is 0.10. The probability that both will qualify the examination is
0.02. Find the probability that
(i) Both Anil and Ashima will not qualify the examination.
(ii) Atleast one of them will not qualify the examination.

97
SOLUTION
Sample Question Paper -1
1 (d) B – A = {8}
2 (d)31
No of proper subsets = 2n -1 = 25 -1= 31
3 (c) x = - 4 and y = 1
4 (b)
3
x = 4 and y = 4
1

(Equating the real and the imaginary parts , we get 4x = 3, 3x – y = 2 )

5 (a)1 + 2 𝑖
1

6 (a) x ∈ [ 4 , ∞)
7 (b) x ∈ [ - 2 , 4]
8 (d) 36
6! 6!
( 4! + 5! = 𝑥 ⇒ 𝑥 = 6 × 5 + 6 = 36 )
9 (d) 4
𝑛
If n is even the {(x + y)n –(x - y)n)} has 2 terms
10 (b) 64 ∴ [(1+8)n+1 -8n-9] ⇒ {1+(n+1)8 +… higher terms of 8 } -8n-
9
𝑥 𝑦
11 (a) 2 x + 3 y - 6 = 0 ( ∴ +2=1 )
3
12 (a) − 2
3

13 (a) 2
1

𝑎
14 (a) 𝑏
15 (b)− 𝑥 2
1

16 (a) 4
17 (b) 11
18 ((b) 1/3

19 (c) A is true but R is false.

20 (c) A is true but R is false

21 Clearly
(AUB)∩ 𝑩’ = A and A’ U A=N

22 f(x)= g(x) ⇒ x = - 2, ½
OR
[0,9]

23 LHS=
sin20° sin40° sin80°
cos20° cos40° cos80°
(2 sin40° sin80°) sin20° (cos40°− cos120°) sin20°
= =
(2 cos40° cos80° ) cos20° (cos40°+ cos120° ) cos20°

98
(cos40°− cos(90+30°)) sin20° (cos40°+1/2) sin20°
= =
(cos40°+ cos(90+30° )) cos20° (cos40°−1/2 ) cos20°
(2cos40°+1) sin20° 2cos40° sin20°+ sin20°
= =
(2cos40°−1 ) cos20° 2cos40° cos20°− cos20°
sin60°− sin20°+ sin20° sin60°
= = = tan 60° =RHS, Hence Proved
cos60°+ cos20°− cos20° cos60°
24 √3
OR

either (0, 1, 0) or (0, 5, 0)


25 1/2

26 Domain = {1,2,3} , range = {6,7,8}


27
x = 4n where n ∈ 𝐼

OR
Find x-iy then evaluate x2 + y2 , by multiplying (x+iy) (x-iy)
28 562.5< x < 900
29 2(x6+15x4+15x2+1), 198
30 y – 2 = 3 (x – 1) ⇒ y=3x-1 (if lines are parallel then slops are equal)
OR
k = - 1 (∴ m1 m2 = -1 )
31 √3
{2x cosec x – x2 cot x cosec x
2
OR
−2
(𝑥 − 1)2

32 3 2 25
𝑠𝑒𝑐 2 𝑥 = 1 + 𝑡𝑎𝑛2 𝑥 = 1 + ( ) =
4 16
5
sec 𝑥 = ±
4
5
sec 𝑥 = − 𝑎𝑠 sec 𝑖𝑠 𝑛𝑒𝑔𝑎𝑡𝑖𝑣𝑒 𝑖𝑛 𝑡ℎ𝑒 𝑡ℎ𝑖𝑟𝑑 𝑞𝑢𝑎𝑑𝑟𝑎𝑛𝑡
4
−4
So, cos 𝑥 =
5
−4
𝑥 1−cos 𝑥 1− 9
Now, 𝑠𝑖𝑛2 2 = = 5
= 10
2 2
𝑥 3  𝑥 3
Thus, sin 2 = (2 < 2 < 4 )
√10
−4
𝑥 1 + cos 𝑥 1+
𝑐𝑜𝑠 2 = = 5 = 1
2 2 2 10
𝑥 −1  𝑥 3
Thus, cos 2 = (2 < 2 < 4 )
√10
𝑥
𝑥 sin 2
tan = = −3
2 cos 𝑥
2

99
𝑥 3 𝑥 −1 𝑥
sin = , cos = 𝑎𝑛𝑑 tan = −3
2 √10 2 √10 2
33 2b=a+c,
c2=bd,
2/d=1/c+1/e
then prove that c2 = ae
OR
Let b=ar. c=ar2, d=ar3
Prove LHS= RHS

34 Major axis +8, Minor axis=2, eccentricity= √15/4 ,Foci (0, ±√5) Vertex
(0, ±4 ) latus rectum=1/2
OR
Transverse ais =6 ,Conjugate axis= 5, e=√61/6 ,foci (±√61/2,0)
Latus rectum= 25/6

35 MEAN= 20,
STANDARD DEVIATION = 3.035

36
(i) 5
(ii) 50
(iii) 5
OR
60
37 There are 12 letters, of which N appears 3 times, E appears 4 times
and D appears 2 times and the rest are all different. Therefore The
12!
required number of arrangements 3!4!2! =1663200
(i) Let us fix P at the extreme left position, we, then, count the
arrangements of the remaining 11 letters. Therefore, the
11!
required number of words starting with P are = 3!2!4! = 138600
(ii) There are 5 vowels in the given word, which are 4 Es and 1 I.
Since, they have to always occur together, we treat them as a
single object EEEEI for the time being. This single object
together with 7 remaining objects will account for 8 objects.
These 8 objects, in which there are 3Ns and 2 Ds, can be
rearranged in 8! /(3! 2!) ways. Corresponding to each of these
arrangements, the 5 vowels E, E, E, E and I can be rearranged
in 5!/ 4! ways. Therefore, by multiplication principle, the
8! 5!
required number of arrangements 3!2! × 4!= 16800
(iii) The required number of arrangements = the total number of
arrangements (without any restriction) – the number of
arrangements where all the vowels occur together.
= 1663200 – 16800 = 1646400
OR

100
Let us fix I and P at the extreme ends (I at the left end and P at the
right end). We are left with 10 letters. Hence, the required number
of arrangements
10!
= = 12600
3! 2! 4!
38 Let E and F denote the events that Anil and Ashima will qualify the
examination, respectively.
Given that P(E) = 0.05, P(F) = 0.10 and P(E ∩ F) = 0.02.
Then
(i) The event ‘both Anil and Ashima will not qualify the
examination’ may be expressed as E´ ∩ F´. Since, E´ is ‘not E’, i.e.,
Anil will not qualify the examination and F´ is ‘not F’, i.e.,
Ashima will not qualify the examination.
Also E´ ∩ F´ = (E ∪ F)´ (by DE Morgan’s Law)
Now P(E ∪ F) = P(E) + P(F) – P(E ∩ F) or P(E ∪ F) = 0.05 + 0.10 – 0.02
= 0.13 Therefore P(E´ ∩ F´) = P(E ∪ F)´ = 1 – P(E ∪ F) = 1 – 0.13 = 0.87
(ii) P (atleast one of them will not qualify) = 1 – P(both of them will
qualify) = 1 – 0.02 = 0.98

101
Sample Question Paper -2
CLASS: XI
Session: 2022-23
Mathematics (Code-041)

Time Allowed: 3 hours Maximum Marks: 80


General Instructions:
7. This Question paper contains - five sections A, B, C, D and E. Each section is
compulsory. However, there are internal choices in some questions.
8. Section A has 18 MCQ’s and 02 Assertion-Reason based questions of 1 mark
each.
9. Section B has 5 Very Short Answer (VSA)-type questions of 2 marks each.
10. Section C has 6 Short Answer (SA)-type questions of 3 marks each.
11. Section D has 4 Long Answer (LA)-type questions of 5 marks each.
12. Section E has 3 source based/case based/passage based/integrated units of
assessment
(4 marks each) with sub parts.

SECTION A
(Multiple Choice Questions)
Each question carries 1 mark
1 If A and B are any two sets, then
(a) A-B =A- (A∪ 𝐵) (b)A-B =A- (A∩ 𝐵)
(c)A-B =A- (A−𝐵) (d) A-B =B- (A∩ 𝐵)
2 Roster form the set A ={x: x ε Z, x2<20} is
(a) {0,-1,1,-2,2,-3,3,-4,4} (b) {,-1,1,-2,2,-3,3,-4,4}
(c) {0,-1,1,-2,2,-3,3,-4,} (d) {,-1,1,-2,2,-3,3,-4,}
3 If 𝑓(𝑥) = |𝑥|,x is real number then the range of 𝑓 is
(a)set of all non negative integers (b)set of all negative integers
(c) set of all non negative real numbers (d) set of all negative real numbers
4 What is the argument of a complex number −1 − 𝑖 ?.
(a) 1350 (b) -450 (c) 450 (d) None of The Above
5 Value of 𝑖 −35
is…..
(a) 1 (b) -1 (c) -i (d) i
6 Solution of the inequality x + x/2 +x/3 < 11 is…
(a) (-∞, 6] (b) [-∞, 6) (c) [-∞, 6] (d) (-∞, 6)
7 Solution of the inequality for x is
7𝑥
-3 ≤ 4 − 2 ≤ 18
(a) [-4,2] (b) [-4,2) (c) (-4,2] (d) (-4,2)
8 If 8C x = 8C1then x will be:

(a) 6 (b) 5 (c) 7 (d) 4


9 Number of terms in the expansion (2 −
3 99
) are
𝑥
(a) 99 (b) 98
(c) 100 (d) 101

102
10 Last term in the expansion of (2𝑥 2 − 1 10
) is
𝑥
−1 1
(a)𝑋 10 (b)𝑋 10
−1 1
(c)𝑋 3 (d)𝑋 3
11 Equation of the line that has y-intercept 4 and is perpendicular to the line y = 3x -2 is
1 1
(a) y = 3 𝑥 +4 (b) y = −3 (𝑥 +4)
(c) y = 3𝑥 +4 (d) y = -3𝑥 +4
12 Find the value of x for which the points (x,-1), (2,1) and (4,5) are collinear.
3
(a) 1 (b) -1 (c )0 (d) 2
13 1
lim𝑥 𝑠𝑖𝑛 𝑥 is equal to.
𝑥→0
(a)0 (b)1 (c)-1 (d)not exist
14 Ifa and b are fixed none zero constant, then the derivative of 𝑎 − 𝑏 is ma+nb,where
4 2 𝑥 𝑥
(a) m=4x3 ,n=-2/x3 (b) m=-4/x5 ,n=-2/x3
(c)m=-4/x5 ,n=-2/x3 (d) m=4x3 ,n=2/x3
15
: =

(a)1/ (b) 2 / (c) 0 (d) limit does not exist

16 Standard deviation of 15 items is 6 and if each item decreased by 1,then new Standard
deviation will be
91
(a) 5 (b)7 (c) (d)6
15
17 Variance of first n natural no is..
n(n+1) n(n−1) (n2 +1) (n2 −1)
(a) 2
(b) 12
(c) 12
(d) 12

18 In a non leap year, what is the probability of having 53 Tuesdays or 53 Wednesday


(a)1/7 (b)2/7 (c)3/7 (d)4/7
ASSERTION-REASON BASED QUESTIONS
In the following questions, a statement of assertion (A) is followed by a statement of
Reason (R). Choose the correct answer out of the following choices.
(i) Both A and R are true and R is the correct explanation of A.
(j) Both A and R are true but R is not the correct explanation of A.
(k) A is true but R is false.
(l) A is false but R is true.

19 Assertion (A) All functions are relation


Reason (R) All relations are function
20 Assertion (A)tan(α + β) = 𝑡𝑎𝑛α+tanβ
1−𝑡𝑎𝑛αtanβ
𝜋
Reason(R) If tanα= ½ and tanβ= 1/3 ,then the value of α+β= 4

SECTION B

103
This section comprises of very short answer type-questions (VSA) of 2 marks
each
21 If U={1,2,3,4,5,6,7,8,9,10} A = {3, 6, 9, },
B = { 4, 8,}, C = { 2, 4, 6, 8, 10 }, D = {5, }; find
(i) A-(B − C)c (ii) D∩ B
22 Find the domain and range of function 𝑓(𝑥) = √16 − 𝑥 2 .
OR
Write 𝐴 × 𝐴 × 𝐴, if A={0,1}.
.
23 Prove that
3 3
cos + x  − cos − x = − 2 sinx
4  4 
OR

Find the value of tan150

24 Find the ratio in which the line segment joining the points (6, 10, –8) and
(4, 8, 10) is divided by the XY- plane.
OR
Find the points on the X-axis, which are at a distance of 2 units from the point (1, -
2, 3).
25 dy
If y = (x cos x ) find
dx

SECTION C
(This section comprises of short answer type questions (SA) of 3 marks each)
26 Define Signum Function, and draw its graph and hence find its domain and range.
27 1+ 3i
Find the modulus and argument of the complex number.
1− 2i
OR
3+2isinθ
Find the real ‘θ’ such that 1−2isinθ is purely imaginary
28 The marks obtained by a student of class XI in first and second terminal examination
are 62 and 48 respectively. Find number of minimum marks he should get in the annual
examination to have an average of at least 60 marks
OR
Solve the following inequalities:-
2x + 1 x+7
> 5, >2
7x − 1 x−8

29 Prove that ∑𝑛
𝑟=0 3 Cr=4
𝑟n n

OR
Using binomial theorem, indicate which number is larger (1.1)10000 or 1000.
30 A line perpendicular to the line segment joining the points (1, 0) and (2, 3) divides it
in the ratio 1: n. Find the equation of the line.
31 Find the derivative of f(x) = sec x with respect to x by first principle.
SECTION D

104
(This section comprises of long answer-type questions (LA) of 5 marks each)
32 Prove that : 𝑐𝑜𝑠2𝑥 𝑐𝑜𝑠 𝑥 − 𝑐𝑜𝑠3𝑥 𝑐𝑜𝑠 9𝑥 = 𝑠𝑖𝑛5𝑥 𝑠𝑖𝑛 5𝑥
2 2 2
33
Let S be the sum , P the product and R the sum of the reciprocals of n terms in GP. Prove
that P2Rn = Sn
OR
The Arithmetic mean of two numbers is three times their Geometric Mean, show that
the numbers are in the ratio of (3 + 2√2) ∶ (3 − 2√2).
.
34 A rod AB of length 15 cm rests between two coordinate axes in such a way that
the end point A lies on x-axis and end point B lies on y-axis. A point P(x, y) is taken on
the rod in such a way that AP = 6 cm. Show that the locus of P is an ellipse
OR
Find the equation of the circle passing through the points (2, 3) and (-1, 1) and whose
centre lies on the line 𝑥 − 3𝑦 = 11.
35 Find the mean , variance and the standard deviation for the following frequency
distribution :
xi 0-10 10-20 20-30 30-40 40-50
fi 5 8 15 16 6
SECTION E
This section comprises of 3 case-study/passage-based questions of 4 marks
each with sub part. First two case study questions have three sub parts (i),
(ii), (iii) of marks 1, 1, 2 respectively. The third case study question has two
sub parts of 2 marks each.)

36

Case-Study1: In a survey of 25 students, it was found that 15 had taken Maths, 12 had
taken Physics, 11 had taken Chemistry, 5 had taken Maths and Chemistry, 9 had taken
Maths and Physics, 4 had taken Physics and Chemistry and 3 had taken all the three
subjects. Find the number of students that had taken
(i) Only Chemistry
(ii) Physics and Chemistry but not Maths
(iii) Maths and Physics but not Chemistry
OR
Only one of the subject

105
37

In how many ways can the letters of the word PERMUTATIONS be arranged if the

(i) Words start with P and ends with S


(ii) Vowels are all together,
(iii) There are always 4 letters between P and S
OR
Consonant are all together
38

Case-Study3:In an interview for a job in call center 5 boys and 3 girls


appeared. If 4 persons are to be selected at random from this group. Find
the probability that
(i) What is the probability that 3 boys and 1 girl are selected.
(ii) What is the probability that 1 boy and 3 girls are selected

106
SOLUTION
Sample Question Paper -2
1 (b) A-B =A- (A∩ 𝐵)

2 (a) {0,-1,1,-2,2,-3,3,-4,4}
3 (a) set of all non negative integers
4 I (d) None of The Above
5 (d) i
6 (d) (-∞, 6)
7 (a) [-4,2]
8 (a)
4 𝑥 1 4 2𝑥 1
Let I=∫2 𝑥 2 +1 𝑑𝑥 = 2 ∫2 𝑥 2 +1 𝑑𝑥 = 2 [log(𝑥 2 + 1)]42
1 1 17 𝑎
= 2 (𝑙𝑜𝑔17 − 𝑙𝑜𝑔5) = 2 𝑙𝑜𝑔 ( 5 ) Using 𝑙𝑜𝑔𝑎 − 𝑙𝑜𝑔𝑏 = log(𝑏)
9 (c)100
10 (b) 1
𝑋 10
11 (b) y = 1
𝑥 +4
−3

12 (a) 1
13 (a)0

14 (c) m=-4/x5 ,n=-2/x3


15 (a) 1/
16 (d) 6
17 (d) (n2 −1)
12
18 (a)1/7
19 (c). A is true but R is false

20 (a) Both A and R are true and R is the correct explanation of A.


.

21 (i) ∅ (ii) ∅
22 Domain [−4 , 4]
Range = [0 , 4]
OR
{(0,0,0),(0,0,1),(0,1,0),(1,0,0),(1,1,0),(1,0,1),(0,1,1),(1,1,1)}

107
23 .

3Π 3Π 3Π 3Π
( 4 +𝑋)+( 4 −𝑋) ( +𝑋)−( 4 −𝑋)
= - 2 sin [ ] .sin [ 4 ]
2 2

= -2 sin(3Π
4
) sin x
Π
= -2 sin ( Π − ) sin x = -2 sin
4
Π
4
sin x

1
=-2 x x sin x =-√2 sin x = R.H.S.
√2

OR
𝑡𝑎𝑛α − tanβ
Tan(α − β) =
1 + 𝑡𝑎𝑛αtanβ
𝑡𝑎𝑛45 − tan30
tan(45 − 30) =
1 + 𝑡𝑎𝑛45tan30
1−1/√3 √3−1
tan(15) = 1+1.1/√3=1+√3 by rationalising 2-√3

24 Let the YZ plane divide the line segment joining points (–2, 4, 7) and (3, –5, 8) in
the ratio k:1.Hence, by section formula, the coordinates of point of intersection

are given by

On the YZ plane, the x-coordinate of any point is zero.

Thus, the YZ plane divides the line segment formed by joining the given points in the ratio
2:3.
OR
(x-1)2+22+32=24 ⇒ (x-1)2=24-13 ⇒ (x-1)2=11
(x-1)=√11 ⇒ Points are

25 dy d(cosx) d (x)
= x. + cosx .
dx dx dx
dy
= −x sinx + cos x
dx
26 1 ,𝑥 > 0
𝑓 ∶ 𝑅 → 𝑅 𝑑𝑒𝑓𝑖𝑛𝑒𝑑 𝑎𝑠 𝑓(𝑥) = { 0 , 𝑥 = 0
−1 , 𝑥 < 0
Diagram

108
Domain = R Range = {−1 , 0 , 1}

27 1 + 3𝑖 1 + 2𝑖 −5 + 5𝑖
× =
1 − 2𝑖 1 + 2𝑖 5
= -1+i
3𝜋
=𝑚𝑜𝑑𝑢𝑙𝑢𝑠 = √2 , 𝐴𝑟𝑔𝑢𝑚𝑒𝑛𝑡 = )
4
OR
3+2𝑖𝑠𝑖𝑛𝜃 1+2𝑖𝑠𝑖𝑛𝜃 3+6𝑖𝑠𝑖𝑛𝜃+2𝑖𝑠𝑖𝑛𝜃−4𝑠𝑖𝑛2𝜃
x =
1−2𝑖𝑠𝑖𝑛𝜃 1+2𝑖𝑠𝑖𝑛𝜃 1+4𝑠𝑖𝑛2 𝜃
( 3−4𝑠𝑖𝑛2𝜃) 8𝑖𝑠𝑖𝑛𝜃
= +
1+4𝑠𝑖𝑛2 𝜃 1+4𝑠𝑖𝑛2 𝜃
If it is purely imaginary number than real part must be zero
3−4𝑠𝑖𝑛2 𝜃
∴ = 0 ⇒ 3-4𝑠𝑖𝑛2 𝜃 = 0
1+4𝑠𝑖𝑛2 𝜃
√3
4𝑠𝑖𝑛2 𝜃 = 3 ⇒ sin𝜃 = 2

28 Let x be the marks obtained by student in the annual exam. Then


62 + 48 + 𝑥
≥ 60
3
110+x≥180
𝑥 ≥70
Thus, the student must obtain a minimum of 70 marks to get an average of at least 60
marks

OR

2𝑥 + 1
>5
7𝑥 − 1
2x+1>35x-5
6>33x
X<2/11
𝑥+7
>2
𝑥−8
X+7>2x-16
7+16>2x-x
X<23
2
Common solution of both inequality is (-∞, )
11

𝑟=0 3 Cr =4
29 We have to prove that ∑𝑛 𝑟n 𝑛

LHS nC030 +nC131+ nC232+ nC333+ nC434+……………

(1+3)n=4𝑛

OR
10000 10000 10000
(1.1)10000
=(1+.1)10000=1+( )(.1)+ +( )(.1)2 +( )(.1)3
1 2 3
10000
+( )(.1)4+…….
4

109
1+10000*1/10+10000(10000-1)/2*1/10+……..
1+1000+…….>1000

30 Let C dividesthe join of A(1, 0) andB(2 , 3) in theratio1: n


2+n 3 
coordinate s of C are , 
 1+ n 1+ n 
3−0
Slopeof AB= =3
2 −1
1
Slopeof requiredline = −
3
2+n
= −  x −
3 1
Equationof requiredline is y − 
1+ n 3 1+ n 
i.e. (1 + n)x + 3(1 + n)y = n + 11
31 f´ (x) = limℎ→0(𝑠𝑒𝑐 𝑥+ℎ−𝑠𝑒𝑐 𝑥)/ℎ

= limℎ→0𝑐𝑜𝑠 𝑥− 𝑐𝑜𝑠 (𝑥+ℎ)ℎ 𝑐𝑜𝑠 𝑥. 𝑐𝑜𝑥 (𝑥+ℎ) = 𝑠𝑖𝑛𝑥𝑐𝑜𝑠2𝑥 = sec x tan x

32 2𝜋 2𝜋
1 + 𝑐𝑜𝑠2𝑥 1 + 𝑐𝑜𝑠 (2𝑥 + 3 ) 1 + 𝑐𝑜𝑠 (2𝑥 − 3 )
+ +
2 2 2
1 2𝜋
(3 + 𝑐𝑜𝑠 2𝑥 + 2 𝑐𝑜𝑠2𝑥. 𝑐𝑜𝑠 )
2 3
1
(3 + 𝑐𝑜𝑠 2𝑥 − 𝑐𝑜𝑠2𝑥)
2

33 𝑎(1 − 𝑟 𝑛 )
𝑆=
1−𝑟
1 1
( 𝑛 − 1) 1 − 𝑟𝑛
𝑅= 𝑎 𝑟 =
1 𝑎𝑟 𝑛−1 (1 − 𝑟)
(𝑟 − 1)
𝑛(𝑛−1)
𝑃 = 𝑎𝑛 𝑟 2
Getting the answer
OR
𝑎+𝑏
Writing 2 = 3√𝑎𝑏
√𝑎+√𝑏 2 √2
Getting 𝑎−√𝑏 = 2 = 1
√ √
√𝑎 √2 +1
Applying C & D and getting = 2−1
√𝑏 √
𝑎 1+2+2√2 𝑎 3+2√2
Squaring both side 𝑏
= 1+2−2 2 ⇒ 𝑏
= 3−2
√ √2

110
34
𝑥
From ∆ PBQ, cos 𝜃 = 9

𝑦
From ∆ PRA, sin 𝜃 = 6

Since cos 2 𝜃 + sin 2 𝜃 = 1

𝑥 2 𝑦 2 𝑥 2 𝑦2
(9 ) + (6 ) = 1 or +
81 36
=1

thus the locus of P is an ellipse.

OR
Let the eqn. of the circle be 𝑥 2 + 𝑦 2 + 2𝑔𝑥 + 2𝑓𝑦 + 𝑐 = 0
Point (2,3 ) lies on it 4𝑔 + 6𝑓 + 𝑐 + 13 = 0
Point (−1 ,1 ) lies on it −2𝑔 + 2 𝑓 + 𝑐 + 2 = 0
𝐶𝑒𝑛𝑡𝑟𝑒 (−𝑔, −𝑓 ) lies on it −𝑔 + 3𝑓 = 11
Solving and getting 𝑔 = −7/2 , 𝑓 = 5/2 𝑐 = 14
eqn. of the circle be 𝑥 2 + 𝑦 2 − 7𝑥 + 5𝑦 + 14 = 0
35 ∑ 𝑓 = 50
𝑖

∑ 𝑓𝑖 𝑑𝑖 = 10

∑ 𝑓𝑖 𝑑𝑖 2 = 50

Mean= 27
Variance formula
Variance = 132 S. D = 11.49

36 Correct Venn Diagram


(i) 5
(ii) 1
(iii) 6 OR 11
37 (i)Words start with P and end with S = 10! = 1814400
2!

8!
(ii) Vowels are together = 2! × 5! = 2419200
10!
(iii) There are always 4 letters between P and S = 2! × 2 × 7 =25401600
OR
6!
Consonants are together = 2! × 7! =1814400
38 Let A : Selection of 3 boys and 1 girl B: Selection of 1 Boy and 3 girls

5𝐶3 𝑋 3𝐶1 3 5𝐶1 𝑋 3𝐶3 1


P(A) = =7 P(B) = = 14
8𝐶4 8𝐶4

111
Sample Question Paper -3
CLASS: XI
Session: 2022-23
Mathematics (Code-041)

Time Allowed: 3 hours Maximum Marks: 80


General Instructions:
13. This Question paper contains - five sections A, B, C, D and E. Each section is
compulsory. However, there are internal choices in some questions.
14. Section A has 18 MCQ’s and 02 Assertion-Reason based questions of 1 mark
each.
15. Section B has 5 Very Short Answer (VSA)-type questions of 2 marks each.
16. Section C has 6 Short Answer (SA)-type questions of 3 marks each.
17. Section D has 4 Long Answer (LA)-type questions of 5 marks each.
18. Section E has 3 source based/case based/passage based/integrated units of
assessment
(4 marks each) with sub parts.

SECTIONA
(Multiple Choice Questions)
Each questioncarries1mark
1 If 𝐴 ⊆ 𝐵 then 𝐵′ − 𝐴′ 𝐼𝑠
(a) A-B (b) 𝐴′ (c) 𝐵′ (d) 𝜙
2 The collection of all intelligent student in a class is
(a) a null set (b) a single set (c) a finite set (d) not a set
3 How many relation define on a set A={1,2,3}
(a) 128 (b) 8 (c) 512 (d) 64
4 The complex number (1+2i)/(1-i) lies in the
(b) I quadrant (b) II quadrant (c)III quadrant (d) IV quadrant
5 3+4𝑖
The multiplicative inverse of is
4−5𝑖

8 31 8 31 8 31
(a) − 25 𝑖 (b) 25 + 25 𝑖 (c) − 25 − 25 𝑖 (d) none of these
25

6 Solution of the inequality |𝑥 − 1| ≤ 2 is

(a)[1,2] (b)[ -1,3] (c) [1,3] (d) none of these


7 Solution of the inequality 7x+3 ≤ 5x+9 is…
(b) (-∞, 3] (b) [-∞, 3) (c) [-∞, 6] (d) (-∞, 6)
8 How many 2 digit even numbers can be formed from the digits 1,2,3,4,5 if the digit
can be repeated ?
(a) 10 (b) 20 (c) 25 (d) none of these

9 10th term in the expansion of (𝑥 − 1)11

112
(b) 11 𝑥 (b) -50 𝑥 (c) -55𝑥 2 (d)none of these

10 Number of terms in the expansion (𝑎 + 𝑏)4 −(𝑎 − 𝑏)4 are


(a) 4 (b 5 (c) 3 (d) 2
11 Slope of the line y-12=0 is
(a) 1 (b) 0 (c) π (d) ) π/2
12 Perpendicular distance of the line y = x -2 is from origin is
(a) 4 (b √2 (c) √3 (d) 2
13 |𝑥|
,𝑥 ≠ 0
Evaluate lim 𝑓(𝑥) 𝑤ℎ𝑒𝑟𝑒 𝑓(𝑥) = { 𝑥
𝑥→0 0 ,𝑥 = 0
(b) 1 (b) -1 (c ) 0 (d) not exist
14 𝑡𝑎𝑛𝑏𝑥
lim is equal to.
𝑥→0 𝑡𝑎𝑛𝑎𝑥
(a) a/b (b) b/a (c) 1 (d) not exist
2
15 Derivative o𝑓(𝑥) = 𝑠𝑖𝑛 𝑥 𝑖𝑠
(a) 2 𝑠𝑖𝑛𝑥 (b) 2𝑐𝑜𝑠𝑥 (c) 𝑠𝑖𝑛2𝑥 (d) 𝑐𝑜𝑠2𝑥
16 The mean deviation about the mean of 6,7,10,12,13,4,8,12 is

(a) 2.50 (b) 2 .75 (c) 1 (d) 2


17 In a leap year, what is the probability of having 53 Monday is
(a)2/7 (b)1/7 (c)3/7 (d)4/7
18 The mean of the squares of the deviation from mean is called
(a) Variance (b) standard deviation (c) median (d) none of these
ASSERTION-REASON BASED QUESTIONS
In the following questions, a statement of assertion (A) is followed by a statement
of Reason(R). Choose the correct answer out of the following choices.
(m) Both A and R are true and R is the correct explanation of A.
(n) Both A and R are true but R is not the correct explanation of A.
(o) A is true but R is false.
(p) A is false but R is true.

19 Assertion (A) Let A={1,2} B={3,4} . Then number of relation from A to B is 16.
Reason (R) If n(A) = p ,n(B)=q then number of relation from A to B is p q.
20 Assertion (A) Domain of sin x is R.
Reason(R) sin x is define for each value of x .

SECTION B
This section comprises of very short answer type-questions (VSA) of 2 marks each
21 Two finite sets having m and n elements. The number of subsets of first set is 56 more
than the total number of subsets of second set. Find the value of m and n.
22 Let R be Relation define as R = {(𝑥, 𝑦): x+2y=10 , x,y are natural number}. Find domain and
range of relation.
OR.
Find domain and range of 𝑓(𝑥) = √𝑥 − 1
23 𝑐𝑜𝑠4𝑥+𝑐𝑜𝑠3𝑥+𝑐𝑜𝑠2𝑥
Prove that = 𝑐𝑜𝑡3𝑥
𝑠𝑖𝑛4𝑥+𝑠𝑖𝑛3𝑥+𝑠𝑖𝑛2𝑥

113
24 Show that points (-4,6,10), (2,4,6), (14,0,-2) are collinear.
OR
Find the equation of the set of points which are equidistance from the points (-1,0,0)
and (1,0,0).
25 tan 2𝑥
Evaluate : 𝑙𝑖𝑚
𝑥→𝜋/2 (𝑥−𝜋/2)
SECTION C
(This section comprises of short answer type questions (SA) of 3 marks each)
26 Draw graph of 𝑓(𝑥) = |𝑥 − 1|and hence find its domain and range.
27 Find the modulus and argument of the complex number 1+𝑖
1−𝑖
OR
𝑢 𝑣
If (𝑥 + 𝑖𝑦)3 = 𝑢 + 𝑖𝑣 𝑡ℎ𝑒𝑛 𝑠ℎ𝑜𝑤 𝑡ℎ𝑎𝑡 𝑥
+ 𝑦 = 4(𝑥 2 −𝑦 2 )
28 A manufacturer has 600 liters of a 12% solution of acid. How many liters of a 30%
acid solution must be added to it so that acid content in the resulting mixture will be
more than 15% but less than 18% .
29 Find the Coefficient of x 6 y 3 in the expansion of (x + 2y)9 .
30
Find the values of k for which the line (k–3) x – (4 – k2) y + k2 –7k + 6 = 0 is
(a) Parallel to the x-axis,
(b) Parallel to the y-axis,
(c) Passing through the origin.
OR
Find the equation of a line perpendicular to the line x-2y+3=0 and passing through the point (1,-2)

31 1 dy
If y = ( 𝑛
) find
𝑠𝑖𝑛 𝑥 dx
OR
𝑐𝑜𝑠𝑥
Find derivative of
1+𝑠𝑖𝑛𝑥

SECTION D
(This section comprises of long answer-type questions (LA ) of 5markseach)
32 Prove that sin 20𝑜 sin 40𝑜 sin 60𝑜 sin 80𝑜 = 3/16
33 Find the sum up to n terms 7+77+777+7777+…………
OR
A G.P. consists of an even number of terms. If the sum of all the terms is 5 times the
sum of terms occupying odd places, thin find its common ratio.
34 An equilateral triangle is inscribed in the parabola y2 =4ax so that one angular
point of the triangle is at the vertex of the parabola. Find the length of each side of
the triangle.
OR
Find the length of major & minor axis , the foci, the eccentricity & length of latus
rectum of the ellipse 9x2 +16 y2 =144

35 Calculate mean and standard deviation of the following frequency distribution.


Class 30-40 40-50 50-60 60-70 70-80 80-90 90-100

114
frequency 3 7 12 15 8 3 2

SECTION E
This section comprises of 3 case-study/passage-based questions of 4 marks each
with sub part. First two case study questions have three sub parts (i), (ii), (iii) of
marks 1, 1, 2 respectively. The third case study question has two sub parts of 2
marks each.)

36 Case Study 1: Read the following passage and answer the question given below
A college awarded 38 medals in football, 15 in basketball and 20 in cricket. If these
medals went to a total of 58 men and only three men got medals in all the three
sports. how many received medals in exactly two of the three sports?

(i) how many received medals in two of the three sports?


(ii) how many received medals in exactly two and three sports?
(iii) how many received medals in exactly two of the three sports?
OR
(iii) how many received medals in at least two of the three sports?

37 Case study 2 : Read the following passage and answer the question given below.
In how many ways can the letters of the word MISSISSIPPI be arranged
Such that

(i) All letters are used


(ii) All I’s are together

115
(iii) All I’s are not together
OR
All S ’s are not together
38 Case study 3: Read the following passage and answer the question given below
In a class of 60 students, 30 opted for NCC and 24 opted for NSS and 24 opted for
both NCC and NSS. If one of these student is selected at random, find the probability
that

(i)The student has opted neither NCC nor NSS.


(ii) The student has opted NCC but not NSS

116
SOLUTION
Sample Question Paper -3
1 (d) 1

2 (d) 1
3 (c) 1
4 (b) 1
5 −1 𝑍̅ 8 31 1
I (c) 𝑍 = |𝑍|2
= − 25 − 25 𝑖

6 (b) 1
7 (a) 1
8 (a) 1
9 (c) 1
10 (d) 1
11 (b) 1
12 (b) 1
13 (d) 1
14 (b) 1
15 (c) 1
16 (b) 1
17 (a) 1
18 (a) 1
19 (c) A is true but R is false 1
20 (a) Both A and R are true and R is the correct explanation of A. 1
.
21 2𝑚 − 2𝑛 = 56 = 8𝑥7 = 8(8 − 1) = 26 −23 3/2
m=6, n=3 1/2
22 Domain =[1, ∞) 1
Range = [0, ∞) 1
OR
R= {(8,1),(6,2),(4,3),(2,4)} 1
Domain = {2,4,6,8} , Range = {1,2,3,4} Codomain = N 1

23 𝑐𝑜𝑠4𝑥 + 𝑐𝑜𝑠2𝑥 + 𝑐𝑜𝑠3𝑥 2𝑐𝑜𝑠3𝑥𝑐𝑜𝑠2𝑥 + 𝑐𝑜𝑠3𝑥


= .
𝑠𝑖𝑛4𝑥 + 𝑠𝑖𝑛2𝑥 + 𝑠𝑖𝑛3𝑥 2𝑠𝑖𝑛3𝑥 𝑐𝑜𝑠2𝑥 + 𝑠𝑖𝑛3𝑥

𝑐𝑜𝑠3𝑥(2𝑐𝑜𝑠2𝑥 + 1)
= 𝑐𝑜𝑡3𝑥
𝑠𝑖𝑛3𝑥( 2𝑐𝑜𝑠2𝑥 + 1)
24 Area of triangle ABC are zero 1
A,B,C are collinear 1
OR
Given AP=BP 1
(𝑥 − 1)2 + (𝑦 − 0)2 + (𝑧 − 0)2 = (𝑥 + 1)2 + (𝑦 − 0)2 + (𝑧 + 0)2 1

117
𝑥=0
25 tan 2𝑥 2tan 2ℎ 1+1
𝑙𝑖𝑚 = 𝑙𝑖𝑚 =2
𝑥→𝜋/2 (𝑥−𝜋/2) ℎ→0 2ℎ
26 Correct graph 2
Domain = R Range = R+ 1
27 1+𝑖 1+𝑖 1
Let Z=1−𝑖 × 1+𝑖 = 0 + 𝑖
1
𝜋
𝑚𝑜𝑑𝑢𝑙𝑢𝑠 = 1 𝐴𝑟𝑔𝑢𝑚𝑒𝑛𝑡 = 2 1
OR
𝑢 + 𝑖𝑣 = 𝑥 + 𝑖𝑦 + 3𝑥 𝑖𝑦 − 3𝑥𝑦 2
3 3 2 1
𝑢 = 𝑥 3 − 3𝑥𝑦 2 , 𝑣 = 3𝑥 2 𝑦 − 𝑦 3 1
𝑢 𝑣 1
+ = 4(𝑥 2 −𝑦 2 )
𝑥 𝑦
28 Let x litres of 30% acid solution is added
15% of (x+60) ≤ 30% of x+12% of 60 ≤ 18% of (x+60) 2
120 ≤ x ≤ 300 1
29 (𝒙 + 𝟐𝒚)𝟗 =𝑥 9 + 9𝑥 8 2𝑦 + 36𝑥 7 4𝑦 2 + ⋯ … 2
Coefficient of 𝑥 6 𝑦 3 = 672 1
30 (a) K=3 1
(b) K=±2 1
(c) K=6 or 1 1
OR
Slope of given line =1/2 1
Slope of perpendicular line = -2 1
Equation of line 2x-y=0 1
31 dy 2+1
= −n 𝑠𝑖𝑛−𝑛−1 𝑥 cos 𝑥
dx
OR
𝑑𝑦 (1 + 𝑠𝑖𝑛𝑥)(−𝑠𝑖𝑛𝑥) − 𝑐𝑜𝑠𝑥(0 + 𝑐𝑜𝑠𝑥) 2
=
𝑑𝑥 (1 + 𝑠𝑖𝑛𝑥)2 1
−1
= 1+𝑠𝑖𝑛𝑥
32 Use formula 1
2𝑠𝑖𝑛𝐴 𝑠𝑖𝑛𝐵 = cos(𝐴 − 𝐵) − cos(𝐴 + 𝐵) 4
Correct proof
33 7 1
𝑆𝑛 = (9 + 99 + 999 + ⋯ … … … + 𝑛 𝑡𝑒𝑟𝑚)
9 1
7 1
𝑆𝑛 = [(10 − 1) + (100 − 1) + (1000 − 1) + ⋯ … … … + 𝑛 𝑡𝑒𝑟𝑚)]
9 1
7
𝑆𝑛 = [(10 + 100 + 1000 + ⋯ … … … + 𝑛 𝑡𝑒𝑟𝑚) 1
9
− (1 + 1 + 1 + ⋯ … … … … . +𝑛 𝑡𝑒𝑟𝑚)]
7 (10𝑛 −1)
𝑆𝑛 = 9 [10 − 𝑛-]
9
OR
Let G.P. has 2n term
𝑎(𝑟 2𝑛 − 1)
𝑆2𝑛 =
𝑟−1 1

118
𝑎[(𝑟 2 )𝑛 −1) 1
Let 𝑆1= sum of terms occupying odd places 𝑆1 = 𝑟 2 −1 1
1
𝑆2𝑛 = 5 𝑆1
1
𝑎(𝑟 2𝑛 − 1) 𝑎(𝑟 2𝑛 − 1)
=5
𝑟−1 𝑟2 − 1
r=4

34 Length of side of triangle =8𝑎√3


OR
Major axis =8 1
Minor axis = 6 1
Foci = (±√7, 0)
1
Eccentricity=√7/4
Latus rectum =9/2 1
1
35 Let assumed mean A= 65 , h=10
∑ 𝑓𝑖 = 50 1

∑ 𝑓𝑖 𝑑𝑖 = −15
1
∑ 𝑓𝑖 𝑑𝑖 2 = 105
1
Mean= 62
1
Variance formula 1
Variance = 201 S. D = 14.18
36 (i) (i) 18 1
(ii) 12 1
(ii) 9 OR 21 2
37 (i) 34650 1
(ii) 840 1
(iii) 33810 OR 33810 2
38 (i) 11/30 2
(ii) 2/15 2

119
Sample Question Paper -4
CLASS: XI
Session: 2022-23
Mathematics (Code-041)

Time Allowed: 3 hours Maximum Marks: 80


General Instructions:
19. This Question paper contains-five sections A, B, C, D and E. Each section is
compulsory. However, there are internal choices in some questions.
20. Section-A has 18 MCQ’s and 02 Assertion-Reason based questions of 1 mark each.
21. Section- B has 5 Very Short Answer (VSA)-type questions of 2 marks each.
22. Section- C has 6 Short Answer (SA)-type questions of 3 marks each.
23. Section-D has 4 Long Answer (LA)-type questions of 5 marks each.
24. Section E has 3 source based/case based/passage based/integrated units of
assessment
(4marks each) with sub parts.

SECTION-A
(Multiple Choice Questions)
Each question carries 1mark
1 Which is not an example of Null set?
(c) 𝑠𝑒𝑡 𝑜𝑓 𝑜𝑑𝑑 𝑛𝑎𝑡𝑢𝑟𝑎𝑙 𝑛𝑢𝑚𝑏𝑒𝑟 𝑑𝑖𝑣𝑖𝑠𝑖𝑏𝑙𝑒 𝑏𝑦 2
(d) set of even prime number
(e) {x: x is a natural number x<5, x>7}
(f) {y : y is a point common to two parallel lines}
2 Let A be the set of all parallelograms, B be the set of rectangles, C be the set of
rhombuses, D be the set of squares and E set of trapeziums. Then A may be
equal to
(a)𝐵 ∩ 𝐶 (b)𝐷 ∩ 𝐶 (c)𝐷 ∩ 𝐸 (d)(𝐵 ∪ 𝐶 ∪ 𝐷) ∩ 𝐴
3 Let n(A)= m, n(B) = n then the total number of non-empty relation from A to B is
(a)mn (b)𝑛𝑚 − 1 (c) 𝑚𝑛 − 1 (d)2𝑚𝑛 − 1
4 Let x and y are real numbers, then x +iy is a non-real complex number if
(a) x = 0 (b)𝑦 = 0 (c)𝑥 ≠ 0 (d)𝑦 ≠ 0
5 If a +ib = c + id then
(a)𝑎2 + 𝑐 2 = 0 (b)𝑏 2 + 𝑐 2 = 0
(c)𝑏 2 + 𝑑2 = 0 (d)𝑎2 + 𝑏 2 = 𝑐 2 + 𝑑 2
6 If x < 5 then
(c) – x < - 5 (b) −𝑥 ≤ −5 (𝑐) − 𝑥 > −5 (d)−𝑥 ≥ −5
7 If x is a real number and |𝑥| < 3, then
(a)𝑥 ≥ 3 (b) −3 < 𝑥 < 3 (c)𝑥 ≤ −3 (d)−3 ≤ 𝑥 ≤ 3
8 n n
If C12 = C8 then n is equal to
(a) 20 (b) 12 (c) 8 (d) 4
9 The total number of terms in the expansion of (𝑥 + 𝑎) + (𝑥 − 𝑎)100 are
100

(c) 101 (b) 100 (c) 51 (d) 202


10 (1.1)10000
which one is smaller between and 1000

120
(a)(1.1)10000 ( b ) 1 0 0 0 (c) both are equal (d) none of these
11 Slope of a line which cuts equal intercepts on coordinate axes is:
(a)1 (b)-1 (c)0 (d) not defined

12 Equation of line passing through (1,2) and parallel to the line y - 3x +1=0
(a) 𝑦 + 2 = 𝑥 + 1 (b) 𝑦 + 2 = 3(𝑥 + 1)
(c ) 𝑦 − 2 = 3(𝑥 − 1) (d) 𝑦 − 2 = 𝑥 − 1
13 lim sin 𝑥
is equal to.
x → π 𝑥−𝜋
(a)1 (b)2 (c)-1 (d)-2
1 𝑑𝑦
14 If y = √𝑥 + , then at x =1 is:
√𝑥 𝑑𝑥
1
(a)1 b) ½ (c) (d) 0
√2
15 𝑑𝑦
If 𝑦 = sin(𝑥 + 9), then the value of 𝑑𝑥 at x =0 is
(a) cos 9 (b) sin 9 (c )−3 (d) 3
16 The mean of first 10 natural numbers is
(a) 5.5 (b) 3.87 (c) 2.97 (d) 2.87
17 If a, b, c, d and e be the observation the mean m and standard deviation is s, then the
Standard Deviation of a+k, b+k, c+k, d+k and e+k is
(a) s (b) 𝑘𝑠 (c) 𝑠 + 𝑘 (d) 𝑠/𝑘
18 Three numbers are chosen from 1 to 20. The probability that they are not consecutive
186 187 188 18
(a) 190 (b) 190 (c) 190 (d) 20
𝐶3
ASSERTION-REASON BASED QUESTIONS
In the following questions, a statement of assertion (A) is followed by a statement of
Reason(R). Choose the correct answer out of the following choices.
(q) Both A and R is true and R is the correct explanation of A.
(r) Both A and R are true but R is not the correct explanation of A.
(s) A is true but R is false.
(t) A is false but R is true.

19 Assertion (A):R= {(1,1), (2,2), (3,3), (4,4), (5,5)} is a function.


Reason (R) : Every element of domain has one and only one image then a relation is
called a function
20 Assertion (A): (1 + tan A)(1 + tan B)= 2
𝜋
Reason(R): A + B = 4
SECTION-B
This section comprises of very short answer type-questions (VSA) of 2 marks each.
21 Let A = {all prime number less than 10} and B = {all odd number less than 10} then
find set 𝐴 − (𝐴 ∩ 𝐵)
OR
Find sets A, B and C such that (𝐴 ∩ 𝐵), (𝐵 ∩ 𝐶)𝑎𝑛𝑑 (𝐴 ∩ 𝐶) are non empty but 𝐴 ∩ 𝐵 ∩ 𝐶 = ∅
22 𝑓(5)−𝑓(1)
Find the value of for 𝑓(𝑥) = 𝑥 3
5−1

23 Draw the graph of sin x and cos x in [0, 2𝜋]


OR
Find the value of sin(n+1)x sin(n+2)x + cos(n+1)x cos(n+2)x

121
24 Find the equation of the set of point P, which are equidistant from (1,2,3) and (3,2, -1).

25 𝑎𝑥+𝑏
Find the derivative of 𝑓(𝑥) = 𝑐𝑥+𝑑
.

SECTION C
(This section comprises of short answer type questions (SA) of 3 marks each)
26 Let f(x) and g(x) be real function defined by f(x) = 2x+1 and g(x) = 4x – 7
(a) For what real number x, f(x) = g(x)
(b) For what real number x, f(x) < g(x).
27 Find the real number of x and y if (x –iy)(3 +5i) is the complex conjugate of (–6 –
24i).
OR
Find the number of non zero integral solutions of |1 − 𝑖|𝑥 = 2𝑥
28 Solve the inequality and represent the solution on number line
5x +1 > -24 , and 5x – 1 < 24
OR
4 6
Solve for x : 𝑥+1 ≤ 3 ≤ 𝑥+1 , (𝑥 > 0)
29 4 4
Find (𝑎 + 𝑏)4 − (𝑎 − 𝑏)4 hence evaluate (√3 + √2) − (√3 − √2)
OR
Show that 16𝑛+1 − 15𝑛 − 16 is divisible by 225
30 Find the equation of line passing through the point ( -3, 5) and perpendicular to
the line segment joining the points (2, 5) and (-3 ,6).

31 Find the derivative of 𝑓(𝑥) = 𝑥 2 sin 𝑥 + cos 2𝑥

SECTION-D
(This section comprises of long answer-type questions (LA) of 5marks each)
32 𝜋 9𝜋 3𝜋 5𝜋
(i) Prove that 2 cos 13 cos 13 + cos 13 + cos 13 = 0
(ii) cot 𝑥 𝑐𝑜𝑡 2𝑥 − cot 2𝑥 cot 3𝑥 − cot 3𝑥 cot 𝑥 = 1

33 The ratio of A.M and G.M of two positive integers ‘a’ and ‘b’ is m : n. show that the
numbers are in (𝑚 + √𝑚2 − 𝑛2 ) ∶ (𝑚 − √𝑚2 − 𝑛2 ).
OR
If A and G be A.M and G.M of two positive numbers, prove that the numbers are
𝐴 ± √(𝐴 − 𝐺)(𝐴 + 𝐺)
34 Find the equation of a circle passing through the points (4,1) and (6,5) whose
centre lie on the line 4x + y = 16
OR
Find the coordinates of foci, vertices, length of major and minor axis and length of
x2 y2
latus rectum in the ellipse 36 + 16 = 1
35 Calculate the mean and standard deviation of the following data
Class Interval 1 – 5 6 – 10 11 – 15 16 – 20 21 - 25
Frequency 3 8 13 18 23
SECTION E

122
This section comprises of 3 case-study/passage-based questions of 4 marks each
with sub part. First two case study questions have three sub parts (i), (ii), (iii) of
marks 1, 1, 2 respectively. The third case study question has two sub parts of 2
marks each.)
36 Case-Study 1: Read the following passage and answer the questions given below.
In a community lunch organised in a school
each student brought some food items except
A. Let B, C and D are 3 other students who
bought food items as B = {1,2, 3, 4}, C= {3, 4, 5}
and D = { 4, 5, 6, 7}
(i) What is 𝐵 ∩ 𝐶 ∩ 𝐷
(ii) Which is empty set?
(iii) Find (𝐵 − 𝐶) ∪ (𝐶 − 𝐷)
OR
(iii) Find 𝐵 ∪ (𝐶 ∩ 𝐷)

37 Case-Study2: Read the following passage and answer the questions given below.

Rohit and Riya are playing with


marbles Rohit has 6 white marbles
with him and Riya has 5 red marble
with her. They mixed all in one bag and
decided to draw 4 marbles from the
bag.
(i) Find the total number of ways in which 4 marbles of any colour can
be drawn.
(ii) Find number of ways if 2 white and 2 red marbles are drawn.
(iii) Find number of ways if 4 marbles of same colour are drawn.
OR
Find number of ways of drawing at least 3 red marbles
38 Case-Study3: Read the following passage and answer the questions given below.

Four candidates A, B, C and D have applied for the job of a sports coach in a school.
If A is twice likely to be selected as B, B and C have same opportunity to be selected
while C is twice likely to be selected as of D.
(i) What is the probability that C will be selected?.
(ii) What is the probability that A is not selected?.

123
SOLUTION
Sample Question Paper -4
1 (b) 1
2 (d) 1
3 (d) 1
4 (a) 1
5 (d) 1
6 (c) 1
7 (b) 1
8 (a) 1
9 (c) 1
10 (b) 1
11 (b) 1
12 (c) 1
13 (c) 1
14 (d) 1
15 (a) 1
16 (b) 1
17 (a) 1
18 (b) 1
19 (a) 1
20 (a) 1
21 For 𝐴 ∩ 𝐵 = {3, 5,7} 1
A- 𝐴 ∩ 𝐵 = {2} 1
OR
Taking any correct set 1
Showing all conditions 1
22 We have𝑓(𝑥) = 𝑥 3 ½
f(5)=125, f(1)= 1 ½
𝑓(5)−𝑓(1) 124 1
= 4 =31
5−1

23 1

1
OR
½
Given expression can be written as cos[nx + 2x – nx – x]
= cos x 1/2
24 PA = PB ½

124
Taking values of P, A and B in distance formula ½
Getting -2x -4y -6z +14 +6x +4y -2z +14 =0 ½
4x – 8z = 0 or x = 2z ½
𝑑 𝑑
25 (𝑐𝑥+𝑑) (𝑎𝑥+𝑏)−(𝑎𝑥+𝑏) (𝑐𝑥+𝑑) ½
By quotient formula 𝑓 ′ (𝑥) = 𝑑𝑥 𝑑𝑥
½
(𝑐𝑥+𝑑)2
(𝑐𝑥+𝑑)𝑎−(𝑎𝑥+𝑏)𝑐 𝑎𝑐𝑥+𝑎𝑑−𝑎𝑐𝑥−𝑏𝑐 𝑎𝑑−𝑏𝑐
= = = (𝑐𝑥+𝑑)2
(𝑐𝑥+𝑑)2 (𝑐𝑥+𝑑)2 1
26 Given 𝑓(𝑥) = 2𝑥 + 1 𝑎𝑛𝑑 𝑔(𝑥) = 4𝑥 − 7 1
(a) Since f(x) = g(x) therefore 2x+1 = 4x -7 => 2x = 8 ➔ x = 4 1
(b) Since f(x) < g(x) therefore 2x+1 < 4x -7 => 2x < 8 ➔ x < 4 1
̅̅̅̅̅̅̅̅̅̅̅̅
27 (x-iy)(3+5i)=−6 − 24𝑖 ½
=(3x+5y)+(5x-3y)=-6+24i ½
By comparing 3x+5y=-6 & 5x-3y=24 1
Solving we get x=3,y=-3 1
Or
Since |1 − 𝑖|𝑥 = 2𝑥 ½
½
(√12 + (−1)2 )𝑥 =2𝑥
𝑥
(√2) =2𝑥 ➔ x=2x or x=0 but we need none zero solution 1
Hence number of none zero solution is zero. 1
28 5x+1 > -24 5x-1<24 1
5x> -25 5x <25 1
x>-5 x<5 1
-5 0 5
OR
Since x>0 therefore x+1>0
4 6 1
≤3≤
𝑥+1 𝑥+1
4 ≤ 3(x+1) ≤ 6
1
4 ≤ 3x+3 ≤ 6
1 ≤ 3x ≤ 3
1
≤𝑥 ≤1 1
3
29 (𝑎 + 𝑏)4 = 𝑎4 + 4𝑎3 𝑏 + 6𝑎2 𝑏 2 + 4𝑎𝑏 3 + 𝑏 4 ½
(𝑎 − 𝑏)4 = 𝑎4 − 4𝑎3 𝑏 + 6𝑎2 𝑏 2 − 4𝑎𝑏 3 + 𝑏 4 ½
Subtracting (𝑎 + 𝑏)4 –(𝑎 − 𝑏)4 = 8𝑎𝑏(𝑎2 + 𝑏 2 ) 1
4 4 1
Now (√3 + √2) − (√3 − √2) = 8 √3√2( 3 + 2) = 40√6
OR
𝑛+1 𝑛+1
16 = (1 + 15) ½
= 1 + (𝑛 + 1)15 + 𝑛+12𝐶 152 + ⋯ + 𝑛+1
𝑛+1𝐶 15
𝑛+1
½
= 15𝑛 + 16 + 152 [ 𝑛+12𝐶 + ⋯ + 𝑛+1
𝑛+1𝐶 15
𝑛−1 ]
1/2
𝑛+1 𝑛+1 𝑛+1
16 − 15𝑛 − 16 = 225[ 2𝐶 + ⋯ + 𝑛+1𝐶 15𝑛−1 ] ½
16𝑛+1 − 15𝑛 − 16 𝑖𝑠 𝑑𝑖𝑣𝑖𝑠𝑖𝑏𝑙𝑒 𝑏𝑦 225 ½
1/2
30 Slope of line joining (2,5) and (-3,6)
6−5 −1 1
m= −3−2 = 5 therefore slope of line perpendicular to it =5
hence equation of line with slope 5 and passing through (-3,5) is 1
y-5=5(x+3) or y=5x+20 1

125
31 Let f(x) =𝑥 2 sin 𝑥 + cos 2𝑥
𝑑 2 𝑑
𝑓 ′ (𝑥) = (𝑥 sin 𝑥) + (cos 2𝑥) 1
𝑑𝑥 𝑑𝑥
2 𝑑 𝑑 2 𝑑 1
=𝑥 𝑑𝑥 sin 𝑥 + sin 𝑥 𝑑𝑥 𝑥 + 𝑑𝑥 cos 2𝑥
=𝑥 2 cos 𝑥 + 2𝑥 sin 𝑥 − 2 sin 2𝑥 1
32 (i) 2 𝑐𝑜𝑠 𝜋 9𝜋 3𝜋 5𝜋
𝑐𝑜𝑠 13 + cos 13 + 𝑐𝑜𝑠 13
13
𝜋 9𝜋 4𝜋 𝜋
= 2 𝑐𝑜𝑠 13 𝑐𝑜𝑠 13 + 2 𝑐𝑜𝑠 13 𝑐𝑜𝑠 13 1
𝜋 9𝜋 4𝜋
= 2 𝑐𝑜𝑠 13 ( 𝑐𝑜𝑠 13 + cos 13 )
𝜋 𝜋 5𝜋 1
=2 cos 13 ( 2 cos 2 cos 26 )
=0 1
(ii) Take cot 3x = cot (x + 2x)
Using cot( x +y) formula expanding 1
Cross multiplication and arranging the terms 1
33

OR

126
1

34 Let the equation be (𝑥 − ℎ)2 + (𝑦 − 𝑘)2 = 𝑟 2 1


Since it is passing through (4, 1) and (6, 5)
Therefore (4 − ℎ)2 + (1 − 𝑘)2 = 𝑟 2 and (6 − ℎ)2 + (5 − 𝑘)2 = 𝑟 2 2
Solving these; h +2k – 11 = 0 and from given 4h + k -16 = 0
Solving these two equation we get h = 3 and k = 4 and r2 =10 1
Hence equation of circle is x2 + y2 -6x- 8y +15 = 0 1
OR
𝑥2 𝑦2
From given equation 36 + 16 = 1, we have a = 6 and b = 4
 C2 = a2 – b2
 36 -16 = 20
 C = 2√5
1
Therefore coordinates of foci = (±𝑐, 0) = (±2√5, 0)
1
Vertices = (±𝑎, 0) = (±6, 0)
1
Length of major axis = 2a = 12 1
Length of minor axis = 2b = 8 1
2𝑏 2
Length of latus rectum = = 16/3
𝑎

35 Class Mid point fi fixi 𝑓𝑖 𝑥𝑖2 2


Interval
0.5- 5.5 3 3 9 27

127
5.5 -10.5 8 8 64 512
10.5 – 15.5 13 13 169 2197
15.5 – 20.5 18 18 324 5832
20.5 – 25.5 23 23 529 12167
Total 65 1095 20735

2
Mean =1095/65=16.85
1
Variance = 𝜎 2 = 𝑁2 [Σ𝑁𝑓𝑖 𝑥𝑖2 − (Σfi xi )2 ]
1 1
= 652 [65 𝑋 20735 − 10952 ]
1
= 652 [ 1347775- 1199025]
1
= 652 [148750]
= 35.21
S.D = √35.21 = 5.93
36 (i) 𝐵 ∩ 𝐶 ∩ 𝐷 = {1,2,3,4} ∩ {3,4,5} ∩ {4,5,6,7} = {4} 1
(ii) Set A 1
(iii)( B –C) ∪ (𝐶 − 𝐷)= { 1, 2} ∪ {3} = {1,2,3} 2
OR
(iii) 𝐵 ∪ (𝐶 ∩ 𝐷) = {1,2,3,4} ∪ {4,5}= {1,2,3,4,5}

37 (i) there are total 11 marbles, 4 can be drawn in 114𝐶 ways 1


11 𝑋 10 𝑋 9 𝑋 8
= = 330 ways
24
(ii) 2 white out of 6 and 2 red out of 5 in 62𝐶 𝑋 52𝐶 ways
6 𝑋5 5𝑋4
= 2 X 2 = 150 ways 1
(iii) 4 marbles of same color that is 4 white or 4 red in 64𝐶 + 5
4𝐶 = 6
2𝐶 𝑋 5
1𝐶
= 15 + 5 = 20 ways 2
Or
At least 3 marbles in 3𝐶 𝑋 1𝐶 + 4𝐶 𝑋 60𝐶
5 6 5

= 52𝐶 𝑋 6 + 51𝐶 𝑋 1
= 10 x 6 + 5 = 65 ways
38 It is given that P(A) =2P(B) OR P(B)= 𝑃(𝐴), Also P(B)=P(C)=2P(D)
2
𝑃(𝐴) 𝑃(𝐴)
THEREFORE =2P(D) OR P(D) = , SINCE P(A) +P(B)+P(C)+P(D)=1
2 4
𝑃(𝐴) 𝑃(𝐴) 𝑃(𝐴) 4
P(A)+ + + =1 or 9P(A)=4 OR P(A) =9 2
2 2 4
𝑃(𝐴) 2
(1) P(C) = =9
2 2
5
(2) P(A’) =1- P(A) =9

128
Sample Question Paper -5
CLASS: XI
Session: 2022-23
Mathematics (Code-041)

Time Allowed: 3 hours Maximum Marks: 80


General Instructions:
1.This Question paper contains - five sections A, B, C, D and E. Each section is
compulsory. However, there are internal choices in some questions.
2.Section A has 18 MCQ’s and 02 Assertion-Reason based questions of 1 mark
each.
3. Section B has 5 Very Short Answer (VSA)-type questions of 2 marks each.
4.Section C has 6 Short Answer (SA)-type questions of 3 marks each.
5.Section D has 4 Long Answer (LA)-type questions of 5 marks each.
6.Section E has 3 source based/case based/passage based/integrated units of
assessment
(4 marks each) with sub parts.

SECTIONA
(Multiple Choice Questions)
Each question carries 1mark
1 Two finite sets have m and n elements. The number of subsets of the first set is
112 more than that of the second set. The values of m and n are, respectively,
(A) 4, 7 (B) 7, 4

(C) 4, 4 (D) 7, 5

2 If A= { 1,5,7,9} and B = { 3,5,7,11} then A-B equals to


(a) { 1,9} (b) { 1, 3 ,9} (c){ 1,5 } (d){ 1,7,11}
3 The relation R defined on the set of natural numbers as R={(a, b):a differs from
b by 3} is given
(a){(1,4),(2,5),(3,6),.....} (b){(4,1),(5,2),(6,3)……….}
(c){(1,3),(2,6),(3,9),.....} (d)None of these

4 𝑖 4𝑛+1 − 𝑖 4𝑛−1
The value of ; where n 𝜖𝑁 is
2
a) i b) -i
c) 1 d) -1
5 2
The Imaginary part of complex no z = 3+4𝑖 ?
8 8
(a) 25 (b) − 25
6 6
( c) 25 (d) − 25
6 The solution of inequalities 6 ≤ −3(2𝑥 − 4) < 12 is
(a) ( 0 , 1 ] (b)(0,1)
(c)(0,1] U [1,∞ ) (d)[1, ∞)
7 The solution set of the inequation3𝑥 + 2 < 5 is
(a) Half plane that contains the origin

129
(b) Open half plane not containing the origin
(c) Whole 𝑥𝑦-plane except the points lying on the line 3𝑥 + 2 < 5
(d)None of these
8 The number of way in which 3 prize can be distributed to 4 children, so that
no child gets all the three prizes ,are
(a) 64 (b) 62 (c) 60 (d)6 1
9 The number of term in the expansion of the expression (1 +√5 x) 7 + (1 -√5 x) 7
(a) 8 (b) 16 (c) 4 (d ) 6

10 Which among the following is /are true


I ) (1.1)1000>100 (II) (1.1)10000<100
(III) (1.1) 10000 >1000 (IV) (1.1)10000< 1000
(a) I and IV are true (b) II and IV are true
( c ) I and III are true ( d ) only IV is true
11 The perpendicular distance of the point (2,3) from the line 3x-4y + 5 =0 is
1 2 4 6
( a) (b ) 5 (c) (d)
5 5 5
12 The equation of a line passing through the points ( -1,-2) with slope -2 is
(a) 2x+ y – 4 = 0 (b)2x+ y +4 = 0 (c)x+ 2y – 4 = 0
( d)x+ 2y +4 = 0
13 1
Find the value of lim 𝑆𝑖𝑛 𝑥
𝑥→0
(a) 0 (b) 1
1
(c) 2 (d) Does not exist
14 32+𝑥 −9
Find lim
𝑥→0 𝑥
(a) 9log3 (b) log3 (c)9 (d) log 9
15 𝑥+1 𝑑𝑦
If y= 𝑥−1 then 𝑑𝑥 equal to
−2 2
(a) (𝑥−1)2 (b) (𝑥−1)2
−2 −1
(c) (𝑥+1)2
( d) (𝑥−1)2
16 Mean deviation about median for continuous frequency distribution is
calculated by using the formula
1 1
(𝑎) 𝑀. 𝐷 = ∑𝑖=𝑛 𝑖=1 𝑓𝑖 |𝑥𝑖 + 𝑀| (b) 𝑀. 𝐷 = ∑𝑖=𝑛
𝑖=1 𝑓𝑖 |𝑥𝑖 − 𝑀|
𝑁 𝑁
1 𝑓 1 𝑓𝑖
(c) M.D=𝑁 ∑𝑖=𝑛 𝑖
𝑖=1 |𝑥 +𝑀| (d) M. D = ∑𝑖=𝑛
𝑖=1 |𝑥𝑖 −𝑀|
𝑖 𝑁
17 The mean for first n natural number is
𝑛 𝑛+1
(a) (b) 2
2
𝑛(𝑛+1)
(c) 2 (d) none of them
18 If P(A) = 0.25 ,P(B) = 0.15 and P(A∩ 𝐵)= 0.12 then P(A or B) is

(a) 0.20 (b) 0.22


(c) 0.40 (d) 0.28

130
ASSERTION-REASON BASED QUESTIONS
In the following questions, a statement of assertion (A) is followed by a
statement of Reason(R). Choose the correct answer out of the following
choices.
(a) Both A and R are true and R is the correct explanation of A.
(b) Both A and R are true but R is not the correct explanation of A.
(c) A is true but R is false.
(d) A is false but R is true.
19 Assertion (A) : If A= { x,y,z} and B = {3,4} then
number of relation from A to B is 25.
Reason ( R ) : No. of relations from A to B is 2𝑛(𝐴) 𝑋 𝑛(𝐵) .
20 Assertion (A) The ratio of the radii of two circles at the center of which two
equal arcs subtends angles of 300 and 700 is 21:10.
Reason(R) Number of radians in an angle subtended at the center of a circle by
an arc is equal to the ratio of the length of the arc to the radius of the circle.

SECTION B
This section comprises of very short answer type-questions (VSA)of 2
marks each
21 IF U={ 1,2,3,4,5,6,7,8,9,10}, A= { 1,3,4} and B = { 5,6} ,Verify that A-B = A ∩ 𝐵′ .
OR
IF U={ 1,2,3,4,5,6,7,8,9,10}, A= { 1,2,3,4} and B = { 5,6,7} ,
Verify that (𝐴 ∪ 𝐵)′ = A ′ ∩ 𝐵′ .
22 Let A= {-1,0,1,2} and B = {3,-2,4} .Let R be the relation defined on A× B such
that
R={ (x, y), x ×y ≥ 0 } Write R as an ordered pair and Find its domain and
Range ?
23 13
If cosec x = − 12 and x lies in third quadrant, Find the value of sin x, cos x, cot x,
tan x ?
OR
Convert 6 radian into degree measure.
24 Find the ratio in which the line joining (2, 4, 5) and (3, 5,-4) is divided by the YZ
–plane.

25 If f(x) = 𝑥100 + 𝑥 99 + 𝑥 98 + 𝑥 97 + 𝑥 96 + … … … + 𝑥 + 1 then find f’(1) .

SECTION C
(This section comprises of short answer type questions (SA) of 3 marks
each)
26 Find Domain and Range of the function y= √25 − 𝑥 2

𝑢 𝑣
27 If (x + iy)3 = u + iv ,then show that 𝑥 + = 4 (𝑥 2 − 𝑦 2 ).
𝑦
OR
𝛽− 𝛼
If 𝛼 and 𝛽 are different complex numbers with |𝛽| = 1, then find |1−𝛽𝛼̅| .

131
28 Solve the following system of inequalities
2𝑥−3 4𝑥
2(2x+3 )-10 < 6(x-2) , 4 + 6≥ 2 + 3
29 Using Binomial Theorem ,expand the following expression (1-3x + 3x2 – x3 )2
OR
Using Binomial Theorem, Evaluate (105)3 .
30 Find the image of the point (3,8) with respect to line x + 3y = 7 .

31 1−𝐶𝑜𝑠𝑥 √𝐶𝑜𝑠2𝑥
Evaluate lim
𝑥→0 𝑥2
SECTION D
(This section comprises of long answer-type questions (LA) of 5 marks
each)
32 Show that Cos2x + 𝐶𝑜𝑠 2 (𝑥 +
2𝜋
) + 𝐶𝑜𝑠 2 (𝑥 −
2𝜋 3
) = 2.
3 3
33 Between 1 and 31, m numbers have been inserted in such a way that the
resulting sequence is an A.P. and the ratio of 7th and (m – 1)th numbers is 5 : 9.
Find the value of m.
OR
𝑎𝑛+1 +𝑏𝑛+1
Find the value of n so that may be the geometric mean between a and
𝑎𝑛 +𝑏 𝑛
b.

34 Find the Co-ordinate of Foci, Vertices, the eccentricity, lengths of latus rectum,
of Hyperbola 9y2 – 4 x2 = 36 .

OR
The cable of a uniformly loaded suspension bridge hangs in the form of a
parabola. The roadway which is horizontal and 100 m long is supported by
vertical wires attached to the cable ,the longest wire being 30 m and the shortest
being 6 m. find the length of a supporting wire attached to the roadway 18 m
from the middle.
35 The mean and variance of eight observations are 9 and 9 , 25, respectively. If six
of the observations are 6 , 7, 10 ,12 ,12 , and 13, find the remaining two
observation.

SECTION E
This section comprises of 3 case-study/passage-based questions of 4
marks each with sub part. First two case study questions have three
sub parts (i), (ii), (iii) of marks 1, 1, 2 respectively. The third case study
question has two sub parts of 2 marks each.)

132
36

Case-Study 1: Two non-empty set A and B are given by


A = { x : x is a letter in “ I LOVE MATHEMATICS “ }
B = { x : x is a letter in “I LOVE STATISTICS” } Based on the above
information

answer the following questions

i)Find the set A-B


ii)Find the no. of proper subsets of set B ?
iii) Find the intersection of A-B and B-A ?
OR
Find the intersection of AUB and A∩ 𝐵.

37

Case-Study2:
The letters of the word ‘COMPUTER’ are arranged in all possible ways

Bases on that answer the following


(1) Find the number of words with or without meaning can be formed?
2) Find the number of words in which vowels comes together?
3) Find the number of words in which vowels do not comes together?
Or
Find the number of words in which vowels occupy odd place?

133
38

Case-Study3: Alex and Justin appeared in an Examination . The Probability that Alex
will qualify the examination is 0.09 and that Justin will qualify the
examination is 0.30. The probability that both will qualify the examination is
0.04 .

On the basis of above information


(i) Find the probability that Alex or Justin will qualify the Examination?
(ii) Find the probability of only one of them will qualify the
examination?
SOLUTION
Sample Question Paper -5
1 b 7, 4

2 a{ 1,9}
3 b {(4,1),(5,2),(6,3)……….}
4 a i
5 8
b −
25
6 a(0,1]
7 a Half plane that contains the origin

8 c 60
9 c 4
10 c - I and III are true
11 1
a
5
12 b 2x+ y +4 = 0
13 d solution does not exist
14 (a) 9log3

15 −2
𝑎
(𝑥 − 1)2
16 b 𝑀. 𝐷 =
1
∑𝑖=𝑛
𝑁 𝑖=1 𝑓𝑖 |𝑥𝑖 − 𝑀|

134
17 𝑛+1
b
2
18 d 0.28

19 (d) A is false but R is true.


20 (d) A is false but R is true.

21 IF U={ 1,2,3,4,5,6,7,8,9,10} A= { 1,3,4} and B = { 5,6} ,Verify that A-B = A ∩ 𝐵′ .


A-B = { 1,3,4} 1
B’ ={ 1,2,3,4,7,8,9,10} 1
A ∩ 𝐵′ = { 1,3,4} 2
1
OR
IF U={ 1,2,3,4,5,6,7,8,9,10} A= { 1,2,3,4} and B = { 5,6,7} , 2
1
(𝑨 ∪ 𝑩)′ = { 8,9,10} 1
A’∩ 𝑩’ = { 8,9,10}
22 R= { (-1,-2) ,(0,3) (0,-2),(0,4) ,(1,3) ,(1,4) , (2,3), (2,4) } 1
Domain = { -1,0,2} ½
Range = { 3,-2,4} ½
23 13
Cosec x = − 12 and x lies in third quadrant,
1
12 5 12 5
Sin x = − 13 , Cos x =− 13, Cot x = tan x = 12 2
5
𝑚𝑎𝑟𝑘𝑠
𝑒𝑎𝑐ℎ
OR
6 radian =
180
x 6 degree 1
𝜋
1080 𝑋 7
= 22 1
= 343038’ 11’’

24 line joining (2, 4, 5) and (3, 5,-4) is divided by the YZ –plane

Let the required point be (x,y,z) which divide the given line segment in K:1 1

3𝑘+2 5𝑘+4 −4𝑘+5


X = 𝑘+1 , y = 𝑘+1 , z = 𝑘+1 1
Given point lying on YZ Plane i.e 2
X= 0
3𝑘+2 1
=0
𝑘+1 2
2
K= − 3 or ( 2:3 external )
25 f(x) = 𝑥100 + 𝑥 99 + 𝑥 98 + 𝑥 97 + 𝑥 96 + … … … + 𝑥 + 1 then find f’(1)
f’(x) =100 𝑥 99 + 99 𝑥 98 + 98𝑥 97 + 97𝑥 96 + 96𝑥 95 + … … … + 1 . 1
f’(1) = 100 + 99 + 98 +97 + 96 + …………………..+ 1
f’(1) = 5050 1

26 Domain = [ -5,5] 1
1
2
Range = [0,5]
1
1
2

135
27 If (x + iy)3 = u + iv
x3-i y3 + 3ix2y -3xy2 = u + iv 1
𝑢
u = x3 -3xy2⇒ 𝑥 = 𝑥 2 − 3𝑦 2
𝑣 1
v = 3x2y - y3 ⇒ = 3𝑥 2 − 𝑦 2
𝑦
𝑢 𝑣 1
+ = 4 (𝑥 2 − 𝑦 2 )
𝑥 𝑦

OR
|𝑍|2 = Z𝑍̅

𝛽− 𝛼 2 𝛽− 𝛼 ̅̅̅̅̅
𝛽− 𝛼
| | = ( )( )
̅
1−𝛽𝛼 ̅
1−𝛽𝛼 ̅
1−𝛽𝛼

28 2(2x+ 3 )-10 < 6(x-2)


4x + 6 -10 < 6x – 12 ⇒ 4x- 4 < 6x – 12 1
4x-6x < -12 + 4 ⇒ -2x < -8 ⇒ x > 4
The solution set is (4, ∞) ……………………(1)
1
2𝑥−3 4𝑥
+ 6≥ 2 + 3 1
4
2𝑥 − 3 + 24 6 + 4𝑥

4 3
2𝑥 + 21 6 + 4𝑥

4 3
3(2x + 21 )≥ 4 ( 6+ 4x) ⇒ 6x + 63 ≥ 24 + 16 x
39
6x-16x ≥ 24-63 ⇒ -10x ≥ -39 ⇒ x ≤ 10
The solution set is (- ∞, 3.9) …………………(2)

The given system of inequalities has no solution.


29 ( 1-3x + 3x2 – x3 )2
= [(1 − 𝑥)3 ]2 1
= (1 − 𝑥)6
= 6C0 - 6C1 𝑥 +6C2 𝑥 2 - 6C3 𝑥 3 +6C4 𝑥 4 - 6C5𝑥 5 + 6C6 𝑥 6
= 1 – 6x + 15x2 – 20x3 + 15 x4 – 6 x5 + x6 1
1
OR
(105)3 = (100 + 5)3
= 100C0 + 100C1 𝑋 5 +100C2X52 + 100C3 X53
= 1 + 100 X 5 + 4950 X25 + 161700 X125
= 1 + 500 + 123750 + 20212500
= 20336751
30 The image of the point (3,8) with respect to line x + 3y = 7 .

𝑏−8 1
Slope of AB =𝑎−3 , while the slope of line (1) = - 3
Since line (1) is perpendicular to AB,

𝑏−8 1
(𝑎−3) 𝑋 (− 3) = -1

136
1/

𝑏−8
=1 2
3𝑎−9
 𝑏 − 8 = 3𝑎 − 9
 3a-b =1 ……..(2)
𝑎+3 𝑏+8
Mid–point of AB = ( 2 , 2 )
1/
The mid-point of line segment AB will also satisfy line (1), 2
Hence ,from equation (1) ,we have
1
𝑎+3 𝑏+8
( 2 ) +3( 2 ) =
7
 a+3+3b+24 =14
 a+3b = -13 ………(3)
1
on solving equations (2) and (3) , we obtain a= -1 and b= -4
thus ,the image of the given point with respect to the given line is (-1,-4 ).
31 1 − 𝐶𝑜𝑠𝑥 √𝐶𝑜𝑠2𝑥
lim
𝑥→0 𝑥2
1−𝐶𝑜𝑠𝑥 √𝐶𝑜𝑠2𝑥 1+𝐶𝑜𝑠𝑥 √𝐶𝑜𝑠2𝑥
=lim X
𝑥→0 𝑥2 1+𝐶𝑜𝑠𝑥 √𝐶𝑜𝑠2𝑥

1−𝐶𝑜𝑠2 𝑥 𝐶𝑜𝑠2𝑥 1−𝐶𝑜𝑠2 𝑥 (2𝐶𝑜𝑠2 𝑥−1)


= lim =lim
𝑥→0 𝑥 2 ( 1+𝐶𝑜𝑠𝑥√𝐶𝑜𝑠2𝑥) 𝑥→0 𝑥 2 ( 1+𝐶𝑜𝑠𝑥√𝐶𝑜𝑠2𝑥)
1−2𝐶𝑜𝑠4 𝑥 +𝐶𝑜𝑠2 𝑥 (1−𝐶𝑜𝑠2 𝑥)(1+2𝐶𝑜𝑠2 𝑥)
= lim = lim
𝑥→0 𝑥 2 ( 1+𝐶𝑜𝑠𝑥√𝐶𝑜𝑠2𝑥) 𝑥→0 𝑥 2 ( 1+𝐶𝑜𝑠𝑥√𝐶𝑜𝑠2𝑥)
𝑆𝑖𝑛2 𝑥 1+2𝐶𝑜𝑠2 𝑥 3
=lim X lim =
𝑥→0 𝑥2 𝑥→0 ( 1+𝐶𝑜𝑠𝑥√𝐶𝑜𝑠2𝑥) 2

32 L.H.S = Cos2x + 𝐶𝑜𝑠 2 (𝑥 +


2𝜋
) + 𝐶𝑜𝑠 2 (𝑥 −
2𝜋
)
3
4𝜋 4𝜋
3 2
1+𝐶𝑜𝑠2𝑥 1+𝐶𝑜𝑠(2𝑥+ ) 1+𝐶𝑜𝑠(2𝑥− )
= + 3
+ 3
2 2 2
1 4𝜋 4𝜋
= 2 [3 + 𝐶𝑜𝑠2𝑥 + 𝐶𝑜𝑠 (2𝑥 + ) + 𝐶𝑜𝑠 (2𝑥 − )] 1
3 3
1 4𝜋
= 2 [3 + 𝐶𝑜𝑠2𝑥 + 2𝐶𝑜𝑠2𝑥 𝐶𝑜𝑠 ] 1
3
1 1 3
= 2 [3 + 𝐶𝑜𝑠2𝑥 − 2𝐶𝑜𝑠2𝑥 𝑋 ] =
2 2 1
33
Let A1, A2, A3, A4, … Am be m A.Ms.between 1 and 31.

Therefore, 1, A1, A2, A3, ……… Am, 31 are in A.P.

Let d be the common difference of AP.

Here, the total number of terms is m + 2 and Tm + 2 = 31


⇒ 1 + (m + 2 – 1) d = 31 1
⇒ (m + 1) d = 30
⇒ d = 30m+1 ……………….(i) 1
A7 = T8 = a + 7d 2
30 𝑚+211
=1+7 X = (FROM EQ. (1))
𝑚+1 𝑚+1
………..
………..

137
30
𝐴𝑚−1 = 𝑇𝑚 = 1 + (m – 1 )d = 1 + ( m- 1) 𝑚+1
𝑚+1+30𝑚−30 1
= (from eq . (1) )
𝑚+1

31𝑚−29
= 𝑚+1 1
(𝑀+211)
𝐴7 𝑚+211
= 𝑚+1
31𝑚−29 = 31𝑚−29
𝐴𝑚−1
𝑚+1

𝐴7 5
But =9 (given )
𝐴𝑚−1
1
1
𝑚+211 5 2
= ⇒ 9m + 1899 = 155m – 145
31𝑚−29 9
⇒ 146m = 2044⇒ m = 2044146 = 14∴ m = 14
1
OR
𝑎𝑛+1 +𝑏 𝑛+1
Given , = √𝒂𝒃
𝑎𝑛 +𝑏 𝑛

1 1
𝑎𝑛+1 +𝑏𝑛+1 𝑎21 𝑏21
 ,
𝑎𝑛 +𝑏 𝑛
= 1
➔ 𝑎 𝑛+1 + 𝑏 𝑛+1 = (𝑎𝑛 + 𝑏 𝑛 ) (𝑎2 𝑏 2 )
1
1 1 1 1
𝑛+ 𝑛+
𝑎 𝑛+1 + 𝑏 𝑛+1 = 𝑎 2 𝑏 +𝑎 𝑏
2 2 2

1 1 1 1
𝑎 𝑛+1 + 𝑏 𝑛+1 − 𝑎𝑛+2 𝑏 2 + 𝑎2 𝑏 𝑛+2 = 0
1 1 1 1
(𝑎 𝑛+1 − 𝑎𝑛+2 𝑏 2 ) + (𝑏 𝑛+1 − 𝑎 2 𝑏 𝑛+2 ) =0
1
1 1 1 1 1 1
𝑛+ 𝑛+
𝑎 2 (𝑎 − 𝑏 ) – 𝑏
2 2 2 (𝑎 − 𝑏 ) = 0
2 2

1 1 1 1
(𝑎𝑛+2 − 𝑏 𝑛+2 ) (𝑎2 − 𝑏 2 ) = 0
1 1 1 1
𝑎𝑛+2 − 𝑏 𝑛+2 =0 ( since 𝑎2 − 𝑏 2 ≠ 0 )
1
1 1
𝑛+ 𝑛+
𝑎 2 = 𝑏 2

1 1
𝑎 𝑛+2 𝑎 𝑛+2 𝑎 0
(𝑏) =1 ⇒ (𝑏 ) = (𝑏 )

On comparing the power of base (a/b) on both sides , we get


1 1
n+ =0⇒n=-
2 2
34 The given equation is 9𝑦 2 - 4𝑥 2 = 36,

It can be written as 1
2

138
𝑦2 𝑥2 𝑦2 𝑥2
9𝑦 2 - 4𝑥 2 = 36 ⇒ − =1 ⇒ − 32 = 1 ……… (1)
4 9 22

On comparing equation (1) with the


standard equation of hyperbola i.e.,
𝑦2 𝑥2 1
− 𝑏2 = obtain a=2 and b=3,
𝑎2 2
We know that 𝑎2 + 𝑏 2 = 𝑐 2 ,
Since 𝑐 2 = 4 +9 =13 ⇒ c= √13
therefore, the coordinates of the foci 1
are (0, ± √13 ),
1
the coordinates of the vertices are (0, 1
±2 ),

𝑐 √13
Eccentricity, e = 𝑎 = 2 1
2𝑏 2 2𝑋9
Length of latus rectum = = =9
𝑎 2
OR
Hence , AB and OC are the shortest wires, respectively, attached to the cable, DF is the
supporting wire attached to the roadway, 18 m from the middle,
100
Here , AB = 30 m, OC =6 m, and BC = =50 m,
2

The equation of the parabola is of the from 𝑋 2 = 4𝑎𝑦( 𝑎𝑠 𝑖𝑡 𝑖𝑠 opening upwards)


The coordinates of point A are ( 50, 30-6) = (50, 24),
Since A (50, 24) is a point on the parabola, 1
50𝑋50 625
(502 ) = 4𝑎 (24) ⇒ a = =
4𝑋24 24
625
1
equation of the parabola, 𝑥 2 = 4 x x y or 6𝑋 2 = 625y,
24

the X- coordinates of point D is 18,


1
hence , at X = 18,
6(18)2 = 625y
6𝑋18𝑋18
1
 y= ⇒ y = 3.11 (approx) DE = 3.11 m
625
1
DF = DE +EF = 3.11 m + 6 m = 9.11 m
Thus ,the length of the supporting wire attached to the roadway 18 m from the
middle is approximately 9.11m.
35
Let the remaining two observation be x and y ,
Therefore ,the observations are 6, 7, 10, 12, 12, 13, x, y,
6+7+10+12+12+13+𝑥+𝑦
Mean , 𝑥̅ = =9
8

139
 60 +x +y = 72
 x + y =12
1 1
variance = 9.25 = 𝑛 ∑8i=1(xi − x)2
1
9.25 = 8 [(−3)2 + (−2)2 + (1)2 + (3)2 + (3)2 + (4)2 + 𝑥 2 + 𝑦 2 −

2 𝑋 9 (𝑥 + 𝑦 ) + 2 𝑋 (9)2 ]
1
9.25 = 8 [9 + 4 + 1 + 9 + 9 + 16 + 𝑥 2 + 𝑦 2 − 18 (12) + 162]
1
9.25 = 8 [ 𝑥 2 + 𝑦 2 − 6]

⇒ 𝑥 2 + 𝑦 2 = 80 2
From (1) , we obtain
𝑥 2 + 𝑦 2 + 2𝑥𝑦 = 144 … … . (3)
From (2) and (3), we obtain 1
2xy = 64 ………(4)
Subtracting (4) from (2), we obtain
𝑥 2 + 𝑦 2 − 2𝑥𝑦 = 80 − 64 = 16
x–y =± 4 ………(5)
1
therefore, from (1) and (5), we obtain
x = 8 and y = 4, when x – y = 4
x = 4 and y = 8, when x – y = -4
thus , the remaining observations are 4 and 8 .
36 i) A-B = { M,H}
ii) the no. of proper subsets of set B = 29 − 1 = 512-1 = 511
iii) A-B = { M,H} AND B-A = ∅
A-B ∩(B-A) = ∅
OR
AUB ={ I,L,O,V,E,M,A,T,H,C,S)
A ∩ 𝐵 = { I,L,O,V,E,S,T,A,C}
(AUB) ∩ (A ∩ 𝐵) = { I,L,O,V,E,S,T,A,C}
37 (1) The no. of words with or without meaning can be formed = 8! =40320
(2) The number of words in which vowels comes together = 6! X 3!= 4320
(3) The number of words in which vowels do not comes together = 40320-
4320=36000.
Or
The number of words in which vowels occupy odd place = 4P3 X 5!= 24
X120=2880

38 (1) The probability that Alex or Justin will qualify the Examination = 0.35
(2) The probability of only one of them will qualify the examination = 0.3 1

140

You might also like